Sunteți pe pagina 1din 98

An Introduction to Mechanics of

Materials

©
Vijay Gupta

Lovely Professional University, Punjab

1
Table of Contents Summary 54

4 FORCES AND MOMENTS IN BEAMS 56


1 STRUCTURES, LOADS AND STRESSES 4 4.1 Introduction 56
1.1 Mechanics of material 4 4.2 Sign convention 57
1.2 Deformation and resisting forces 4 4.3 Loads and supports 58
1.3 Other loadings, stresses and strains 5 4.4 Determining shear forces and bending moments 59
1.4 The concept of stress at a point 7 4.5 General procedure for drawing shear force and bending moment diagrams
1.5 Stress on oblique planes 11 by method of sections 61
1.6 Notation for stress: double-index notation 12 4.6 The area method of drawing the SFDs and BMDs 64
1.7 Equivalence of shear stresses on complementary planes 13 Summary 69
1.8 Stresses in a thin circular pressure vessel 14
1.9 Summary 15 5 STRESSES IN BEAMS 71
5.1 Introduction 71
2 DEFORMATIONS, STRAINS AND MATERIAL PROPERTIES 17 5.2 Relating curvature of the beam to the bending moment 72
2.1 Fundamental strategy of mechanics of deformable mechanics 17 5.3 Composite beams 78
2.2 Statically indeterminate problems 20 5.4 Stresses in beams carrying shear forces 82
2.3 Lateral strain: Poisson ratio 23 5.5 Relating shear stresses to the shear force in a beam 83
2.4 Shear strain 25 5.6 Shear flow in beams 86
2.5 Thermal Strains 27 5.7 Shear centre 88
2.6 Tensile test 28 5.8 Plastic deformations in beams 88
2.7 Idealized stress-strain curves 30 5.9 Strain energy in bending 89
2.8 Pre-stressing 32 Summary 90
2.9 Strain energy in an axially loaded members 33
2.10 Calculating deflections by energy methods: Castigliano theorem 33 APPENDIX B PROPERTIES OF AREAS 93
2.11 Strain energy in an elastic body 38 B.1 First moments of area and centroid 93
Summary 39 B.2 Second moments of area 93
B.3 Parallel axes theorem 94
3 TORSION OF CIRCULAR SHAFTS 41 B.4 Perpendicular axes theorem 96
3.1 Introduction 41
3.2 Relating angle of twist to twisting moment 41
3.3 Stresses and strain in a circular shaft 43
3.4 Hollow shaft 46
3.5 Statically indeterminate shafts 47
3.6 Composite shaft 49
3.7 Torsion of thin-walled tubes 50
3.8 Plastic deformation in torsion 51
3.9 Limit Torque 52
3.10 Strain energy in torsion 53
2
3
Clearly, this part of the beam is not in equilibrium with just the external force, P.
We need additional external (i.e., external to this part of the beam) forces and/or
moments. The open arrows in Fig. 1.1b show the external force and moment
required to balance the applied load P. We will for the time being refer to these
as the reaction forces and the moments.
Where do these forces and
moment come from?
As we apply the force P to
the beam and if these
reactions do not kick in,
the beam will tend to
shear from the stump built
1 Structures, loads and stresses into the wall at the left-
end. The distortion of the
beam so produced results
in generation of material
forces within the beam
that resist this shearing action. When we consider the part of the beam shown in
1.1 Mechanics of material the free body of Fig. 1.1b, these material forces appear as external forces (and
moments) on the beam. Of course, there are equal and opposite reaction on the
The subject matter of a course on mechanics of materials deals with structures. A
stump of the beam built in the wall.
table or a chair is a structure. A building is a structure. A bridge is a structure. A
TV tower is a structure. So is a printed circuit board, the casing of a fax machine, We can summarize the above as:
or the body of a car. Among the many purposes of the various structures, one
common purpose is to resist and/or transmit forces acting on it. By resisting a  The external forces acting on a structure result in deformation of the
force we mean that the structure would not break under the force. The structure structural members.
of a building is designed to resist the loads which include the weight of the  The deformation so caused result in resisting forces within the material
people and things occupying it, the forces of wind acting on it in a storm, even of the members.
the load imposed by an earthquake, and the self-load of the building itself. The  When we consider the equilibrium of a part of the member, these
structure of an aeroplane resists the aerodynamic loads, the weight of its internal forces come into play as external forces and balance the applied
occupants (including the dynamic loads during acceleration and deceleration), forces or moments.
the load imposed by the thrust produced by the engines, and of course the weight
of the structure itself.
1.2 Deformation and resisting forces
How does a structure resist loads?
Consider a vertical rod anchored as shown in Fig. 1.2. It is common knowledge
that when you apply a longitudinal force P to this rod, it elongated a definite
amount (depending on its dimensions and its material). Consider a portion of the
Consider a simple case of a cantilever beam loaded as shown in Fig. 1.1. If the rod enclosed by the broken line rectangle. The free body diagram (FBD) of this
beam is in equilibrium, the net force or moment on the beam or on any part of it part is shown in Fig. 1.1b. Since the rod is in equilibrium after the elongation,
must be zero. Let us consider the part of the beam within the dashed-line box there must be a force that balances the applied force P. Where does that force
shown. This is known as a free-body diagram (FBD). come from? Clearly, there are internal forces which are holding this part from
running away from the rest of the member. These internal forces as seen in the
previous section are the consequence of the distortion produced in the bar. Now
4
more the force we apply, more is the materials. Steel has about the largest value of the elastic modulus of about 200
elongation, suggesting that the resisting GPa1. Cast Iron has about half this value. Aluminium is still lower at 70 GPa.
force that develops in the rod depends
The summary statements of the previous section can now be recast as:
on the elongation. Robert Hooke, a
British scientist is credited to be the first  The external forces acting on a structure result in strains in the structural
to explore the relationship between the members.
resisting force and the elongation. He  The strains so produced result in stresses within the material of the
found out in 1678 that for a given members.
material, the resisting force does not  The stresses, for the most part, are proportional to the strains produced.
depend on the elongation but on the  The constant of proportionality is termed as the modulus of elasticity.
relative elongation produced. He
introduced the term strain to denote the
elongation relative to the original length 1.3 Other loadings, stresses and strains
of the bar. If l denotes the original
Fig. 1.2 A bar loaded
length, and δ the elongation, the strain is
longitudinally
defined by
Strain, (1.1)
Strain is dimensionless and has no units.
Hooke also found out that it is not the force, but the intensity of force measured
as the force per unit area that needs to be considered. He called it stress. If A is
the area of cross-section of the bar, the stress is defined as the resisting force P
divided by A. Fig 1.3. The columns supporting a highway deck, the boom of a
crane and a foundation block are all structural members in
Stress, (1.2) compression
Stress has the dimensions of force per unit area (hydraulic pressure, too, has the
same dimensions) and has SI units of Newton per meter squared (M/m2) which is We had in the previous section considered one kind of load that tends to elongate
termed as Pascal and abbreviated as Pa. a member, leading to one type of strain (longitudinal) and one type of stress
(tensile). It is possible to load members in various other ways. Compression
He further found that the stress and strain, in a large part, have a simple linear load is a familiar example. Compression results when two bodies are pressed
relation for bars made of the same material: together. Columns that support elevated highways (Fig. 1.3), water tanks or roofs
Stress Strain, or, (1.3) are all compression members. A compression member shrinks in length because
of the load, resulting in compressive strains and stresses. The footing of a
The constant of proportionately, E, is termed as the elastic modulus, and depends machine is also under a compressive load. So also is the boom of the crane.
on the material of the bar. Strain being dimensionless, the dimensions (and
units) of E are the same as those of stress. Compression also results in the situation shown in Fig. 1.4. Here two plates are
riveted together. As a force is applied to the plates that tends to pull them apart,
Combining Eqs. 1.1 -1.3, we get: the rivet compresses the plates at the rivet holes as shown. The compressive
(1.4) stresses so produced in the plates are also termed as the bearing stresses.

The value of the elastic modulus E for most construction materials is quite high,
denoting that it takes fairly large forces to produce small elongations. Table B.1
in Appendix B gives the values of the elastic modulus for some common 1
A GPa is 109 Pa, or 109 N/m2
5
Another type of load is the shear load. Consider a block of rubber glued to a compressive stresses therein, while the lower fibres elongate introducing tensile
table on one side and a board on the other (Fig. 1.5). If we apply a load P as strain and tensile stresses therein.
shown, the block of rubber will undergo distortion and it will tend to slide off the
The magnitude of these compressive and tensile stresses is such that they
table. The distortion results in what are termed as shear strains, which, in turn,
integrate out to zero, which they must, since there is no applied force on the bar.
result in shear stresses in the material.
Shear stress and shear strain is also produced in the rivet of Fig. 1.4. As the force
is applied the rivet has a tendency to shear at the middle. Shear stresses develop
to counter this action and keep each half the rivet in equilibrium.

Bearing stresses in Shear stresses in


the plate the rivet
Fig 1.6 Twisting of a shaft. The shear stresses in the cross-section of the shaft
give rise to a resisting twisting moment.
Fig. 1.4 Riveted plates
But the resultant moment is non-zero as is required to resist the applied bending
moment and ensuring equilibrium.
Shear strain and shear stresses are also produced when a shaft is twisted as in Fig
Let us summarize what was learnt in this section:
1.6. If we consider the shaft as an assembly of thin slices stacked together, the
twisting action of the shaft tends to make the slices slip on one another. Internal  Forces that tend to reduce the size of a structural member produce
compressive strains which, in turn, produce compressive stresses.
Within limits, the magnitude of compressive stresses vary linearly with
P the compressive strain, the constant of proportionality being the same as
the modulus of elasticity, E introduced in the last section with tensile
stresses and strains.

Fig 1.5 Shear in a block of rubber

forces develop which resist these motions. These internal resisting forces are the
shear stresses and the resultant of these is a moment, termed as the twisting
moment. Fig. 1.7 Bending of a beam.
Another type of distortion occurs when a moment is applied to a bar which tends
to bend it. As shown in Fig. 1.7, the upper fibres of the beam tend to shorten
from their original length, introducing compressive strain and resulting in

6
 Forces that tend to distort the shape of a member, as in Fig. 1.5, produce uniformly on this material. Consider an elemental area 2 δA on a face of a
shear strains which in turn produce shear stresses. The more the shear structure exposed by making a cut at that location. Let δF represent the force
strain, the more is the shear stress. that the material that has been removed was applying at the elemental area δA.
 A twisting moment applied to a shaft produces shear strains in the shaft. This force arises from the deformation of the material under whatever external
These shear strains give rise to shear stresses across the cross-section of load is being applied to the structure.
the shaft, which result in a moment which balances the external twisting The stress at a point is defined as the intensity of the internal force of
moment. The more the external moment, more is the strain, more the deformation at this point:
stress, and more the resisting moment.
 A moment tending to bend a member as in Fig. 1.7, produces both Stress vector on this face at this point is t
compressive and tensile strains and stresses in the beam. These stresses
give rise to a resisting moment which balances the bending moment. The stress vector t depends upon the location as well as the direction of the
surface. If we had made the cut (at the same point) with a different inclination
(i.e., with the outward normal in some other direction), the stress vector would
1.4 The concept of stress at a point have been different as is shown in Fig. 1.9
In Section 1.2 we were dealing with a very simplistic case of a straight bar of
area A with a longitudinal load P and we defined stress simply as load per unit
area. This assumes a uniform distribution of stresses – a very severe
assumption. In practice, the deformations due to load and, consequently, the
strains and stresses will vary from point to point. It is, therefore, convenient to
define stress as the intensity of force at a point.

(a) A solid object (b) Section through X along bb (c) Section through X along cc
Fig. 1.9 Stress depends on the direction of the cut

We can take the component of this vector along the normal to the surface and
Area δA along the surface:
δF F The component of the stress vector normal to the surface is termed as the
normal stress and is denoted by the Greek letter ζ. The component of the stress
F
vector along the surface is termed as the shear stress and is denoted by the Greek
letter η. As noted before, the dimensions of stress are Force/Area, or FL-2, or ML-
1 -2
T , and its SI unit is N/m2 or Pascal, Pa. A Pascal is a very small unit and it is
(a) A gear (b) A cut-out portion of the tooth common to have stresses in kPa or MPa.

Fig. 1.8 Stress at a point


Fig. 1.10a shows a bar loaded uniformly in tension as shown. Because of this
load the bar elongates setting up axial strains and stresses along any cross
section. Fig. 1.10b shows the resulting free body. It stands to reason that if we
Consider a gear which is meshing with another gear which applies a force P on a take a section of this bar at any level, the resulting distribution of stresses must
tooth (Fig. 1.8). Let us consider the equilibrium of this tooth. Fig 1.8b shows the be the same. However, if the loading is not uniform but concentrated at a point in
part of the tooth as a separate free body. The force that balances the external
force F is the force which was binding this tooth to the rest of the gear. This is
an external force that arises because of the loading of the gear and its consequent 2
Area is a vector quantity, its direction denoted by the direction of the outward
straining. Let us imagine this straining and the internal forces are not distributed normal to the surface.
7
the bar as shown in Fig. 1.11a the story distributed loading) is uniform everywhere, and therefore, far away from the
changes. There is no reason to expect that point of concentrated loading in Fig. 1.11, we can take the stresses to be
the stress distribution at a section is uniformly distributed.
uniform. In fact, it is not. Moreover, it
We shall, in this elementary text, will routinely make the assumption of uniform
changes as we move from section to
stresses across sections, unless the context of the problem forbids it.
section. If we take a section at different
heights along planes b, c, or d, the
Example 1.1 Bearing stresses in foundations
resulting free bodies are shown in Fig.
1.11b, c and d. There total of the internal (a) (b) Consider a wooden column (Fig. 1.12) resting on a concrete footing.
forces or stresses at each of these sections Determine the maximum value of the load P if the maximum permissible
Fig. 1.10 The stress distribution
must be equal to P to balance the applied stress in concrete is 60 MPa and in wood is 25 MPa.
on a uniformly loaded bar
force. But the distribution of these
stresses is quite different at the three Solution:
sections. The distribution of stresses becomes more uniform as we move
upwards, away from the point of application of the force P. We assume that the stresses at any cross-section are
distributed uniformly. If P is the external load, the
In fact, there is a principle known as St. Venant principle, named after a stress in the wooden column is P (N)/ [π× (0.010 m)
nineteenth century French theoretician, which lays down this behaviour: 2
] = 3,183 P (N/m2 or Pa). Similarly, the stress in
The difference between the stresses caused by statically equivalent load systems the concrete footing is P (N)/ [(0.030 m) 2] = 1,111
is insignificant at distances greater than the largest dimension of the area over P (N/m2 or Pa). Equating these stresses to 25 MPa
which the loads are acting. and 60 MPa, respectively, we find the value of P
from the first as 7.85 kN, and from the second as 54
kN.
Clearly, the maximum permissible load will be the
lesser of the two, i.e., 7.85 N. This suggests that we
can reduce the footing size drastically, if bearing
d this load is the only design consideration.
Further we have considered only the compressive Fig. 1.12
c
stresses. Thin columns may buckle under
compressive load much earlier than they fail in compression. We will need to
b check that too. We shall deal with this in a later chapter.

P P P P
(a) (b) (c) (d)
Fig. 1.11 The stress distribution becomes uniform as
we move away from the point of loading

The loading in Figs. 1.10 and 1.11 are statically equivalent since they result in
the same net force and moments on the structure. The stresses in Fig. 1.10 (with
8
Example 1.2 A truss ∑ (a)

A truss structure is a load bearing structure assembled such that its ∑ (b)
members carry only axial loads, either tension or compression. It is made up Solving the two simultaneously, we get
of straight, two-force members3 joined together by frictionless pins. All the
loads are applied at only the joints. Further, there are no external moments TBA = 0.51P, and TBC = 0.78P
applied to such structure. The stresses in the bars are obtained by dividing the A

A truss is a gross approximation to an actual structure. There are no practical above load by the cross-sectional area (assuming
frictionless pins, and most members carry some bending loads as well. But the uniform stress distribution), which is π× (0.003 m) 2
advantage of considering only two-force members (in simplifying calculations) is = 2.83×10-5 m2. The stress along BC will be the 1m

limiting stress, since it is higher. Thus, B


so enormous that we make the truss approximation wherever feasible. C

20kN
Fig. 1.13 shows a pinned structure consisting of two bars AB and BC from
1m
which a weight P hangs as shown. If the diameter of each bar is 6 mm and This gives the maximum load P as 2.9 kN Fig. 1.14
the permissible stress in the bar is 80 MPa, find the value of the maximum
load P. Example 1.3 A truss with roller support

Solution: Consider a structure shown in Fig. 1.14. If the members of the structure are
mild steel with a radius of 20 mm, find the stresses in members AB, BC and
Consider the AC.
FBD of point B
where the load P Solution:
is being applied.
Since AB is a The support at point A is termed as a hinged support. This means that the support
cannot apply any moment at this point to the structure and the structure can
pinned member,
articulate at this point. The only possible external reactions (from the support) at
and hence a two-
Fig. 1.13 this point is a force with components both along and normal to the surface.
force member
there can only be There is no reaction moment at this point. The support at point B is termed as a
tension along it. Let us call it TBA. Similarly, there will be tension TBC along the roller support. A roller support, besides permitting rotation of the structure,
member BC. permits translation of structure along the supporting surface. This means that not
only is the reaction moment zero, but the force component along the support
The equilibrium of a body requires that the vector sum of forces and moments surface is also zero. The only reaction possible at such a support is a force
should vanish: component normal to the surface.
∑ ∑ (1.5)
We need to apply only the force equations here. Taking the component-wise
sum of the forces, we get

3
A member on which external forces act only at two distinct points (and there is
no external torque acting on it) is termed as a two-force member. The forces
acting on a two-force member are collinear and opposed.

9
To find the stresses in the members, we need to determine the forces on them by Fig. 1.15b shows the forces on the member AC. It is clearly that the net force on
the member is along the length4 of the member, tensile, and equal in magnitude
20 kN to √( ) ( ) The member BC is under a compression
RAy
A of 20 kN.
RAx 20 kN Further, since at point B
there is no vertical force,
C there cannot be any
20kN
RBx force5 in member AB.
C 20kN
The stress in the two
20kN members can be found
1m by dividing the forces
with the respective cross-
(a) FBD whole structure (b) Force along member AC sectional areas. Thus, (a) (b) (c)
Fig. 1.15 the stress in member AC Fig. 1.16 Shear stresses in a key
doing a statical analysis. The first step in solving this problem consists of is 28.3 (kN)/(π/4)(15×10-
3
determining the reactions at the supports A and B. m)2 = 160.1 MPa, tensile, and that in member BC it is − 20 (kN)/(π/4)(15×10- 3
m)2 = 113.2 MPa, compressive.
The reactions at support are found by considering the equilibrium of the FBD of
the whole structure as drawn in Fig. 1.15a. Here all the external forces have Example 1.4 Key
been shown: the applied load of 20 kN, the horizontal and vertical reactions RAx
and RAy at the pinned support A, and the horizontal reaction RBx at the roller Consider the transmission of power by belt and pulleys. Fig. 1.16 shows a pulley
support B. The structure should be in equilibrium under the action of these being driven clockwise by a belt. If the pulley is turning at 100 RPM and if the
forces. There are two conditions for equilibrium: vector sum of all the forces as power transmitted is 1 kW, what is the shear stress in the key? The shaft dia. is
well as all the moments must be zero. 20 mm, and the dimensions of the key are 4 mm × 4 mm and a length of 25 mm.
Writing the sum of horizontal and vertical forces as zero gives us: Solution:
∑ (a) A key is a common device used to couple a pulley with a shaft, so that as the
∑ N (b) pulley rotates the shaft rotates with it. It is a metal piece inserted so that a part of
it is inside the shaft (in a slot termed as a keyway), and a part is within the pulley.
Eq. (b) gives RAy = 20 kN. There is only one equation to determine the other two As the pulley turns, the key moves with it. The pulley applies a force on the key
unknowns. We need one more equation. That we get from the moment balance. towards the right as shown in Fig. 1.16b. The key whose lower part is enmeshed
We can take the moments of forces about any convenient point. Here we take it with the slot within the shaft makes applies a force on the shaft to make it turn
about the point A (because then the two force components at this point contribute with the pulley. As the key pushes the shaft, the shaft, in turn, pushes the key
no moment): back towards the left.
∑ ( ) ( ) ( ) (c)
4
This gives = 20 kN, and, then Eq. (a) gives 20 kN, that is, 20 kN in This should have been obvious without the calculations since members AC and
a direction opposite to that shown on the FBD in Fig. 1.15a, or 20 kN towards BC are two-force members. The forces acting on a two-force member are
left. collinear and opposed.
5
This is quite interesting. If there is no force in member AB, we really do not
need that member. Can the structure survive without the member AB? The
answer is: yes, if we are concerned only with the equilibrium, and no, if we
worry about the stability.
10
Now, if we look at Fig. 1.16b, we notice that the upper part of the key is being Further, the pull force will tend to shear the plates as shown in Fig. 1.16d. This
pushed to the right while the lower part of the key is being pushed to the left. shearing action acts on a total area of 2×(10 mm × 2 mm) = 4×10 -5 m2 for the
We may say that the key will have a tendency to shear in the middle. Fig. 1.16c force in each rivet, which is 1 kN. Thus, the shear stress in the plates will be 1
shows the FBDs of the lower and the upper parts of the key. The internal shear kN/4×10-5 m2 = 25 MPa.
stresses acting on each half balance the externally applied forces.
Further the pull force will tend to crush the area of the plate as shown in Fig
Since the power transmitted is I kW, and the RPM is 100, the torque is 1000 W/ 1.16e, causing compressive or bearing stresses in the area in immediate contact
(2π r d/rev×100 rev/min/ 60 s/min) = 95.5 N.m. The force acting on the key (at of the rivet. Surely these will not be distributed uniformly over the area. But if
a radius of 10 mm) which transmits this torque is obtained as 95.5 Nm/0.01 m = we make the assumption that they are, the resulting bearing stresses will be 1
9,550 N kN/(2 mm × 10 mm) = 50 MPa.
The total shear force
acting on the key (the 1.5 Stress on oblique planes
lower or the upper part)
is, thus, 9,550 N. The Consider again a bar of area A loaded axially as shown in Fig. 1.18a. Let us look
area on which it acts is 4 at the stresses on an oblique plane b inclined at an angle6 θ. It is clear that if we
mm × 25 mm and, draw the FBD of the lower portion of this bar as in (b), the total internal force
therefore, the shear stress acting on this section is P as shown. But what is the stress here?
is 9,550 N/(4×10-3 m × The area on which this force acts in not A but larger than A, equal to A/cosθ. If
25×10-3 m) = 95.5 MPa. we assume, as before, that the stress is distributed uniformly over the area (which
As an aside, note that the shall be quite true if the plane is not to close to the point of application of the
applied load is producing load) the stress vector here will be P divided by ( ) , or t = Pcosθ/A. We
compressive or bearing can resolve this stress vector in two components (Fig. 1.18c), one normal to the
stresses as well. The oblique area, and the other along it. The normal component is the tensile stress:
area on which the ( ) ( ) (1.6)
compressive force (9,550 Fig. 1.17 Lap joint
N) acts is 2 mm × 25 mm This is also termed as the normal stress. The tangential component is a shear
and, therefore, the bearing stress is 9,550 N/(2×10-3 m × 25×10-3 m) = 190.1
MPa.

Example 1.5 Riveted lap joint


Consider two steel plates 2 mm thick joined together as shown in Fig. 1.16(a)
and (b) by two rivets of 10 mm diameter. The centres of rivets are 10 mm from P
η ζ -η
the edge of the plates. Determine the stresses in the rivets if a force of 2 kN is θ ζ
applied to the plates trying to pull them apart. -η η
b
Solution:
As was discussed earlier, the pull force tends to shear the rivets as shown in Fig. P P
(a) (b) (c) (d) (e)
1.17c. Since there are two rivets, we can assume at each rivet sustains half the
total force, i.e., 1 kN. This is the shear force in each rivet acting on the cross- Fig. 1.18 Stresses on an oblique plane
section l re of π× (0.005 m)2 = 7.85×10-5 m2. Therefore, the shear stress in
rivet shanks is 1 kN/7.85×10-5 m2 = 12.7 kPa. 6
The ngle θ is lso the ngle between the norm ls to the new pl ne nd the
original plane.
11
stress: width of the strip as we go around the tube through one circumference, 2πR as
shown in Fig. 1.19b. This angle θ is, therefore,
( ) ( ) . (1.7)
Clearly, the value of the tensile and shear stresses at a location in the bar varies .
with the angle of inclination θ of the plane being considered. The maximum
tensile stress occurs when θ is zero, i.e., the cut is perpendicular to the axis of the The compressive stress in the cardboard tube along the axial direction is 100 N/
bar, and reduces continuously as θ increases. But the behaviour of shear stress is [2π(35×10-3 m)2×(2×10-3 m)] = 227 kPa.
quite interesting. If we note that cosθ sinθ is (1/2)sin2θ, we see that the shear We can now use the Eq. 1.7 to determine the shear stress along the seam, which
stress η first increases as θ increases, attains a maximum value (equal to P/2A, is a surface with the normal in the direction ̂ as shown in the figure. It can be
i.e., half the maximum value of the tensile stress) when θ is π/4, and then seen that the angle θ here is the same as the angle in Eq. 1.7 (the angle between
decreases back to zero as θ increases to π/4. Also note that if the sectioning the normals to the axial plane and the new plane on which stresses are to be
plane is inclined in the other sense (with negative values of θ), the shear stress determined). Therefore,
on it has a reversed sense too. This is also obtained from the expression for η
given above since θ now is negative. ( ) ( ) = 57.7 kPa

Two planes, one at angle θ, and the other at angle - θ are termed as This is the shear stress along the glued seam. The seam would rip if the glue
complementary planes. Shear stress on complementary planes have same cannot sustain this level of shear stress.
magnitude but opposite sense.
Please note that the equations obtained above are valid only for the special case 1.6 Notation for
of axial loading of a straight uniform bar. If the geometry or loading were stress: double-index
different, these would no longer be valid. notation
Example 1.6 A Cardboard tube As is clear from the
discussion above, the stress
A cardboard tube of diameter 70 mm is made from a strip of width 60 mm and
vector at a point depends on
thickness 2 mm wound spirally with the edges glued together as shown in Fig.
the orientation of the
1.19a. The cylinder is subjected to an axial load of 100 N, determine the shear
surface under consideration.
stress in the
We can, at the same point,
glued joint.
consider many differently
Solution: inclined surfaces. The
Fig. 1.20 Specification of stress components
orientation of a surface is
The angle of denoted by the direction of the outward normal to the surface. Therefore, we
the spiral can must specify the direction of the normal to the surface whenever we talk of stress
be obtained by at a point.
imagining that
as the We shall see later (in Chapter 6) that if we know the stress vectors on three
cardboard strip mutually perpendicular planes we can determine the stress vector on any other
is wound up, plane. We, therefore, need to specify stresses on three such planes. Figure 1.20
the strip should shows stresses on three mutually perpendicular planes, with normals in x, y, and
advance in the z directions. We have shown here, the components of stress vectors on each of
axial direction these three planes. The tensile (or the compressive) components are represented
a distance by the symbol ζ, and shear stress components by the symbol η. We use here a
equal to the double-index notation, with the first index of a stress component denoting the
plane on which it acts, and the second index denoting the coordinate direction of
12
a component itself. Thus, ηxy is a component of shear stress acting on a plane It may be verified that all the stress components shown in Fig. 1.20 are positive.
with the normal in the x coordinate direction, the component itself being in the y
coordinate direction. Similarly, ηzy is a shear stress component acting on a plane Example 1.7 Nomenclature and signs of stresses
with the normal in the z coordinate direction, the component being in the y Fig. 1.21 shows stress
coordinate direction, and ζzz is a tensile stress component in the z direction acting components on some faces 8 7
on a plane with normal in the z direction. It should be apparent that there are: labelled (a) to (d). Name
Fa
 three tensile components, ζxx, ζyy, and ζzz, one each on the three faces these planes on which the 9
Face c ce
3 2 d
(note that the indices are repeated in each of them), stresses have been shown. 6
 six shear stress components: two on each face, ηxy and ηxz on the x-face, Name the stresses and
1 10 11
ηyx and ηyz on the y-face, and ηzx and ηzy on the z-face. We shall soon see determine their signs
according to the sign
4
c ea y
that three of them are equal to the other three, so that there are only a 12
convention outlined 5 Face b F
three independent shear stresses at a point (see section 1.7). x
above.
Sign convention: It is convenient to use the following sign convention: z
Solution: Fig. 1.21
We first define a face to be positive or negative: a face with the outward normal
in the direction of the positive coordinate axis is termed as a positive face; else it Plane a is an x-plane, b a y-plane, c a z-plane and d is an x-plane (with the normal
is termed as a negative face. When a positively directed force acts on a positive in –ve x direction. The nomenclature and signs of the stresses are tabulated
face or a negatively directed force acts on a negative face, the stress is assigned a below8:
positive sign. And when a negatively directed force acts on a positive face or a
positively directed force acts on a negative face, the stress component is assigned
a negative sign. Stress Index Index for Symbol Sign Sign of force Sign
of force for stress of component of
A stress component is considered positive when a positively directed force acts plane component plane stress
on a positive face, or a negatively directed force acts on a negative face7. 1 x x ζxx + ve − ve − ve
In other words, if both the sign of the force and the face are the same, positive or 2 x z ηxz + ve − ve − ve
negative, the resulting stress is a positive and if the two signs differ, the stress is 3 x y ηxy + ve + ve + ve
negative. This is summarised in Table 1.1 below: 4 z x ηzx + ve − ve − ve
5 z z ζzz + ve − ve − ve
Table 1.1 Sign convention for stress 6 z y ηzy + ve + ve + ve
7 y z ηyz + ve − ve − ve
Direction of normal Direction of force Sign of stress 8 y y ηxz + ve + ve + ve
In positive coordinate In positive coordinate Positive(+) 9 y x ζzz + ve − ve − ve
direction (+) direction(+) 10 x x ζxx − ve + ve − ve
In negative coordinate In positive coordinate Positive(+) 11 x z ηxz − ve − ve − ve
direction (−) direction(+) 12 x y ηxy − ve − ve + ve
In positive coordinate In negative coordinate Neg tive(−)
direction(+) direction(−)
In negative coordinate In negative coordinate Neg tive(−) 1.7 Equivalence of shear stresses on complementary planes
direction(−) direction(−)

7 8
This sign convention follows from the consideration of action and reaction Note that the signs of stresses on opposite faces a and d are identical, as they
having the same sign. should be. In fact, the sign convention has been designed to ensure this.
13
Consider a small two-dimensional element of dimensions δx and δy and of unit  three shear stress components ηxy, ηyx and ηzx.
depth as shown in Fig.1.22. We have
drawn the stress components on the ζyy We shall show in Chapter 6 that we can, from these six components on three
four faces of the element. We assume orthogonal planes through a point, determine the stress vector on any plane
ηyx
a two-dimensional state of stress, i.e., ηxy through that point.
δy
we assume that there is no loading and, ζxx
hence, no stress component in the third • δx ζxx 1.8 Stresses in a thin circular pressure vessel
ηxy
direction. We shall now consider the y
equilibrium of the element under the ηyx Consider a thin9 cylindrical pressure
action of the forces due to these x ζyy vessel of length L, radius R, and wall
L
stresses: thickness t. Fig. 1.23 shows the
Fig. 1.22 A 2-D infinitesimal element
cylinder with end plate removed. A
cylindrical polar co-ordinate system
Face Area Force with r, θ, and z coordinates is ideal for
(identified by the assume unit this geometry. Let the internal excess θ R
pressure be p. Under the action of t r
direction of depth x-component x-component z
outward normal) these forces, stresses will be set up in
the cylindrical vessel. Following the
x δy•1 ζxx• δy ηxy• δy development of the previous sections,
−x δy•1 −ζxx• δy −ηxy• δy there will be six stress components Fig 1.23 A thin cylindrical vessel with
y δx•1 ζyy• δx ηyx• δx required to describe the state of stress end plate removed
−y δx•1 −ζyy• δx −ηyx• δx in the vessel walls: three tensile
components: ζrr, ζθθ, and ζzz, and three shear stress components: ζrθ, ζθz, and ζzr.
When we consider the force balance, we verify that there is no net force on the
element: the sum of x- and y- forces sum out to zero independently. We next take Let us first consider the tensile stress component ζθθ. As indicated by the first
the sum of the moments about the centre point of the element. Note that the subscript, θ, it acts on a plane with normal in the θ direction. This stress is
tensile forces produce no moment about that point. However, the shear forces exposed (or m de ‘extern l’) by taking a section as shown by a diametrical
do. The shear forces on opposite faces are equal in magnitude but opposite in cutting plane as shown. Figure 1.24a shows the FBD of the lower half of the
sign and, therefore, constitute two couples, one anticlockwise (positive) and the cylinder (ends removed). We can determine the stress component ζθθ by
other clockwise (negative). The moments of the two couples are found by considering the equilibrium of the vertical component of all forces.
multiplying the magnitudes of the forces and the perpendicular distance between The total vertical force due to the tensile stresses is ζθθ times the area on which
the two lines of action. The moment balance equation gives: these stresses act, which is 2×(L×t). Therefore, this force is 2ζθθLt. This force is
( ) ( ) , being balanced by the vertical component of the pressure forces acting on the
inside the half-shell. The integration of the vertical component of pressure forces
which on simplification gives = . acting on this half of cylindrical shell is not straightforward. But it can be
determined quite easily by resorting to a frequently used trick. The pressure force
This is an important result and establishes that the shear-stress components on acting on the shell is equal and opposite to the pressure forces acting on the gas at
adjacent (orthogonal) faces are equal in magnitude. We can, similarly, show that
the shell wall (the principle of action and reaction). Consider the ‘FBD’ of the
= and = . As much was stated in Section 1.6 without proof. We ‘g s’ cont ined in this h lf shell s shown in Fig. 1.24b. The vertic l net pressure
reiterate what was stated there: forces on the curved surface this FBD (which is the same and opposite to the
The state of stress at a point can be established with six components of stresses:
 three tensile components, ζxx, ζyy, and ζzz, one each on the three 9
‘Thin’ refers to the condition th t the w ll thickness t is much less than the
orthogonal faces, and radius R.
14
pressure forces of Fig. 1.24a) is balanced by, and hence equal to the pressure class of arguments termed as the symmetry arguments. Consider the two halves
force on the diametrical plane of this FBD. This is p(2RL). Therefore, the of the cylinder as shown in Fig. 1.26. We have shown the shear stress
integrated pressure force on the curved surface of the FBD of Fig. 1.24a is 2pRL. components ηθr on the upper half. Note that on the left-hand side, the plane has a
normal in the +ve θ direction (being counter-clockwise), and the stress is in the
2 ζθθLt = 2pRL, or,
+ve r direction. Therefore, the stress is ηθr and is +ve. On the right-hand side of
(1.8) the upper half, both the signs are negative and therefore the stress is +ve again.
The two stresses having opposite sense is correct, since the horizontal forces
This tensile stress in the circumferential direction is also termed as the hoop acting on this FBD should some out to zero (the sum of the horizontal component
stress. of uniform pressure forces can safely be assumed to be zero).
We next consider the tensile stress ζzz which is the stress on the plane with Now on the lower half, the Newton third law (the
normal in the axial principle of action and reaction) dictates that the
σθθ
direction. Such a stresses should have a sense opposite to that on the upper half. This is as shown.
face is exposed when But here we run into a problem. The stresses are both outwards on the upper
we cut the cylinder σθθ
half, and both inwards on the lower half. This violates symmetry. It is easy to see
with a r-θ plane p that it is not possible to distinguish between the upper and the lower halves of the
parallel to the ends cylinder. If somebody came and switched the two halves while a reader was
as shown in Fig. 1.25 away, there is no way by which the reader can tell which is which. This
which shows the symmetry requires that the state of stress on the two halves must be the same,
FBD of one part of either both inwards or both outwards. Thus, the requirements of third law and
the vessel so (a) (b) that of symmetry are in contradiction, and the both must hold! The resolution
exposed. The this contradiction is possible only if the stress ηθr vanishes. This is a sufficient
external forces in the Fig 1.24 FBD of one half of thin cylinder exposing ζθθ proof. We can construct similar symmetry arguments to show that the other two
z-direction acting on shear components ηrz and ηzθ too must vanish. Thus,
this FBD are due to the tensile stress ζzz acting on area which can be
ηθr = ηrz = ηzθ = 0 (1.11)
approximated by 2πRt, and the pressure p acting on the end plate area of πR2. The
z-force equilibrium, then, gives: ζzz×2πRt - p× πR2. This gives:
(1.9) 1.9 Summary
 The external forces acting on a structure result in strains in the structural
The third tensile stress is ζrr in the radial direction. Note that on the inner
members. The strains so produced result in stresses within the material
curved surface the pressure p acts radially and,
of the members. The stresses, for the most part, are proportional to the
therefore, is the stress ζrr at that point. But on strains produced.
the outer curved surface, the pressure is zero, so ζZZ
 The constant of proportionality is termed as the modulus of elasticity
the stress ζrr is zero. Therefore, ζrr varies from
which is a property of the material of which the structural member is
0 to p across the thickness of the cylinder. In p made.
any case, since r >> t, the value of ζrr (between
 In general, the stresses vary from point to point, but the use of the St.
0 and r) is much less than ζzz or ζθθ. Therefore,
Venant principle permits us in many simple situations to assume
in comparison to the other tensile stresses in the
uniform stress distribution across a section. This is a useful particularly
cylindrical shell, it is common to neglect ζrr,
at sections fairly distant from the points of application of loads.
(1.10)  Stress vector at a point describes the intensity of internal forces that
develop within a material in response to distortions that are produced
Let us next consider the shear stresses. We first
Fig. 1.25 FBD for determining ζzz under the application of external loads.
discuss ηθr. We shall use a very interesting

15
 The stress vector at a point depends on the orientation of the surface on
which the stress acts. Eqs. 1.6 and 1.7 describe the formula for
calculating the tensile and shear stresses on an oblique plane in the case
of longitudinal loading of a bar.
 The stress at a point can be described by stating the stress vectors on
three mutually perpendicular planes with a total of three tensile
components and six shear components.
 A double index notation is used to designate stress components, the first
index representing the direction of the outward normal to the surface,
and the second index the direction of the component itself.
 A stress component is considered positive when a positively directed
force acts on a positive face, or a negatively directed force acts on a
negative face.
 Equilibrium of forces require that the shear stress components occur in
pairs, so that the shear stresses on complementary planes are equal:
= , = and = . Thus, there are only three independent
shear-stress components.
 We showed by symmetry arguments that the shear stress components in
a thin walled cylindrical pressure vessels all vanish.

16
different slopes collapse into one line, as long as the material of the various bars
is the s me. This f ct w s st ted s Hooke’s l w (Eq. 1.3), , where the
constant of proportionately E is termed as the elastic modulus, and is the
property of the material of the bar.
The above discussion can be seen in a slightly different light. The deformation
produced in the bar depends upon the load, the geometry of the bar and its
material. So does elongation. But the geometry of the bar is absent from Eq.
1.3. Once you convert the load to stress and deformation to strain, the
geometry of the structure becomes irrelevant.
The above is a very powerful insight and forms the basis of the fundamental
strategy of analysis in mechanics of deformable materials. This strategy can be
stated in the following manner: To determine deformations in a structure under a
given loading, we first convert the loading to stresses using equilibrium
considerations, convert stresses to strains using the material properties, and the
use the geometry of the structure to determine deformations from the strains so
calculated. This is also stated as a formula: macro to micro, conversion at micro
2 Deformations, strains and level, and then micro to macro. Loading is macro, stress and strain are micro
level, and deformation is macro level.
material properties We may, at time, need to go in the reverse direction. We may be given the total
deformation from which we need to determine the loading: we calculate strains
from deformation, convert strains to stress using material properties, and the
2.1 Fundamental strategy of mechanics of deformable integrate stress to find the loading. The strategy here too is macro to micro to
mechanics micro to macro.
All structures resist loads by deforming. A structure deforms as a load is applied Example 2.1 Tug of war
to it. As it deforms, the stresses build up within the structure to resist the applied
load. More the load more is the deformation and more are the stresses. The Consider a tug of war in which 6 young men are pulling on a manila rope (cross-
deformation increases till the resulting stresses are sufficient to balance the sectional area: 6 cm2) with forces as shown. Find the net elongation of the rope.
applied load.
We had considered in Sec. 1.2 the
δ
ε = δ/L

deformation of a uniform bar under a


Area A
longitudinal load. If we take bars of
l
various cross-sectional areas and of
various lengths and plot the variations
of deformation with the loads, we
δ
obtain plots as shown in Fig. 2.1b.
P P ζ =P/A
The variations are largely linear with
slopes that are different for different (a) (b) (c)
bars. We had seen earlier that if we
convert the load to stress (stress, ζ = Fig. 2.1 Elongation of bars with loads
load, P/cross-sectional area, A) and
deform tion to str in (str in, ε = elong tion, δ /original length, L), the lines with
17
By simple balance of forces we obtain TAB as 250 N. Similarly, from the other
FBDs we get TBC = 500 N, TCD = 800 N, TDE = 550 N, and TEF = 300 N.
If we assume a uniform distribution of stresses, which will be quite true away
from the points where loads are applied (St. Venant principle), we can find the
stresses in each section by dividing its tension by the cross-sectional area of the
rope. Thus, the stress in section AB is ζ = 250 N/0.0006 m2 = 416.7 kPa. The
stresses in other sections can be determined in a similar manner and are given in
column 3 of Table 2.1 below.
Stresses to strain:
Stresses nd str ins re rel ted by Hooke’s l w: ζ =Eε, where E is the elastic
Fig 2.2 Tug of war modulus of the material. A search of literature reveals significant variations in
the value of E for manila ropes. A value of 100 MPa appears to be a good
approximation10. We get the strains by dividing the stress values by this value of
Solution: E as shown in column 4 of Table 2.1.
Each section of the rope has a different tension, resulting in different stresses, Table 2.1 Calculation of elongation of the rope of Example 2.1
strains and elongations. Let us take the four steps of our strategy in sequence.
Loading to stresses: (1) (2) (3) (4) (5) (6)
Section Tension, N Stress, kPa Strain Length, m Elongation, m
To find the stresses, we first need to determine the tension in each section of the AB 250 416.7 4.16×10- 3 1.5 6.24×10- 3
rope. There are five distinct sections carrying tensions. (The sections at either BC 500 833.3 8.35×10- 3 2.0 16.70×10- 3
end do not have any tension, and, therefore, do not need any consideration.) This CD 800 1333.3 13.33×10- 3 1.5 20.02×10- 3
is done by making appropriate FBDs which will make the required tension force DE 550 916.7 9.20×10- 3 1.5 13.78×10- 3
EF 300 500.0 5.00×10- 3 2.0 10.00×10- 3
Strain to deformation:
Once the strains are calculated, we can determine the elongation in each section
by multiplying the strains with the length of the respective section. This has been
shown in column 6 of the table. The total elongation is 66.74×10 - 3 m, or 6.67
cm.

Example 2.2 A hanging cable


Consider a cable of uniform section hanging as shown in Fig. 2.4. A steel cable
hangs from a roof under its own weight. It is important to see that the tension
along the length of the cable is not constant, but varies. This can be seen by
considering two sections, one at level b, and the other at level c. We have, in Fig.
2.4b, drawn the FBD of the cable up to level b. The tension T1 is clearly equal to
weight W1. The weight W2 in the FBD of Fig. 2.4c is no doubt greater than W1,
Fig. 2.3 FBDs for determining tensions in the rope and therefore, T2 is greater than T1. Thus, the tension, and hence the stress along
an external force. Fig. 2.3 shows a sequence of FBDs drawn for this purpose.
The first of these has externalised the tension TAB in the section AB of the rope. 10
The value of E for the rope accounts also for its unraveling as it stretches.
18
the cable increases from bottom to top. . / , neglecting terms of second
The diameter of the cable is dictated by
order in dx.
the maximum stress in the cable. There is
a considerable wastage of material, since The force balance will then give, on simplification:
the material near the bottom is not loaded
to its capacity. , where the negative sign indicates that diameter D decreases
In situations where the material costs are as x increases. Using the boundary condition that the diameter D = D0 at x = 0,
heavy, it often pays to reduce the cable we can solve this to get11
diameter as we go down from the top.
Fig. 2.5 shows a cable where it has been
done. Its diameter at x = 0 is Do, and at x How would we now calculate the total elongation of the cable?
= L is D1. The variation in diameter is
such that the stress at any given location (a) (b) (c) We first convert the stress at each section to the strain at that section. Since the
is constant equal to ζo. Find the stress everywhere is the same, equal to , the strain is also the same everywhere
Fig. 2.4 A hanging cable
differential equation governing the equal to , where E is the elastic modulus. And, therefore, the total
variation of diameter with x. Also set up an equation to determine the total elongation is easily determined as .
extension of the cable as a
function of the various Example 2.3 Deflection in an elementary truss
parameters involved. Consider again the truss with a roller support (Fig. 2.6) discussed previously as
Example 1.3, where the forces and stresses in the various members AC, BC and
Solution: AB were determined. It was shown that there is no force in AB, a tension of 28.3
What is important to realize in kN in member AC, and a compressive force of 20 kN in the member BC. The
the problem is the fact that the respective stresses were found to be 0, 160.1 MPa, and 113.2 MPa. Determine
tension at any location varies the displacement of point C where the load of 20 kN is applied. All members are
with x, the distance from the made of steel (E = 210 GPa).
roof. The weight supported by
the section at x reduces as x Solution:
decreases. We first determine The equilibrium part has already been solved. So also the stress determination.
the tension as a function of x. We now need to find the displacement of point C to C1.
Fig. 2.5 Cable of variable diameter
To determine the tension at x, For this purpose, we first find the elongation (or shortening) of members AC and
we take a slice of thickness dx of the cable at x, and draw its FBD. This FBD has BC. The stresses in the two members have already been calculated. The strains
three forces, the upward tension T at x, the downward tension T + dT at x + dx, are now calculated as stress/E. Thus,
and the weight dW of this slice of the cable. Clearly equilibrium requires that the
algebraic sum of the three be equal to zero. If the variable diameter of the cable εAC = 160.1 MPa/ 210 GPa = 0.76×10-3, and
is denoted by D as a function of x, the tension T is easily estimated in terms of εBC = − 113.2 / 210 G = − 0.54×10-3,
the stress ζo as ( ), the stress being given as constant throughout.
The weight dW is easily estimated as ( ) , where ρ is the density of
the cable material and g the acceleration due to gravity. To determine the tension
T + dT, we have to note that the area at the bottom of the FBD is different from 11
What about the second boundary condition: D = D1 at x = L? There is no other
that at the top. The tension here is thus,
constant to determine. Or is there not? The resolution of this question lies in the
fact that the constant stress ζ0 cannot be specified independently of D1 and L.
[ . . / / ]
How would you determine ζ0 in terms of the other parameters?
19
the negative sign a perpendicular to AC at E (to represent the tangent to the arc through E. The
indicating that the point where the two perpendiculars meet is the location of C1. We express the
A
strain is compressive. A new location by specifying the x- and y- displacement of the point C. The x-
displacement is easy to determine. It is nothing but EC, which is δBC = 0.54 mm.
We convert the strains
into elongations. The We note that the y-displacement is EC1 = EF + FC1 = CG + FC1. But, CG =
lengths of the two bars 1m CD/sin 45o = 1.07 mm/ 0.707 = 1.52 mm. we also see that FC1 = FG = 0.54 mm.
are 1.41 m and 1 m, Thus, the y-displacement of the point C is 1.25 mm.
B
respectively, and the C B E C The procedure outlined here is quite a standard procedure for determining the
member AC elongates D displacement in pinned structures like this truss.
by δAC = (0.76×10- 20kN
3 C1
)×1.41 m = 1.07×10-3
1m
m (or 1.07 mm), and 2.2 Statically indeterminate problems
the member BC by δBC (a) (b)
= − (0.54×10-3)×1.0 m Fig. 2.6 Determining displacement in a truss
In the examples solved above
= − 0.54×10-3 m (or we first obtained loads and
0.54 mm). stresses in the members
before moving to the next
To find the new location of point C, we use the following strategy: We change step of determining strains
the lengths to the rod to their new length and make them rotate in arcs about their and stresses. There are,
pivot points A and B, respectively. The point where the two arcs intersect is the however, problems where we
deflected location of point C. cannot determine the forces
Fig. 2.6b shows the geometric construction for determining the displaced location in members without bringing
C1 of point C. Bar AC has been shown elongated to AD (with CD being equal to in the considerations of
δAC =1.07 mm. We draw an arc with centre at A and AD as radius. The deformation.
displaced point C should be on this arc. Similarly, we displace point C to E by Consider as an example a
an amount δBC = − 0.54 mm. We dr w n arc with centre at B and BE as radius. beam supported on two ends
The displaced point C should be on this arc as well. The point of intersection of as shown in Fig. 2.8. Since
the two arcs then represents the new location C1 of C. the be m’s deflection is Fig. 2.8 A propped beam
Determining the intersection excessive, it is propped up in
point of two arcs is a fairly the middle by a column. Also shown is the FBD of the beam. There are now
lengthy process. Fortunately, three unknown reactions and only two equations (vertical force balance and
since the changes in lengths are moment balance) to determine them. The problem is, therefore, statically
very small fractions of the indeterminate.
lengths of the rods, and It is easy to see that the reaction from the central prop depends on the rigidity of
therefore the angles of swing the beam. If the beam is quite rigid, the beam may not even touch the central
are quite small, we can adopt an prop and there will then be no reaction there. But as the rigidity of the beam
approximate procedure wherein decreases, the load borne by the central prop increases. Therefore, it is necessary
we replace the arcs by the to bring in the considerations of deformation even to solve the problem of
tangents to the arc. We draw a equilibrium.
perpendicular to AD at D (to
represent the tangent to the arc The general strategy for solving the problems of the mechanics of deformable
through D. Similarly, we draw Fig. 2.7 bodies consists of three major steps:

20
Consideration of static equilibrium and determination of loads, force it applies on the board be R2. Fig 2.7c shows the resulting FBD of the
Consideration of relations between loads and deformations, (first converting board.
loads to stresses, then transforming stresses to strain using the properties
Equilibrium analysis
of the material, and then converting strains to deformations), and
Considerations of the conditions of geometric compatibility. There are two unknown forces R1 and R2 in this problem. There is also the
unknown distance x that is to be determined. The equations of equilibrium give:
In statically indeterminate problems we cannot take these steps in a linear
sequence, because there are not enough equations of equilibrium to solve for all ∑ : (a)
the unknown loads. We have to consider simultaneously all three steps, even if
we were interested in only one, say, in determining the forces in the system, as in ∑ ( ) (b)
the problem above. The moments have been taken about the location of the first support. There is no
We illustrate this strategy first with a simple example involving springs, and then other equation of equilibrium. We cannot solve for three unknowns from these
with a little more involved problems. two equations. This is why this problem is statically indeterminate. We
nonetheless move on.
Example 2.3 A spring board Load deflection equations
Fig. 2.9a shows a rigid board mounted on two similar springs, each of spring Using the spring constant k:
constant12 k, and length h. A man with weight P walks out on the board till he
reaches a point a distance x from the second spring when the tip of the board (c)
touches the foundation platform as shown in Fig. 2.9b. What is the value of x in (d)
terms of P, L, h and k? Neglect the weight of the platform.
This introduces two more equ tions, but two more unknowns, δ 1 nd δ2 as well.
Solution:
Geometric compatibility
It is clear from the
geometry of the From the geometry of the similar triangles in Fig. 2.9b, we can conclude that the
problem that as the length of the first spring after deformation will be twice that of the second spring
man walks out, the first after deformation.
spring elongates ( ) (e)
applying on the board a
downward force, while The problem is solvable now. There are five equations and five unknowns, R1,
the second spring R2, δ1, δ2, and x, the distance to be determined.
compresses applying an Writing R1 in term of R2 from Eq. (a), calculating δ1 and δ2 from Eqs. (c) and (d)
upward force on the in terms of R2 only, and then substituting for δ1 and δ2 in Eq. (e), we can solve
board. Let the first for R2 to obtain
spring elongate by an
amount δ1 and let its
tension be R1. Let the
second spring shorten And then from (a):
by δ2 and the upward Fig. 2.9 A spring board
Using these values of R1 and R2 in Eq. (c) we get the desired result,

12
Spring constant k is defined as the force required for producing a unit
deflection in a spring. Thus, if a force F produces an extension (or a
compression) of δ, the spring constant is given by k = P/δ. Its unit is N/m.
21
Example 2.4 A bolt and a sleeve L1 = L2 =5×10-2 m,
A brass sleeve of length 50 mm (ID: 14 E1 = 110 GPa (for brass), and E2 = 200 GPa for steel.
mm, OD: 18 mm) is held between a nut
Using these values, we get P = 225 N.
and the head of a steel bolt of dia 10 mm
(Fig. 2.10). The nut is brought flush with The resulting stresses will be 225 MPa in brass sleeve, and 282 MPa in the steel
the sleeve and then tightened one quarter- bolt. Please note that these stresses are about the maximum that the sleeve and
turn. If the pitch of the bolt is 0.7 mm, the bolt can sustain. Any more tightening would result in failure of either or
determine the tension in the bolt. both.

Solution: Example 2.5 A kinky spring


This is a statically indeterminate problem. Fig. 2.11 shows an elementary design of a spring which has one spring constant
As we tighten the nut, a compressive for the initial deflection (till the load is compressing only the inner aluminium
force builds up in the sleeve which tube), and then it increases when the
shortens in length. Simultaneously, a Fig. 2.10 A bolt, sleeve and nut load plate touches the outer brass
tension is built up in the bolt which cylinder so that the load deflection
elongates. Even though we are interested only in calculating a force, we cannot curve looks like that shown in the
do so without bringing in the considerations of stress, strain and elongation. figure. For the inner tube of 40 mm
dia and 2 mm wall thickness and the
Equilibrium analysis
outer tube of 50 mm dia and 2 mm
This is simple. If a compressive force P builds up in the sleeve, the tension in the wall thickness, find the two spring
bolt is P. constants.
Force deformation analysis Solution:
The force deformation analysis is done using the macro-micro-micro-macro Till the load is borne only by the Fig. 2.11 Kinky spring
strategy developed in the previous section. To find the contraction in the length inner aluminium cylinder
of the sleeve in terms of the compressive force P, we first find the stress ζ = P/A,
the convert it into the strain ε = ζ/E = P/AE. The total contraction then is δ1 = εL For a load P the stress in the cylinder is obtained by dividing the load by its
= PL1/A1E1. The elongation of steel bolt is, similarly, δ2 = PL2/A2E2. cross-sectional area, which is πDt = π(40×10- 3 m)(2×10- 3 m) = 2.5×10- 4 m2.
Therefore, the stress is P/2.5×10- 4 Pa (compressive).
Geometric compatibility
We convert this stress into strain by dividing it by the elastic modulus for
The nut moves through a total distance of ¼ of 0.7 mm, or 1.75×10 -4 mm. If the aluminium, which is about 70 GPa. Therefore, the strain is (P/2.5×10- 4
steel bolt did not elongate at all, this would be the magnitude of δ1. But the bolt Pa)/70×109 Pa = 5.71×10- 8 P (compressive).
elongates through δ2. This means that the contraction of the sleeve would
decrease by an amount equal to δ2: Thus, For a length of 300 mm, this strain results in a shortening of the cylinder by ζ×L
-4 = (5.71×10- 8 P)×(300×10- 3 m) = 1.71×10- 8P m.
δ1 = 1.75×10 m − δ2.
The spring constant13 is load per unit deflection, i.e., P/(1.71×10- 8P) = 58.3
Substituting the values of δ1 and δ2 in this condition, we get: MN/m
PL1/A1E1 = 1.75×10-4 m – PL2/A2E2
Here, A1 = π[(18×10-4 m)2 − 14×10-4 m)2]/4 = 1.00×10-6 m2,
A2 = π(10×10-4 m)2/4 = 0.78×10-6 m2, 13
One can find the spring constant quite directly by noting that the stress is P/A,
strain is stress/E, or P/AE. Deformation, then, is δ = PL/AE. The spring
22
The spring constant changes when the compression exceeds 0.1 mm, or when the brass as 110 GPa, which is about in the middle of the range of values given in
load exceeds (58.3×106 N/m)×(0.1×10- 3 m) = 5.83 kN handbooks for brass or bronze as between 100 and 125 GPa.
After the outer cylinder kicks in Putting in the values of the parameters in Eq. (c), we obtain:
After the outer brass cylinder also starts bearing the load, the nature of the 1.71×10- 8F1 = 8.80×10- 9F2 + or,
problem changing drastically. We no longer can use equilibrium alone to
determine the loads borne by the two cylinders independently. It is a statically F1 = 0.51F2 + (d)
indeterminate problem where considerations of equilibrium are interwoven with Eqs. (a) and (d) can be solved simultaneously to obtain14
the consideration of deflections.
and
Equilibrium of forces
. (e)
Let us assume that after the total deflection exceeds 0.1 mm, the load borne by
the inner cylinder is F1 and that by the outer cylinder is F2. Clearly, the Finding the spring constant for this part is a little tricky. Note that we should
equilibrium requires that: subtract 0.1 mm from the deflection δ and 5.83 kN from the load P to obtain the
slope of the second part of the load deflection curve of Fig. 2.11. We note that
P = F 1 + F2 (a) . The spring constant after the kink is given
There is no other equation of equilibrium that can help us determine the two by:
forces.
Force deformation considerations
The value of δ2 in terms of F2 is given obtained from Eq. (b) as δ2 = 8.80×10-
Let us move ahead and determine the stresses in the two cylinders, which are 9
F2. Eq. (e) gives F2 in terms of P. Thus, we obtain
F1/A1 and F2/A2, respectively, where A1 and A2 are the two areas.
, or
The corresponding strains will be F1/A1E1 and F2/A2E2, both compressive, where ( )
E1 and E2 are the two elastic moduli. The contractions will be:
,
( )
δ1 = F1L1/A1E1 and δ2 =F2L2/A2E2. (b)
This is more than three times the value when only the aluminium cylinder was
We have taken all steps according to the procedure outlined in Sec. 2.1 and
being deformed.
illustrated in the examples there: load (macro) to stress (micro) to strain (micro)
to deformation (macro).
Geometric compatibility 2.3 Lateral strain: Poisson ratio
We know that the contraction of outer cylinder begins only when the inner Consider a bar of length L subjected to an axial load P as shown in Fig. 2.12.
cylinder has compressed by a full 0.1 mm. Thus, the geometric compatibility Under the action of this load, the bar elongates by an amount δL so that an axial
requires that: strain is set up equal to . From our experience with elastic materials
like rubber bands we expect that as the bar elongates its cross-sectional area
(c) decreases, i.e., transverse strains εyy and εzz are also set up. Simeon Poisson, a
The values of the various parameters are: L1 = L2 = 0.3 m, A1 = 2.5×10- 4 m2, A2 = French mathematician in the early 19th century proposed that the strain in a
3.1×10- 4 m2, E1 = 70 GPa, and E2 = 110 GPa. We have used a value of E for

14
One can verify this result by using P = 5.83 kN, the value where the
constant, therefore, is k = P/δ = AE/L. Plugging in the values of L, A and E we brass cylinder just kick in and finding that and as it
obtain the same value as above. should be.
23
transverse direction is a fixed These are the tensile (or compressive, if the algebraic sign is negative) strains on
(negative) proportion of the strain in the material due to tensile stresses. We can show using symmetry arguments that
the axial direction. Thus, P shear stresses do not cause any tensile strain. Therefore, Eqs. 2.2-2.4 can be used
to convert stresses into strain. These relations can be taken as extensions of
Hooke law for uniaxial stress, and collectively are known as generalized Hooke
(2.1) L law.
where ν is termed as the Poisson ratio
Example 2.6 Compression of a block of rubber
and is a property of the material15.
z A block of rubber 50 mm×50 mm×30 mm is placed in a cavity and compressed
Next, consider the case where a
y with a force of 1 kN (Fig. 2.13). Determine the decrease in the thickness of the
second normal stress ζyy is also P
x block.
present. Because the stress–strain
relation is linear, the additional Fig. 2.12 Transverse strain under axial Solution:
strains produced due to ζyy will load
simply be superposed on the strains As the rubber block is confined within a
due to ζxx. Further, since the properties of the material are independent of the cavity, its transverse dimensions will not
direction (termed as the property of isotropy), the strains due to ζyy are: change and therefore there will be no strain
in the y- and z-directions. The (negative)
stress in the x-direction produces a (positive)
Similarly, the strains due to ζzz are: strain in the transverse directions. The walls
of the cavity prevent the rubber from
. expanding in that direction, and as a
consequence, compressive stresses in y- and
Since these three strains in the three directions due to three different stresses are z- directions result. This is, thus, a multi-
all additive, the total strains in the three directions will be: axial loading situation and the generalized
( ), (2.2) Hooke law (Eq. 2.2-2.4) applies. Clearly, ζyy Fig. 2.13 Compressing a rubber
and ζzz re equ l (= ζ, s y) bec use of block
( ), and (2.3) symmetry. (we cannot distinguish between
y- and z- directions. The stress ζxx is equ l to (−1 kN)/ (50 mm× 50 mm) = −400
( ). (2.4) kPa, the negative sign indicating that it is a compressive stress. The value of E
for rubber is about 500 kPa with the value of Poisson ratio ν of 0.5 (note that this
value indicates that the rubber is essentially incompressible!). The three
equations then give:
-
15
It is interesting to explore what change in volume results when a uniaxial load ( ), (a)
is applied. Consider a small cuboidal element of dimensions δx, δy, and δz in the
x-, y-, and z-directions. If it is subjected to a load that causes an x-direction strain ( ) (b)
equal to εxx, the resulting y- and z-str ins re − νεxx each. The new dimensions of
the elements now are, δx(1 + εxx), δy(1 − νεxx), and δz(1 − νεxx). Therefore, the ( ) (c)
deformed volume is δx•δy•δz• (1 + εxx)•(1 − νεxx)•(1 − νεxx) ≈ δx•δy•δz•(1 + εxx −
νεxx − νεxx) = δx•δy•δz•[1 + (1 − 2ν) εxx], neglecting terms of higher order in ε. The Eqs. (b) and (c) are identical (as expected, because of symmetry), and from these
fractional change in volume is, thus, (1 − 2ν) εxx]. If the material is we obtain ζ = −400 k . Using this v lue in Eq. ( ), we get
incompressible, the v lue of ν, the oisson r tion must be ½. εxx = 0, i.e., no strain at all.

24
What is going on? is obtained by dividing it by the cross-sectional area of the cylinder which
sustains this load, i.e., 2 . The two stresses are superposed, and the net stress
This strange result is the consequence of the value of Poisson ratio being taken as
in the axial direction is given by
0.5, which is the same thing as assuming the material to be incompressible (see
footnote 5). Clearly, no strain can be produced in an incompressible material (a)
confined in a cavity!
The negative sign is because the force F causes compressive stresses. The hoop
Let us redo this problem with a different material. Let us assume that the value
stress is given by Eq. 1.8 as:
of E is 500 kPa and that of ν is 0.4, the other conditions remaining the same. The
three equations now are: (b)
-
( ), (d) The force F makes no contribution to the hoop stress.

( ) (e) The strain in the axial direction is given by Eq. 2.4 as

( ) (f) ( )

As before, Eqs. (e) and (f) are identical, and from these we obtain ζ = −266.7 Where ν is the Poisson ratio. Using the above values and the fact that ζrr is of
kPa. Using this value in Eq. (d), we get εxx = -0.37, a very large strain. The order p and, therefore, negligible, we get
thickness of the block will reduce by 37%. ( )
Example 2.7 A confined pressure vessel The value of ν is always less than 0.5.
A long thin-walled cylindrical tank is snugly fitted between two rigid walls as
shown in Fig. 2.14. It is
then pressurised to an
2.4 Shear strain
excess pressure of p. If A material subjected to shear stresses deforms in
radius of the cylinder is R shape. Fig. 2.15 shows an element of a material
and its wall thickness t, that is under the action of a shear stress. We have
determine the compressive seen earlier that shear stresses occur in pairs: If on
force exerted by the wall on the x-faces of an element there is are stresses ηxy
the cylinder. giving rise to a counter-clockwise couple as
shown, there must occur on the complementary y- Fig. 2.15 Shear strain
Solution: faces stress components that result in a clockwise
The length of the couple. Please note that all the stresses shown here have the same +ve signs 16. If
pressurized unrestrained the direction of one of the stresses was changed, the direction of all the
cylinder increases because components will be changed.
Fig. 2.14 A cylindrical tank between two walls
of the combined action of Though the element is under equilibrium under the action of these four types of
the axial and the hoop stresses. This results in the cylinder trying to push the stresses and the consequent couples, the element deforms in shape. It can be
walls outward, which in turn apply a compressive force F on the cylinder. We shown by some very interesting symmetry arguments17 that these stresses cannot
can determine this force by noting that the net strain in the cylinder in the z-
direction should be zero.
16
In fact, we have deliberately defined our sign convention for stresses to ensure
There are two sources of axial stress ζzz in the cylinder: one is due to the internal this very situation.
excess pressure ( ⁄ from Eq. 1.9), and the other is the axial force 17
The reader is referred to Crandall, Dahl, Lardner, Mechanics of Solids,
applied by the walls on the cylinder. The compressive stress due to this force F McGraw-Hill, 2nd SI edition, Section 5.4 for these fascinating arguments.
25
result in any linear (tensile or compressive) strains: the linear dimensions of the Solution:
element will not change. It is only the shape that changes in this case. The
From the given coordinates, the length of the line
change in shape is manifested in the change in the included angle between two
lines which were parallel to the axes. Thus, angle at A reduces to α. We define AB is √( ) ( ) . The
the shear strain in the element at this point to be the change in angle, i.e., (π/2 – original length of this line aligned with x-axis was
α), for an infinitesimal element in the limit that the dimensions of the element 0.20 mm. This gives a contraction of 0.006 mm,
tend to zero. The symbol used for the shear strain in the x-y plane is γxy, the or a strain εxx of – 0.006 mm/0.2 mm = − 0.03 or −
Greek letter gamma. Thus, 3%.
Similarly, the length of line AD is
. / (2.5) Fig. 2.16 Calculating strains
√( ) ( ) . The original
This is dimensionless as the linear strains were. The angles are measured in length of this line aligned with y-axis was also 0.20 mm. This gives a contraction
radians of 0.004 mm, or a strain εyy of – 0.004 mm/0.2 mm = − 0.02 or − 2%.
As for the tensile stress and strain, shear stress too is linearly related to the shear To calculate the shear strain γxy we determine the angle DAB. That is obtained by
strain. subtracting from π/2 the angle the line AB makes with the x axis
, (2.6) ( ) and adding to it the angle the line AD
makes with the y axis ( ). Thus,
where G is termed as the shear modulus of the material. The shear modulus has
the dimensions of force/area or ML- 1T- 2. It has the same unit as E has, namely The shear strain, then, is
Pascal, Pa. Table B.1 in Appendix B gives some representative values of shear This is the shear strain in the material.
moduli and Poisson ratios of some common materials. Steel has a Poisson ratio
of about 0.27. Aluminium and brass have a little higher values of 0.32 and 0.35, We next calculate the shear stress. From Eq. 2.6,
respectively. Soft rubber has a value of about 0.5, signifying that its volume does
not change appreciably as it stretches. (- ) , or −150 , where
we have used 26 GPa for the value of G for steel.
The shear modulus of most materials is about a third of their elastic modulus18.
We next calculate the linear stresses from Eqs. 2.2 and 2.3:
Shear stress in a plane depends on the shear strain in that plane alone, and not on
any other component of strain, shear or tensile. Similarly, ( ):
and (2.7)
( ):
Eqs. 2.6 and 2.7, together with Eqs. 2.2 – 2.4 form the complete set of strain-
stress relations. Solving these two simultaneously, we get ζxx = −5.57 G , nd ζyy = −2.70 G .
The negative signs indicate that both these stresses are compressive.
Example 2.8 Calculating strains and stresses
Example 2.9 A vibration isolator using rubber in shear
Fig. 2.16 shows a small element of a steel structure after deformation. The
undeformed element was aligned with the axes and its dimensions were 0.2 mm Figure 2.17 shows the schematic of a vibration isolator. The machine whose
× 0.2 mm. The coordinates of its vertices A, B and D after deformation are given vibrations are to be isolated is mounted on the central steel block and applies a
as (in mm): A(0,0), B(0.194, 0.013), and D(−0.012, 0.196). C lcul te the v rious vertical load of 8,000 N as shown. It is suspended from two vertical walls
strains and stresses. through two rubber pads of height 12 cm and cross-sectional area of 10 cm ×10
cm. The two rubber pads act as shear springs. Determine the total deflection of
the central post under the load and the effective spring constant of the two pads
taken together.
18
It will be shown in Chapter 6 that E, G and ν are related: ( )
26
Solution: 8,000 N There is no change in shear strain because of thermal expansion, i.e., a shear
strain arises only due to the respective shear stress component.
Fig. 2.18 shows a pad in deflected
configuration. It shares the total load of 8,000 If the structure in not permitted free expansion due to geometrical constraints,
N with the other pad, and therefore, carries stresses may be setup in the structure. Such stresses are frequently termed as
4,000 N as a shear load. This load acts on an thermal stresses.
area which is 12 cm×10 cm, or, 0.012 m2. The
In examples below we illustrate how the thermal and elastic strains can be
she r stress is −4,000 N/0.012 m2, or,
combined.
. With G for rubber as 1.0 GPa, the
shear train is = −333.3 k /1.0 Wall Wall Example 2.10 Thermal stresses
G = −0.33. The tot l line r deflection, Rubber blocks

then, is −0.33×10 cm = −3.3 cm. Fig. 2.19 shows a 20 cm long 3 cm dia steel
Fig. 2.17 Schematic of a rod held between two rigid supports. The
vibration isolator
Since a load of 8,000 N produces a vertical temperature of steel rod is raised by 100 0C.
deflection of 3.3 cm, the spring constant of What is the force in the bar?
the two pads taken together is 8,000 N/0.033
m = 242.4 kN/m. Solution:
The increase in temperature will tend to
2.5 Thermal Strains increase the length of the rod. But the rigid Fig. 2.19
supports will prevent it from elongating.
As the temperature of a body changes so This results in an axial force. If F is the compressive axial force acting on the rod
does its dimensions. The change of length because of the walls, the stress ζxx = F/A = F/7.06×10−4 m2. Since there is
with temperature is generally quite linear nothing that is applying a force in the other two directions, ζyy and ζzz are both
with changes in temperature of several Fig. 2.18 absent. Also, since there is no net change in the length of the bar, the axial strain
hundred degrees Celsius. If we heat a rod of εxx is 0. Using these values in Eq. 2.9, we get:
length L through a temperature change of ∆T, its length changes as ,
where α is termed as the coefficient of thermal expansion. This can be seen as , or .
producing a strain εT given by
Here α for steel is about 11×10- 6/ 0C, E = 210 GPa, and ∆T = 100 0C. The stress is
(2.8) ( - ) ( ) ( ), or 231 MPa,
−4 2
The unit of α is per . Representative values of α for some materials are given compressive. By multiplying this stress with the area of the bar (7.06×10 m ), we get
in Table B.1 in Appendix B. It should be noted that temperature changes do not the axial force F as 163 kN, a fairly large force.
produce any shear strains.
Example 2.11 Residual stresses in electroplating
These thermal strains are superposed on the elastic strains produced by loading of
members. The total strain produced in the presence of elastic strain is obtained The corrosion resistance of steel is
simply by adding the two strains. Thus, using Eqs. 2.2-2.4, and Eq. 2.8, we get: improved by electroplating it with a
thin layer of copper, usually only a
( ) (2.9) few hundred microns thick. For bright
electroplating the bath temperature
( ) , and (2.10) should be around 50 0C. After
electroplating when the plated steel
( ) (2.11) returns to the room temperature, say Fig. 2.20 Copper plating on steel
0
20 C, the higher coefficient of thermal
expansion of copper compared to that of steel results in copper tending to
27
contract more than that what the steel does. The bonding between steel and sleeve should be heated for fitting, and (b) the resulting interfacial pressure on
copper prevents this, and as a result steel does not let the copper layer contract cooling.
more than it does itself. Steel layer ends up applying a tension to the copper
layer, and the copper layer pulls the steel inwards, the total tension force in Solution:
copper being equal to the compressive force in steel. However, since the steel It is convenient to work in the cylindrical polar coordinates, r, θ, and z, as shown.
layer is an order of magnitude thicker than the copper layer, the stress in steel The relevant equations for strains and stresses are obtained by simply substituting
will be negligible compared to that in copper. r, θ, and z for x, y, and z in the equations developed so far20.
Thus, there is a residual tensile stress19 in the copper layer. Estimate this residual To determine the temperature beyond which the sleeve should be heated for
tensile stress. fitting, we use Eq. 2.8 with εT = 0.1 mm/20 mm = 0.005, and αsteel = 11×10- 6/ 0C,
to get
Solution:
There is no stress at 50 0C. When the temperature decreases to 25 0C (∆T = −25 ( - ) 455 0C
0
C) the net strain in the copper layer will be exactly the same as the net strain in The sleeve has to be heated through a range of more than 455 0C to slip it on the
the steel plate. The steel plate will have only the thermal strain, since we can shaft without forcing it.
assume that the thickness of the steel plate is so large (compared to that of copper
film) that the same force in the two layers (equilibrium requirement) will result in Now on cooling, the ID of the sleeve must be equal to the dia of the shaft. Let us
negligible stress in steel. Thus, look first at the stress and strain in the shaft. If the interfacial pressure is p, the
radial stresses in the shaft are of order p. There is no other stress in the shaft. On
. / ( ) the other hand, the sleeve will be subjected to a radial stress of the order of p and
a hoop stress (radial stress ζθθ (refer to Section 1.8). The hoop stress is given by
Using the property values: 0
11×10- 6/ C;
0
16×10- 6/ C; and Eq. 1.8 as pR/t. Since R >> t, the hoop stress, and consequently, hoop
, we get: strain are much larger than the compressive strain of the shaft. We can, thus,
neglect the compression of the shaft diameter in comparison to the increase in the
( ) ( ) ID of the sleeve. Therefore, we can take the final dia of the shaft to be 20 mm
itself, and the final ID of the sleeve too as 20 mm. As the dia of the sleeve
=( ) (11 – 16) ×10- 6/ 0C×(− 250C) increases from 19.9 mm to 20 mm, its circumference increases from π×19.9 mm
= 15 MPa (tensile), not an insignificant stress to π×20.0 mm, giving a hoop strain of 0.005. Since the axial stress is absent,
and the radial stress is of order p, much less than , we get 0.005 = /E.
Example 2.12 Shrink fit Using the value of E as 210 GPa, and expressing as pR/t, we get p, the
interfacial pressure as 157.5 MPa
A steel shaft (20 mm dia) is to be fitted with a
steel sleeve (20 mm long, OD 26 mm). In
order to tightly fit the sleeve, its internal 2.6 Tensile test
diameter is made as 19.9 mm. It is fitted on to
the shaft by heating it till its internal diameter Tensile test is one of the primary tests that are used to determine experimentally
is larger than 20 mm, slipping on, and then the properties of matter. A standardized specimen (Fig. 2.22) which uses a strip
Fig. 2.21 A shaft with shrink- or a rod of uniform cross-section between two enlarged ends is gripped and a
letting it cool. As the sleeve shrinks, it presses fitted sleeve
on to the shaft creating an interfacial pressure
which results in a good bond. Determine (a) the temperature beyond which the 20
This is possible since none of the equations we have developed so far has
partial derivatives of physical quantities with respect to x, y, and z. Since θ does
19
This residual tension can be very detrimental when the structure is subjected to not have the dimension of length, we cannot simply replace derivatives with
tensile (or bending) load. The copper layer would crack and the corrosion respect to y by those with respect to θ, and the equations that have partial
resistance will be lost. derivatives change in a more complex manner.
28
tension force is applied. An extensometer termed as the elastic limit. It is not
(Fig. 2.23) is used to measure how the length possible to determine either the
between two specified points on the uniform proportional limit or the elastic limit
portion changes as the tension is increased. with any degree of accuracy.
The tension is converted into stress ζ by
The strains in the elastic region are
dividing it with Ao, the cross-sectional area21,
very small, quite insensible without
and the elongation is converted to linear
Fig. 2.22 Tensile test specimens instruments to amplify their
strain ε by dividing the elongation by Lo, the
magnitude. That is why extensometers
gauge length (the undeformed distance
are used which amplify the strain,
between the two points on the specimen). On plotting the two we get a graph
either mechanically or optically.
like that shown in Fig. 2.24 for ductile materials like steel. The initial part of the
When the stress in steel is of the order
curve is a straight line and
of 400 MPa, the relative displacement Fig. 2.24 Typical stress-strain curve for
represents the linear elastic
of two points originally 100 mm apart
range22 of the material. If a a ductile material
would be of order of 0.2 mm. Strains
material is loaded to point A
in most structural materials are of the order of a few tenths of a percent at most.
within this range as shown,
and then unloaded, the As the load is increased further the material stops showing the elastic behaviour,
material will return to its that is to say that if the material is loaded beyond a certain point and then
unloaded state (strain will unloaded, it will not return to its original unloaded state. We say that the
return to zero). We say that material now has a permanent set. In Fig. 2.25, the material is loaded up to point
if the material is loaded and Y. When the load is now decreased, the material follows the straight line path
unloaded within the elastic downwards as shown. The straight line has the same slope E as the original
range, the material exhibits Fig. 2.23 Schematic of an extensometer used in a linear elastic line, but now it meets the strain axis at a point left of origin,
no permanent set. The slope tensile test signifying that there is now a strain even when the loading and stress have been
of the stress-strain curve in reduced to zero. The material is said to have yielded and undergone plastic
the linear elastic range is E, the elastic modulus of the material. deformation.
The stress level up to which the linear elastic range extends is termed as the It is quite difficult to decide
proportional limit. experimentally the point at which the
plastic deformation begins. It is for
As the stress is increased, non-linearity comes into play, but the material still
this reason that we, quite arbitrarily,
shows elastic behaviour, i.e., on unloading the material exhibits no permanent
designate the point Y, where the
set. The stress level up to which the elastic range (linear or non-linear) extends is
permanent set (the residual strain when
unloaded after loading to that point) is
21
Note that the stress here is only a nominal stress since we are not accounting 0.2%, (or a strain of 0.002) as the yield
for the changed cross-sectional area as the specimen elongates. This is not point of the material. The
significantly different from the true stress till the strain becomes quite large. In corresponding stress ζY is termed as
fact, the decrease in the stress at very large strains as shown in Fig 2.23 arises the yield strength of the material.
only because we are not using the true stresses. The load does not decrease as Consider a material that is loaded
strain increases. beyond the yield point Y to a stress Fig. 2.25 Yielding and permanent set
22
The linearity and elasticity are two different phenomena. The elasticity refers level Y1 (> Y). If the material is
to the fact that there is no permanent set, i.e., the material on unloading returns to unloaded now, it will follow a straight line down from Y1 with a slope of E to
the original unloaded state, along the same curve it followed while loading. This point B, and there will be a permanent set as shown in Fig. 2.26. If, after this
may or may not be linear. In fact, a rubber band is largely non-linear elastic.
29
initial yielding and unloading, the  Beyond ultimate stress, a neck develops in the specimen; the area
material is loaded yet again, it will decreases locally while the nominal stress decreases.
follow the straight line up from  Ultimately, the fracture takes place at the neck.
point B (with the slope E). This We have discussed so far the stress-strain curve for a ductile material. The
material is now not going to yield corresponding curve for a brittle material is significantly different from that for a
again till it reaches the stress level ductile material. Glass, cast iron, concrete, carbon fibre, and Perspex are some of
of Y1, higher than Y. Thus by the brittle materials. Many material ductile at room temperature may be brittle at
loading the material to a level low temperatures.
higher than the yield strength Y,
we have been able to raise the Most of the differences between the behaviour of ductile and brittle material
effective yield strength of the arises from the fact that there is little plastic deformation in a brittle material. Fig.
material (in subsequent loadings) 2.28 shows the stress-strain curves for glass and cast iron.
to Y1. This phenomenon is known Fig. 2.26 Strain hardening
The first thing to
as strain hardening and is note is that both
frequently employed to increase the wear resistance on materials. Shot peening these materials
and sand blasting are examples of processes employed to strain harden structures. undergo fracture
Cold rolling of structural steel also produces usable work hardening. at very little
Loading of the bar beyond the yield strength produces large deflections with strain, much less
small increases in the stress. The maximum stress that the material is able to than that for a
attain is termed as the ultimate stress. ductile material.
Glass, for
When a material is strained beyond the ultimate stress, the stress appears to be example, shows
decreasing as strains increases. As noted no plastic
earlier this is a false impression, since what we deformation at
are plotting here is the nominal stress obtained all.
by dividing the load by the initial area Ao. The
area of a ductile specimen decreases Another feature Fig. 2.28 Stress-strain curves for some brittle materials
Fig. 2.27 Necking of ductile
significantly when it undergoes plastic materials
to note is that the
deformation. This is, usually, a local strength to fracture is much larger in compression than in tension. This is a usual
phenomenon. The area of cross-section of the specimen suddenly starts characteristic of brittle materials.
decreasing at some point where there are imbedded dislocations in the material.
This is termed as necking (Fig. 2.27). As the necking progresses, the true stress at 2.7 Idealized stress-strain curves
the section increases and the specimen fractures.
The stress-strain curves for the various materials do not admit any simple
We summarize here the sequence of major events in the tensile test of a ductile
mathematical description. However, we do need such descriptions for analysis of
specimen:
structures. It is for this convenience that we model the materials into a few
 Linear elastic deformation till the proportional limit: . classes and use simple stress-strain relations to model the actual behaviour.
 Non-linear elastic deformation till elastic limit. Most problems of high school mechanics are solved by assuming the structural
 Yield point, where the permanent set is 0.2%. members to be rigid. This means that there is zero strain for any level of loading.
 If we unload the material beyond yield point, and then reload it, the Fig. 2.29a depicts the stress-strain curve for a rigid material.
yield strength increases. This is termed as strain hardening.
 As the material undergoes plastic deformation, the stress level increase
up to ultimate stress.
30
The stress-strain curve for a perfectly elastic material is as shown in Fig. 2.29b, a The strain hardening model is shown in Fig. 2.29e. Note that the elastic-perfectly
sloping straight line whose slope is the value of the elastic modulus E. We have plastic graph of Fig. 2.29c cannot model the strain hardening phenomena, since
used this idealization of materials in this book so far. This is a very useful model the slope of the plastic part is zero. Reloading the material after unloading
following some plastic deformation will not increase the yield strength. But the
non-zero slope of the plastic part of the curve of Fig 2.29e ensures that the yield
strength increases on re-loading after the material has been de-stressed after it has
undergone some plastic deformation.

Example 2.13 Fitting an idealized curve


The solid line in Fig. 2.30 shows the stress-strain curve of cold-rolled 1020 mild
steel. Find a simple mathematical relation for load vs. elongation of a 20 cm
long 20 mm × 20 mm cross-section bar.

Solution:
A look at the actual stress-strain curve suggests that though the material exhibits
a bit of strain hardening, adopting an elastic-perfectly plastic idealization will not
be bad at all. We first determine the yield strength of this material by drawing a
line parallel to the elastic portion of the curve from a permanent set value of
Fig. 2.29 Idealized stress strain curves 0.2% or from strain ε = 0.002. The light broken line so drawn meets the curve at
and is used extensively in most structural analysis, because most structures are a stress of about 580 MPa, which may be taken as the value of yield stress ζY for
designed to work within the elastic range. this material. Therefore, the idealized curve looks like the broken heavy line
shown in the figure. In other words, we may model 1020 CR mild steel as elastic
Another commonly used idealized model is the elastic-perfectly plastic one up to a stress level of 580 MPa with a value of E = 580 MPa/ 0.003 = 193.3 GPa
shown in Fig. 2.29c. in this, we assume that once the material yields, the stress (0.003 being the strain at the yield point). Thereafter, the steel may be taken to
remains constants at the yield strength level. We shall have occasion to use this deform perfectly plastically.
in a few problems.
A stress of 580 MPa
Fig. 2.29d shows the corresponds to a load of
behaviour of a 580 MPa×(20×10-3 m)2,
perfectly plastic or 232 kN. In this
material. In such a region, the elongation
material there is no will be δ = PL/AE =
strain till the stress P×0.2 m/(20×10-3 m)2 ×
reaches a critical (193.3 GPa) = 2.59×10-
level, after which 9
P m, where P is the load
there is no increase in Newton. We draw a
in stress as the line through the origin
material deforms. up to the yield point with
This idealization P = 232 kN. After this,
may be used when the deflection increases
Fig. 2.30 Stress-strain curve for cold rolled 1020 mild steel Fig. 2.31 Load-deflection curve
there are very large without any increases in
plastic deflections, the load. Fig. 2.31 graphs this load vs. elongation behaviour.
so that the elastic-strain region of the stress-strain curve can be neglected.

31
2.8 Pre-stressing tendons, while there is equivalent compression in the concrete. This residual
compression in concrete helps increase the tensile load capacity of the concrete.
Many brittle materials have much higher yield strengths in compression than in
tension. For example, concrete has a tensile strength of only 10-15 % of its Example 2.14 Pre-stressed concrete
compressive strength. It is
common to use Let us consider a simple pre-stressed concrete structure to serve as an illustration
reinforcement in concrete to of the calculations involved. Let a concrete beam of cross-sectional area 5 cm×5
give it increased strength in cm and length 2 m be cast with a 10 mm dia mild steel rod under a tension of 20
tension. This is used quite kN. The external tension in steel released after the concrete is set. What is the
effectively in the design of residual compressive stress in the concrete?
beams. We had seen in
Section 1.3 that in a beam Solution:
2.32 Reinforcement in a concrete beam
subjected to bending under A tension of 200 kN in the steel bar results in a stress of 20 kN/[¼π(0.010 m)2] =
vertical loads, its upper layers are in compression while the lower layers are in 255 MPa. (This is quite close to the yield strength of commercial structural steel.)
tension. It is common to increase the tension strength of concrete by reinforcing The strain under this stress is ε = 255 MPa/ 210 GPa = 1.21×10- 3, and the total
it with tendons of high tensile-strength material, such as steel rods (Fig. 2.32). elongation of the bar will be (1.21×10- 3)×2 m = 2.42 mm.
But many a times this is not sufficient. One of the very interesting ways to After the tension is released, the steel rod shortens, but it does not reach its
overcome this problem is pre-stressing. If we build up within the concrete some unstrained length, and there is a residual tension in the steel tendon and an equal-
residual compressive stresses when it is being cast, subsequent tension loading of magnitude residual compression
the structure will first relieve the compressive stresses before generating tensile in the concrete. This force
stresses. In this way, the structure will be able to support enhanced tensile loads cannot be determined by statics
before the tensile stresses reach the critical level. alone. We have to make
recourse to deformations and
apply a geometric compatibility
condition.
If δs represents the final
elongation of the steel bar (from
its unstrained condition, and δc
represents the contraction of the (a) Tendon as cast
concrete, it is clear from the (b) The no-stress length of the tendon
(c) The final configuration on release of tension
construction of Fig. 2.34 that δs
+ δc = 2.42 mm. This is the
Fig. 2.34
geometric compatibility
condition.

Fig. 2.33 Pre-stressing Let F represent the force, tension in steel and compression in concrete. Then, δs
= FLs/AsEs = F(2 m)/[π/4(0.010 m)2]×210 GPa, and δc = FLc/AcEc = F(2 m)/(0.05
Fig. 2.33 explains how the pre-stressing works. Tendons, most often steel wires, m×0.05 m)×20 GPa.
are put under external tension and the concrete cast around them. After the Thus, from the geometric compatibility condition we get,
concrete is set, the external tension is released. The tendons now try to shrink to
their no-stress lengths, but the concrete around them prevents them from ( ) ( ) ,
shrinking to their original length. In the process, there is a residual tension in the

32
Which gives F = 15.03 kN. This is the residual force which produces a forces, F1, F2, F3, …, Fn. Let the total strain energy be U, which is a function of
compressive residual stress of 15.03 kN/(0.05 m×0.05 m) = 6.01 MPa. This F1, F2, F3, …, Fn: ( ).
leads to considerable strengthening of the structure in tension.
Let us first load the body with all the forces F1,
F2, F3, …, Fn, and, then, increase one of the load,
2.9 Strain energy in an axially say Fi, by a small amount, say δFi. The additional
strain work done can be written as:
loaded members
When a force F is slowly applied to a
deformable body (that is supported in such a (a)
way that no rigid-body motion is possible)
work is done as the body deforms. The work Let us next reverse the order of loading. We first
Fig. 2.35 The area under the apply the small load δFi on the body before any
done U in deformation is force-displacement curve is the Fig. 2.37 An elastic body
other load is applied. After this load is applied,
work done in deformation
∫ (2.12) all other loads F1, F2, F3, …, Fn are applied on the body. Under the combined
action of all these loads, the point of application of load Fi moves through an in-
This is represented by the area under the F-δ curve of Fig. 2.35. For purely
line distance of δi. The work done by this small force δFi is then23
elastic deformations, the force-deformation curve is linear and therefore the work
done in deformation is given by . This work is stored within the body dU = δFiδi (b)
as mechanical energy and is termed as the strain energy. It is a conservative Since the order of loading (in the linear elastic case) should be immaterial, we
energy and is released when the load is removed and can equate the two values of work given by Eqs. (a) and (b), to get:
the body returns to its original configuration.
(2.14)
Let a number of forces F1, F2, F3, … ct on the body
(or the structure) and the resulting displacements at This is known as Castigliano theorem24. It states that if the total strain energy of
their points of application be δ1, δ2, δ3, …. The work a structural system is expressed in terms of the external loads applied to it, the
done in deformation is independent of the order in deflection at any one loading point (in-line with the load) is obtained by taking
which the forces are applied. We apply the forces the partial derivative of the strain energy with respect to the load at that point.
simultaneously such that the total work of
deformation is given by the sum of the work done by We illustrate the use of this theorem, first to two simple cases in Examples 2.15
individual forces: and 2.16, and then apply it to a more involved problem of a truss. Example 2.17
introduces a process that can be generalized and can also be used for solving
∑ (2.13) Fig. 2.36 A bar under statically indeterminate cases as illustrated in Example 2.18.
an axial load P
Let us calculate the strain energy of a bar subjected Example 2.15 An elementary truss
to an axial load as shown in Fig. 2.36. The deformation δ (elongation, in this
Consider again the truss with a roller support (Fig. 2.38) discussed previously as
case) is given by FL/AE, and the strain energy U is then . Example 2.3. Determine using the energy method the displacement of point C

2.10 Calculating deflections by energy methods: Castigliano 23


The factor of ½ is missing in this expression because the load δFi remains
theorem constant while the displacement δi takes place.
24
There are some simple extensions to Castigliano theorem. One of the simple
Consider an elastic body which is fully supported in the sense that its rigid body
extensions is its application to non-linear systems where the strain energy
translation and rotation are not permitted. Let it deform under the action of n
∫ is replaced with ∫ , which is termed as the complementary
energy.
33
where the load of 20 kN is applied. All members are made of steel (E = 210 same as was obtained in Example 2.3.
GPa) and have a circular cross-section of 20 mm dia.
To determine the horizontal displacement at C, we take the partial derivative of U
Solution: with respect to Q:
( )
, which on substitutions of values gives ,
Since the energy method involves taking the derivative with respect to the
force applied at the point where the deflection is to be calculated, we replace the negative sign indicating that it is in a direction opposed to that shown for Q.
the load of 200 kN with a variable load P. Also, since the use of Castigliano The amount of work involved here with the energy method is far less than that in
theorem gives displacement only in line of the force with respect to which Example 2.3, and the results here are more accurate.
the derivative of the energy is taken, we cannot find the horizontal
displacement of point C since there is no horizontal force present there. Example 2.16 Tug of war revisited
However, we can overcome this deficiency by introducing a dummy force Q
Consider once again the tug of war discussed as Example 2.1. We determine here
in the horizontal direction at C, and then taking the derivative with respect
the total elongation of the rope using the energy method. The cross-section of the
to Q before equating Q to zero.
rope was given as 0.0006 m2, and the value of the elastic modulus for the manila
A simple equilibrium analysis of the joint at C using the FBD given in Fig. 2.39b, rope was taken as 100 MPa.
gives:
A
Solution:
from the vertical force
balance, and
from the 1m
horizontal force balance.
B
C
These give , and
20kN
. The force in the element AB
1m
is zero as before. The total strain energy,
then, is: Fig. 2. 38 An elementary truss
( )

( )
Fig 2.40 Tug of war

To find the total elongation by energy method, we need to anchor the rope at one
The vertical displacement of point C end (say, at point A) and apply a force P at the other end (see Fig. 2.41). The
(in line with force P) is obtained by total energy will be determined in terms of P and partially differentiated with
taking the partial derivative of U
with respect to P:
( )

Replacing P with 100 kN, Q with 0,


LCA with m, LCB with 1 m, E with Fig. 2.41 FBD for application of energy method
210 GPa, and A with π(0.02 m)2 /4,
respect to P. To find the total strain energy of the system, we need to determine
we get (a) Truss (b) FBD of pin C
the load in each section of the rope which can be obtained by simple equilibrium
= 0.0058 m, nearly the Fig. 2.39 Introducing forces P and Q

34
analysis. Once we obtain the forces we can calculate strain energies by Solution:
evaluating . Table below organizes these calculations:
As before, we need to introduce dummy forces P and Q, respectively, at points F
Table 2.2 Calculation of elongation of the rope of Example 2.16 and B in-line with the displacement desired. The method of solution is exactly
the same as in the previous examples, but because of the larger numbers of
Segment Length, L Force, F F2L ( ) members involved we devise a scheme that will permit reduction of labour.
2
EF 2m P 2P 4P
DE 1.5 m P + 250 N 1.5P2 + 750P + 93750 3P + 750 The total strain energy of the truss will be the sum of the strain energies of the
CD 1.5 m P + 500 N 1.5P2 + 1500P + 375000 3P + 1500 individual members:
BC 2m P + 200 N 2P2 + 800P + 80000 4P + 800
AB 1.5 m P − 50 N 1.5P2 − 150P + 3750 3P − 150 ∑
The total strain energy is given as ∑ , where summation is taken The deflection in the direction of force P is obtained by taking the partial
over all the segments of the rope. The deflection at the free end is obtained by derivative of U with respect to P:
taking the partial derivative of the total energy with respect to P.
( ) ∑ ∑ ∑ (a)
(∑ ) ∑
( ) Attention is drawn to the fact that the here are the forces in the members in
where the value of ∑ has been substituted from Table 2.2. Plugging in the the presence of all loads, including P and Q. The values of may be imagined to
value of A = 0.0006 m2 and that of E as 100 MPa, and the value of P as 300 N, be made up of two parts, one due to the actual loads on the structure, and the
we get m, or 6.67 cm, the same as was obtained in Example 2.1. other due to the
application of dummy
Example 2.17 Unit force method loads P and Q. After
Consider the truss shown in we evaluate the sum
Fig. 2.42. The truss is specified in Eq. (a)
pinned at point A and is above, we plug in the
supported on rollers at value of P and Q as
point B. This implies that zero. This implies
while at A, there could be that in the final step,
horizontal as well as the value of ’s will
vertical reactions, there is be the original values
no horizontal reaction at B. of ’s without P and
All joints are assumed as Q.
Fig. 2.43
pin joints, and as has been The term , which
explained earlier, all
members are two force is the rate of change of with P can be interpreted as the additional force in the
Fig. 2.42 Example 2.17
members and hence carry ith member due to a unit load at P. This interpretation is valid because of the
only axial loads, tensile or compressive. We will illustrate the use of energy linear dependence on P of the additional load in the ith member.
method to determine the vertical displacement at joint F and the horizontal We, therefore, organize our calculations in two distinct parts: We first calculate
movement at the roller support B. All members are made of steel and have a the actual values of ’s, i.e., the v lues of ’s without P and Q. These are used
cross-sectional are of 6 cm2. as values for the first part of the terms in Eq. (a). We, then, calculate the values
of ’s th t would result when unit force is pplied in pl ce of P, without any of

35
the other external forces being present. These values of ’s will be used in pl ce m2)×(210 GPa) = 1.588×10−8 N−1. Table 2.3 gives the values of F and L/AE for
of the partial derivatives . all the members of the truss.
Calculation of through
This procedure is termed as the unit force method.
calculation of the Fi’s for a unit
load of P
Refer to Fig. 2.45 which shows
the FBD of the truss for a unit
load at the location of P and no
other load. Clearly, the reactions
at A and B are 0.5 N, each. We
again go from pin to pin, draw
the relevant FBD and calculate Fig. 2.45 FBD for unit load at F
the force in each member.
These have been shown in the 4th column of Table 2.3.

Fig. 2.44 Some FBD’s for determining the forces


Table 2.3 Calculation of vertical movement of pin C
Determination of ’s in absence of forces P and Q:
Member Force L/AE
We first determine the reactions R1x, R1y, and R2y by considering the structure as a
whole bin the FBD shown in Fig. 2.44a. The equilibrium equations are: Fi, kN (×10−8 N−1)
(×10−5 m)
∑ : ,
AD −16.7 1.24 −0.67 13.9
∑ : , and AF + 13.3 1.59 0.89 18.8
∑ : . BE −25.0 1.24 −0.67 20.8
BF + 20.0 1.59 0.89 28.3
The three equations can be solved simultaneously to give , , CD −16.7 1.24 −0.67 13.9
and . CE −16.7 1.24 −0.67 13.9
Once the reactions are known, we can go from pin to pin to calculate the axial CF +5.0 1.59 1 8.00
forces in all the members. Consider, for example, the FBD of pin B shown as DF 0 1.24 0 0.00
Fig. 2.44b. By writing the two force balance equations and noting that EF −8.3 1.24 0 0.00
( ) , we get Total 117.6
∑ : ( ) ,
The last column of Table 2.3 shows the product for each member of
∑ : ( ).
the truss. The sum of these over all the members gives the deflection of point D,
Solving the second of these first, we get , and then . in-line with the dummy force P. Thus,
Δv,E = 1.176×10−3 m, or 1.18 mm.
We can similarly move from pin to pin and calculate all the forces.
This is quite a general procedure for trusses.
We next calculate the value of L/AE for each member of the truss. For example,
for member AF which is 2 m long, the value of L/AE is given as (2 m)/(0.0006 Horizontal deflection at the roller support at B:

36
We next calculate the The last column of Table 2.4 shows the product for each member of
horizontal deflection
at the roller support at the truss. The sum of these over all the members gives the horizontal deflection
B. For this purpose, of point B (in-line with the dummy force Q), which is seen as 52.9×10−5 m,
we place a dummy slightly more than half a millimetre.
horizontal load at
point B. Example 2.18 Application of energy method to a statically
indeterminate problem
Here again, we use
Let us once again consider
the same procedure Fig. 2.46 FBD for a unit horizontal load at B
as above. The the same truss as in
calculation of Fi’s is Examples 2.16 and 2.17 and
does not depend on the dummy loads and we get the same values as in column 2 shown in Fig. 2.42 with one
of Table 2.3. The values of L/AE for the various members are also unchanged. change: The roller support is
replaced by a pinned
We, however, have to calculate dFi’s under the unit lo d Q. For this purpose we
support. This is as shown in
use an FBD as shown in Fig. 2.46 and calculate the resulting tensions in each of
the members. Clearly, the vertical reactions R1y and R2y are zero, and the Fig. 2.47. This will
horizontal reaction R1x is – 1 N. It is easy to see that there will be tension only in introduce a horizontal
the members AF and BF, both +1 N. reaction at support point B.
There will now be four
Table 2.4 organizes the calculation of the horizontal movement at the roller reactions and only three
support at B. Here the values in the second and the third columns (Fi, and L/AE) equilibrium equations to
are the same as in Table 2.3, but column 4 shows the values of the axial forces in determine them. The Fig. 2.47 A statically indeterminate truss
the various members when only the load Q (equal to 1 N) is applied at point B. problem, therefore, is
As was noted above, only two non-zero entries (for members AF and BF) result. statically indeterminate. We illustrate below the general strategy to solve such
problems using energy method.
Table 2.4 Calculation of horizontal movement of the roller support B
Solution:
Member Force L/AE The problem is quite
Fi, kN (×10−8 N−1) easily solved by making
(×10−4 m) the point B free to move
horizontally but applying
AD −16.7 1.24 0 0 a horizontal force Q ( in
AF + 13.3 1.59 1 21.1 kN) at this point to
BE −25.0 1.24 0 0.00 control its movement. Fig. 2.48 Replacing reaction with an undetermined force
BF + 20.0 1.59 1 31.8 The strategy consists of
CD −16.7 1.24 0 0 calculating the
CE −16.7 1.24 0 0 horizontal movement of point B in terms of the load Q, and then determining the
value of Q for which this movement is zero. It should be clear that this value of
CF +5.0 1.59 0 0
Q for which the movement is zero is the value of the horizontal reaction at point
DF 0 1.24 0 0 B.
EF −8.3 1.24 0 0
Total 52.9

37
The vertical components of the reactions remain unchanged from the ( )( )
determination in Example 2.16: and . The forces in
the various members can be calculated by considering the FBDs of the various
( ) , where δV is
pins. This is quite straight forward, and the values so obtained are tabulated as the volume of the element. For a finite body
column 2 in Table 2.5 below. The values in column 3 and 4 of this table will be then we can find the strain energy by
integration over the whole volume:

Table 2.5 Calculation of horizontal movement of the roller support B ∫ `


(2.15)
Member Force L/AE
It stands to reason that the stress ζxx does not
Fi, kN (×10−8 N−1)
do any work with the strains εyy and εzz, since Fig.2.49 An infinitesimal
(×10−5 m) these strains result in displacements which are element under tension
AD −16.7 1.24 0 0 perpendicular to this stress. Similar work will
AF + 13.3 + Q 1.59 1 21.1 + 1.59Q be done by the other stress components.
BE −25.0 1.24 0 0.00 Let us now look at the work done by shear
BF + 20.0 + Q 1.59 1 31.8 + 1.59Q stress components. Consider a shear element
CD −16.7 1.24 0 0 as shown in Fig. 2.50 where the lower
CE −16.7 1.24 0 0 surface is anchored and the upper surface has
CF +5.0 1.59 0 0 a shear stress of ηyx. Let the element undergo
DF 0 1.24 0 0 a shear strain of γyx as shown. This implies
that the upper surface with a force of ηyxδxδz
EF −8.3 1.24 0 0
acting on it undergoes a displacement of
Total 52.9 + 3.18Q γyxδy. The strain energy of this infinitesimal
element, thus, is Fig. 2.50 A Shear element
the same as in the corresponding column of Table 2.5, and then column 5 is
modified as shown. The horizontal displacement of point B under the action of a ( )( )
force Q at that point is thus (52.9 +3.18Q) ×10−5 m, with Q in kN.
( )
This displacement is zero when Q = − 52.9/3.18 = − 16.6 kN. Since the ctu l pin
B is restrained from moving, the reaction at point B must be 16.6 kN, inwards. For a finite body then we can find the strain energy by integration over the whole
volume:
We have, thus, solved the statically indeterminate problem. Once this is known,
we can calculate loads in all the members of the truss. ∫ ` (2.16)

Similarly for the other shear components. Combining the strain energies for
2.11 Strain energy in an elastic body normal and shear strains, we can write the equation for the general state of strain
We have so far considered the strain energy of axially loaded structures. Let us for a body of volume V as
extend this to a more general state of stress and strain. Consider an infinitesimal
∫ ( )
cubical element of dimensions δx, δy, and δz along the three coordinate axes. Let
this be acted upon by a force which produces a stress ζxx in the x-direction. The ` (2.17)
force acting on these faces is . If the strain in the x-direction is εxx, the
elongation is εxxδx. The strain energy of this infinitesimal element, thus, is

38
Summary
 The general strategy for solving the problems of the mechanics of where ν is termed as the Poisson ratio and is a property of the material. With
deformable bodies consists of three major steps: this, the net longitudinal strain in any direction is related to all the three
1. Consideration of static equilibrium and determination of loads in tensile stress components.
members of the structures. This equilibrium analysis invariably requires
drawing up strategically chosen free body diagrams (FBDs),  Change in temperature of a structural member introduces thermal strains so
2. consideration of relations between loads and deformations, and that the net strains of an element subject to a general state of stress and to a
3. consideration of the conditions of geometric compatibility. change in temperature is given by:

The second step above requires the considerations of the materials as well as ( ) (2.9)
the geometry of the structures. This step in itself may be divided into three
distinct phases: ( ) , and

a. We convert loads to stresses in the various members (macro to micro), ( )


b. the stresses are then converted to strains using the material properties
(micro to micro transformation), and finally,  We define the shear strain γxy in the element at a point as the distortion in the
c. with the use of the geometry of the structure, we determine shape of the element as measured by the change in angle between two
deformations from the strains so calculated (micro to macro). mutually perpendicular lines (parallel to x- and y- directions).
The strength of this method lies in the fact that the first and the third steps The shear stress is related to shear stress by the relation: . Shear
above require the consideration of the geometry of the structure (and are strain in a plane depends only on the shear stress in that plane and no other
independent of the material), while the second step requires considerations stress component. Heating of a material too does not introduce any shear
of only the material of the structure. Thus, the stress-strain relation obtained strain.
from the tensile test with its most simple loading conditions can be used to
predict behaviour of very complicated structure, since the second step above  The stress-strain properties of a material are determined by a tensile test. In
is independent of loading and geometry and depends entirely on the Sec. 2.6 we introduced the following concepts:
material. Proportional limit: The stress up to which the stress is linearly proportional
to strain.
 We may, at time, need to go in the reverse direction. We may be given the Elastic limit: The stress up to which the material upon unloading returns to
total deformation from which we need to determine the loading: we its undeformed length. Both proportional and elastic limits are difficult
calculate strains from deformation, convert strains to stress using material to determine.
properties, and then integrate stress to determine the loading. The strategy Permanent set: The residual strain in a material after unloading.
here too is macro to micro to micro to macro. Yield strength: The stress level at which the permanent set is 0.2% (or the
 There are many problems in which we cannot take the steps enumerated residual strain is 0.002). A material is modelled as elastic up till this
above in a linear sequence, because there are not enough equations of level of loading.
equilibrium to solve for all the unknown loads. In such cases we have to Strain hardening refers to the increase in yield strength of a material due to
consider simultaneously all the three steps, even if we were interested in its plastic straining. It is exhibited by materials in which the stress-
only one, say, in determining the forces in the system. Such problems are strain curve slopes upward in some part of the plastic region.
known as statically indeterminate problems. Examples 2.3 to 2.5 illustrate Ultimate strength is the maximum nominal stress in a material before
the methodology to be adopted for solving such problems. necking begins.
 As we apply an axial load to a member and it develops a strain εxx in the Necking refers to sudden and localized sharp reduction of cross-sectional
axial direction, there is also a transverse strain in the y- and z- directions. The area.
strain in a transverse direction is a fixed (negative) proportion of the strain in Most of the differences between the behaviour of ductile and brittle material
the axial direction. arises from the fact that there is little plastic deformation in a brittle

39
material. A brittle material undergoes fracture with very little strain,
much less than that for a ductile material. Glass, for example shows no
plastic deformation at all. Another feature to note is that the strength to
fracture is much larger in compression than in tension.
 As a structure is loaded the displacements of the points of application of
external loads result in work being done. This work done is stored as the
strain energy of the body given as ∑ . The work done on a simple
bar by an axial load is calculated as .
 Castigliano theorem states that if the total strain energy of a structural
system is expressed in terms of the external loads applied to it, the deflection
at any one loading point (in-line with the load) is obtained by taking the
partial derivative of the strain energy with respect to the load at that point:
.
The procedure consists of applying a load P at the point where the deflection
is to be calculated and in the direction in which the deflection is desired.
This may involve replacing an existing load with P, or inserting a dummy
load. The loads Fi’s in the v rious members re c lcul te in terms of the
unknown load P, and then the total energy U is calculated. After taking the
partial derivative of U with respect to P, the actual value of P is plugged in
(or P is replaced by zero, in case it is a dummy load). This gives the desired
deflection.
 In the case of more involved trusses, it was shown that we could simplify the
calculation of the deflection at the location of a load P by rewriting the
partial derivative as:
∑ ∑ ∑ .
Here we first calculate Fi’s without the lo d P, and then calculate the value
of as the increment in ’s for unit lo d P. This is known as the
unit load method whose calculations can be organized as in Table 2.3 or 2.4
 Statically indeterminate problems can also be solved using the energy
method. The strategy consists of relaxing one of the boundary constraints by
letting that boundary point move. A dummy load is introduced at that
location in place of the reaction that would have been there if the constraint
was not relaxed. The deflection of the boundary point is then calculated in
terms of that dummy load. We next obtain the value of the dummy load
required to make the movement of that point as zero. This value is the
desired reaction at that point.
 The strain energy per unit volume of the body is given by
( )
The total strain energy can be obtained by integrating it over the entire
volume of the body.
40
To obtain the relations between twisting moment, angle of twist, and the stresses
and strains we shall follow the strategy that we outlined in the last chapter, but in
a reversed order. The reason for this is that unlike the situations involving axial
loading treated in the last chapter, it is not possible to make the assumption of
uniform stresses in cases of torsional loading. In fact, it will be shown that the
stresses on the cross-section of a shaft vary with the distance from the axis of the
shaft.
For a constant diameter shaft, we shall first assume a total angle of twist θ for an
applied twisting moment T. We then use the geometric considerations to convert
this macro quantity to the strain at various points in the shaft. This is the macro
to micro stage. We next use the material properties to convert strains to stresses.
This is the micro to micro transformation. In the last stage we convert the
stresses into torque, a micro to macro conversion. This completes the solution to
the problem.

3 Torsion of circular shafts 3.2 Relating angle of twist to twisting moment


Fig. 3.2 shows a circular shaft25
of radius R and length L
clamped at one end and
subjected to a twisting moment
3.1 Introduction T. Let the shaft undergo a twist
through an angle θ as shown.
A slender rod which is subjected primarily to twisting or torsional load is termed We first determine the strain
as a shaft. One of the more important uses of torsional members is in the components in the shaft. For this
transmission of power or motion through purpose we make some
circular shafts. A shaft is used to transmit simplifying assumptions26: Fig. 3.2 A circular shaft under torsion
the torque generated by an electric motor
to the load it rotates. The torque produced  Circular cross-sections
by the engine of a car is transmitted to its of the shaft perpendicular to the axis remain plane, i.e., there is no
wheels by a shaft. When a coil spring warping of the shaft,
elongates, the wire constituting its coil is Fig. 3.1 Torsion bar used as  cross-sections of the shaft do not deform, i.e., there are no strains within
twisted. Torsion bars are also used as suspension in automobiles a cross-section of the shaft. Radial lines within the cross-sections remain
springs in automobile suspension (Fig. straight and radial, and
3.1).  there is no change in length of the shaft, i.e., there is no axial strain.
In problems involving torsion of shaft we may be interested in determining:
 The stresses that result when the shaft carries a specified twisting
moment. We may consequently determine the maximum twisting 25
We assume a circular shaft because many of the assumptions made here are not
moment that the shaft can carry without failure. valid for non-circular shafts
 The twist in the shaft when carrying a specified twisting moment. In 26
These assumptions can be shown to be true by using some fascinating
application to torsional springs, we may like to determine the spring symmetry arguments. See Crandall, Dahl and Lardner, An introduction to the
constant, which is the angular twist per unit twisting moment. mechanics of solids,2 ed., McGraw-Hill
41
The above assumptions imply that there are no linear strains. Therefore, using the Conversion of strain to stress is simple. Since there is no strain other than the
cylindrical polar coordinates as shown, shear strain γθz (and its complementary shear strain γzθ), there is only one
component of strain ηθz (and its complementary
shear stress ηzθ). These stress components on an
No distortion of the sections rules out shear strains and γrθ and γzr also vanish. element are shown in Fig. 3.5. The shear stress
The only non-zero component of strain is γrθ which we now proceed to component can be obtained by multiplying the
determine. strain with the shear modulus G:
Calculation of strain from macro distortion ( ) ( ) , Fig. 3.5 Shear stresses on
For this purpose we take a (3.2) an element
slice of length dz at a distance
where G is the shear modulus of the
z from the clamped end as
material. Please note that neither
shown in Fig. 3.3. The
the geometry nor the nature of
detailed geometry of
loading enters this step. The
distortion is shown in Fig.
variation of shear stress with the
3.4. Let the twist of the face
radius r is shown in Fig. 3.6.
at z be θ, and that at z + dz be
θ + dθ. To calculate the shear Converting stress into loading
strain in the shaft, consider a Fig. 3.6 The shear stress distribution on
Consider an elemental area in the
‘rect ngul r’ element ABCD the cross-section of the shaft
cross-section of the shaft at radius r
on the surface of the cylinder.
and angle θ as shown in Fig. 3.7. The shear stress at this location is ( )
After distortion, the shape of
this element changes to . If the area of the element is dA, the shear force on this element is
A1B1CD. The angular twist dθ Fig. 3.3 The slice whose distortion is studied
. It is easy to see that the force on an element of equal area which is
of this element is the angle that the diametrically opposite to this element is equal in magnitude but has the opposite
movement from A to A1 subtends at the axis direction. This is true of all elements of the cross-
passing through point O on the axis of the section and, therefore, the net shear force on the
shaft. What, then, is the strain? We see that section is zero. But that is not true for the moment
which was originally a right angle has about the origin. Every elemental shear force
now changed to . Thus, it has reduced contributes a counter-clockwise (hence, positive27)
by . Therefore, the strain here is equal moment, so they add up. The sum of moment can be
to which is seen as AA1/DA. Thus, found by simple integration. The force on the
the strain at r = R is element shown at a radius of r is stress times its area,
. or, . Therefore, its contribution to the
Fig. 3.4
twisting moment about O is radius times this force, Fig.3.7 Shear force on
It can be seen that the strain γθz is not an elemental area
constant with r. Consider the deformation at a radius of r. Here, the point E has
shifted to point E1, so that strain at this radius is given by EE1/dz, or,
27
Consistency of results requires that we define our co-ordinate directions and
( ) . (3.1) sign conventions very clearly. It is common to use only right- handed triad for
the direction of axes: Curl the fingers of your right hand from the positive x- to
Converting strain to stress
the positive y-axis. The thumb then points in the positive z-direction. We shall
use only right-handed triads in this book.
42
or, . Thus, the total twisting moment T is The maximum shear stress in a shaft occurs where the value of r is maximum,
i.e., at r = R. Using Value of Izz from Eq. 3.6 in Eq. 3.4, we get
∫ ∫
( )
The integral ∫ is termed as the second moment of the area about the polar
Thus, the maximum shear stress in a shaft for a given twisting moment T
axis of the section. Many authors term this as the polar moment of area. It is a
decreases as D3.
geometric property of the area and is denoted by28 Izz. This reduces the
expression for the twisting moment T to We illustrate below the use of these formulae.

(3.3) Example 3.1 Diesel generating set


We have completed the solution of the problem of the torsion of a shaft under a The shaft of a 2 MW diesel generating set rotates at 500 RPM. What is the
twisting moment. We can recast Eqs. 3.1-3.3 to obtain any quantity of interest. minimum diameter of the steel shaft connecting the diesel engine to the electric
generator if the shear stress in not to exceed 40 MPa?
Replacing dθ/dz from Eq. 3.3 in the expression for the shear stress η,
Solution:
( ) (3.4)
We first calculate the torque from the power being transmitted: P = T×ω. Here ω
The total twist θ is obtained by integrating dθ/dz over the entire length of the is the angular speed equal to 2π×RPM/60 = 52.36 rad/s. Torque T then is (2×106
shaft: W)/(52.36 rad/s) = 3.82×104 N.m. This is the twisting moment in the shaft. The
shear stress varies across the section of the shaft with the maximum stress
∫ ∫ (3.5) occurring at the surface at r = R, where R is the radius of the shaft.
The product GIzz is termed as the torsional rigidity of a shaft. The more its value, To calculate the stress in the shaft at r = R, we use Eq. 3.4 with the value of Izz
the less is the twist produced for a given length and twisting moment given by Eq. 3.6:

( )
3.3 Stresses and strain in a circular shaft
With T =3.82×104 N.m and η(R) = 40 MPa, we get R = [2×3.82×104 N.m/(π×40
We now evaluate the value of Izz for a circular shaft of radius R. We replace the MPa)]1/3 = 0.085 m, or 8.5 cm. Thus, the diameter of the shaft needs to be at
elemental area dA of Fig. 3.7 with its value rdθdr, and integrate over θ from 0 to least 17 cm.
2π, and over r from 0 to R:
What is the twist per meter run of the pipe?
∫ ∫ ∫ ( ) ∫ , or To obtain the twist angle θ per meter run, use Eq. 3.5 with L = 1 m:
for a circular shaft. (3.6) ( ) ( )
( ) , or 0.133o per meter run,
( ) , -
The torsional rigidity GIzz of a circular shaft varies as the fourth power of its a negligible amount.
radius or diameter.
Example 3.2 Torsion bar
Fig. 3.8 shows a torsion bar used as suspension in an automobile. The load from
the wheels is applied to a rigid arm as shown. Calculate the spring constant for
28
Many texts use the symbol J to denote the second moment of area about the the bar.
polar (z-) axis. We prefer here the symbol I with two subscripts denoting the
distance from the specified axis.
43
Solution: the right-hand thumb29 rule for directions of torques, it is pointed in the positive
z-direction, and hence, positive. We arbitrarily show the twisting moment at z to
The twist in the bar
be positive T. The moment balance of this FBD shows that T must be −40 kN.m.
then is given by Eq.
This is valid for all vales of z between 0 and 1.5 m.
3.5 as: . For
If, however, the value of z is larger than 1.5 m, the resulting FBD is as shown in
an applied force F, the Fig. 3.10b, and the value of T is −20 kN.m. The v ri tions of T with the value of
torque T is F×(0.7 m), z have been plotted in Fig. 3.10c. This is known as the Twisting or Torsion
L = 2 m, G for steel is Moment Diagram (TMD)30
80 GPa, and Izz =
π(0.04)4/32. Plugging Fig. 3.8 Torsion bar The value of shear stress is
in the values, we get given by Eq. 3.4. The maximum
for a load F, a twist of 6.96×10−5F. This twist results in an upward motion of the shear stress will occur in the
load point of 0.7×θ, or 4.87×10−5F. Thus, the effective spring constant for the section AB where the twisting
torsion bar spring is k = 20.53 kN/m. moment is the maximum. And
it will be at the surface where
Example 3.3 Power shaft the value of r is the maximum
(equal to R):
Fig. 3.9 shows a power shaft of diameter 4 cm transmitting power to two
machines. The shaft is driven by a belt drive that applies a torque of 40 kN.m at ( )
pulley A. The power take-off is again through belts at pulleys B and C. if the ( )( )
shaft diameter is 8 cm, determine the total twist of the shaft and the maximum
value of shear stress in the shaft.
[The diameter of the shaft is 8
cm throughout. The value of Izz
is πD4/32, or π(0.08 m)4/32 =
4.02×10−6 m4.] Fig. 3.10 Twisting moment diagram
This is a fairly large value, close
to the failure strength of structural steel. We probably need a thicker shaft to
give it a margin of safety.
The twist of the shaft section AB of length L = 1.5 m which carries a uniform T of
−40 kN.m is given by Eq. 3.5 s
Fig. 3.9 Power shaft

Solution:
29
Right hand thumb rule states that if you curl the fingers of your right hand in
We first draw a diagram depicting the variation of twisting moment along the the sense of the torque, the thumb points in the direction of the moment vector.
axis of the shaft. To determine the twisting moment T at any location in the 30
These TMDs are a big help in the analysis of shafts which do not have uniform
shaft, we externalize the moment there by taking a section at that point and draw twisting moments along them. We can draw the TMDs directly without drawing
an FBD. Fig. 3.10a shows an FBD for determining the value of T for all values FBDs if we follow the following procedure: (1) start from T = 0 at the left most
of z between 0 and 1.5 m. The torque acting on pulley A is 40 kN.m, and using end; (2) at every location where a concentrated moment To acts, decrease the
value of T by To, if To is positive, or increase by To if it is negative. We will
come across similar procedures for drawing shear force diagrams (SFDs) and
bending moment diagrams (BMDs) in Chapter 4.
44
( )( )
( )(
∫ ( ) .
)

A reasonable value of G for steel is 80 GPa. We insert here the value of Izz in terms of the diameter D(z), which, in turn, is
expressed in terms of z as above.
Similarly, the twist of shaft section BC is exactly half of this, since it carries only
h lf the twisting moment (−20 kN.m), i.e., 0.093 r d. ∫ ( ) , * +
= 0.024 rad or about 1.4
-
o
The tot l twist of the sh ft is the lgebr ic sum of the two: −0.279 r d or −16 . degrees.
The negative sign indicates that the end at C twists clockwise with respect to the Where and what will be the magnitude of the maximum shear stress in this
end A. tapered shaft? The shear stress is maximum (at a given section) at the outer most
location, i.e., at r = R. Thus,
Example 3.4 Twist in a tapered shaft
We next consider a tapered shaft in which the polar moment of area, and ( )
therefore, contribution to the twist varies
along the axis of the shaft. Fig. 3.11 Since Izz varies as R4, and T is constant along the shaft, the value of maximum η
shows a 1 m long tapered steel shaft along the length of the shaft varies like R−3, i.e., the maximum value of the shear
subjected to a twisting moment of 1 stress along the length occurs where R has the minimum value. The maximum
kN.m. If the base diameter is 10 cm and shear stress, therefore, occurs at the tip where D is 0.07 m, Izz is π×(0.07 m)4/32 =
the tip diameter is 7 cm, determine the 2.36×10−6 m4, and T = 1 kN.m.
total twist of the shaft. Fig. 3.11 Tapered shaft ( ) ( )
( )

Solution: Example 3.5 Cable in a sleeve


The contribution dθ to the twist Fig. 3.13 shows a 4 mm dia long cable in
of the shaft by a section of a sleeve used to control a setting
length dz at z is given by remotely. As we turn the knob the
(T/GIzz)dz. In this problem the connecting cable twists inside the sleeve.
twisting moment T is constant The sleeve is provided so that the cable
along the length of the shaft, but does not flip. The friction between the
Izz varies with z. It can be sleeve and the cable applies a torque on
verified that the linear variation the cable which needs to be overcome.
of diameter D is given by This torque is estimated to be about 0.3
( ) ( ). N.m per meter run of the cable. If the Fig.3.13 Cable in a sleeve
The value of Izz varies as fourth torque needed at the setting point is 0.5
power of the diameter, i.e., as N.m, determine the maximum length of the cable, if the cable material can
Fig. 3.12 Variations of twisting
πD4/32. Fig. 3.12 shows the moment, area, polar moment of withstand a shear stress of 100 MPa. What would be the play? (The play is the
variations of the twisting area, and the contribution to twist angle through which you need to turn to effect a change in setting when you
moment, area, the polar moment along a tapered shaft reverse the setting.)
of area, and the contribution to
twist along the length of the tapered shaft. The total twist can be obtained by Solution:
integrating the contribution of an elemental length. Thus,
Fig. 3.14 shows the TMD of a cable of length z. The maximum torque in the
∫ cable is (− 0.5 −0.3z) N.m, and occurs at the knob-end. The maximum shear

45
stress would be at the surface of the cable, i.e., at r = 0.002 m. Eq. 3.4 gives kN.m. This torque is negative, i.e.,
( m) ( )N.m
clockwise looking from the right end.
( ) ( )
( m)
Fig. 3.16 shows the twisting moment
Using the condition that diagram for the two shafts.
maximum shear stress is
limited to 100 MPa31, we The maximum torque in a shaft is
get the maximum value of z given by ( ), where
as [(100×106/79.6×106) – R is the radius of the shaft. The
0.5]/0.3 m, or 2.5 m. maximum value of the shear stress in
The maximum twisting the first shaft is ( )
moment, then, is (−) 1.25 ( ) , ( ) -
N.m, and the twist is given . This is quite safe. The Fig. 3.16 Twisting moment diagram for
Fig. 3.14 TMD of the cable
by Eq. 3.5 as maximum value of the shear stress in geared shafts
( ) ( )
or 88o. This is the play in the cable. the second shaft is ( ) ( ) , ( ) - . Thus, the
( ) , ( m) - maximum (shear) stress in the shafts is 70.8 MPa.
Example 3.6 Geared shafts Let us next find the total twist at the free end. The twist in the each of the two
shafts is obtained from the equation .
Fig. 3.15 show two steel shafts geared together. One end of the first shaft is built
in a wall. Find the maximum shear stress in the shafts and the total rotation of Twist in the first shaft, ( ) ( )
the free end of the second shaft. ( ) , ( ) - , counter-clockwise
looking from right.
Solution: Twist in the second shaft (with respect to the geared end
The first step in of the shaft,
solving this problem ( ) ( ) ( ) , ( ) -
is to determine the , clockwise looking from right.
twisting moments To find the total twist of the free end with respect to the
along the two shafts
fixed end of the first shaft we cannot just add the two
and to draw the
twists. Consider the motion of the gears as shown in Fig.
twisting moment
3.17. Angle θ1 represents the clockwise movement of the
diagrams. Clearly, larger gear (on shaft 1). It is clear that angle θ2 which Fig. 3.17
the torque in the Fig. 3.15 Geared shafts
represents the counter-clockwise movement of the Geometry of twist
thinner (second) shaft smaller gear due to gearing alone is 10/6 of θ1. This is the rotation of the left end
is 3 kN.m throughout
of shaft 2. Since θ1 is 0.0064 rad, angle θ2 is 0.01 rad counter-clockwise. The
its length. To determine the torque in the thicker (first) shaft one has to be
twist of shaft 2, which is 0.03 rad counter-clockwise is superimposed on the
careful. The torques in the two shafts are not equal. (Why? Why does a simple
motion of the gear, to obtain the total rotation of the free end as 0.01 rad + 0.03
torque balance not give the correct result?32) The problem can be solved if it is rad = 0.04 rad or 2.3 degree.
realized that the contact force F between the two gears must be equal and
opposite. And therefore, the torque in the first shaft is 10/6 of 3 kN.m, or 5
3.4 Hollow shaft
31
We need to use a negative sign with ηmax here. We shall see here that hollow shafts provide excellent rigidity for weight of
32
It is because we have not shown the bearings that will be required to hold the material. Consider a hollow circular shaft of inner radius Ri and an outer radius
shafts in place. The bearings will apply forces on the shaft which will produce of Ro as shown in Fig. 3.18. The value of Izz can be obtained by replacing dA in
moments that should also go in the moment balance equation. r2dA with rdθdr and integrating over θ from 0 to 2π and over r from Ri to Ro.
46
( ) cannot be determined by considerations of equilibrium alone. In such shafts the
∫ ∫ ∫
geometrical constraints on twists need to be invoked to solve the problems.

( ) (3.7) We follow the same three-step process that was invoked in the last chapter:

The maximum shear stress in the shaft will be near  Use equilibrium analysis to write equations for the twisting moments in
the surface, i.e., at r =Ro. Therefore, the maximum the various parts of the shaft. There may not be enough equations to
stress in the shaft ( ) ⁄ varies determine the moments explicitly.
inversely as Izz. If we define the torque carrying  Convert twisting moments to twists in each part.
capacity of the shaft as the maximum torque the  Write geometrical compatibility conditions to complete the analysis.
shaft can carry without the stress exceeding a Fig. 3.18 Hollow We illustrate the procedure with a few examples.
specified limit, it is clear that this varies directly as cylindrical shaft
the value of Izz (for a fixed value of Ro). The curve Example 3.7 A shaft built in at both ends
labelled (a) in Fig. 3.19 which plots the variation of Izz (as the fraction of the Izz of
the solid shaft of radius Ro) also represents the variation of the torque carrying Consider a composite steel
capacity. The curve (b) plots the variation of the weight (which varies as the area shaft ABC built up of two 1 m
of cross-section) of long steel sections, AB of 10
the shaft. The curve cm dia and BC of 15 cm dia
(c) shows the (Fig. 3.20). A torque of 100
torque carrying kN.m is applied to the collar
capacity per unit in the middle. The two ends
weight of the shaft of the shaft are built into
showing clearly the walls so that there is no
cost effectiveness33 rotation of the ends. What is
of a hollow shaft34. the twist produced in the
shaft?

3.5 Statically Solution:


indeterminate Equilibrium analysis
(a) Variation of the value of Izz of hollow shaft
shafts (b) Variation of the weight of hollow shaft Let the reactions at the two
There are many (c) Variation of the ratio of strength to weight ends be T1 and T2 as shown.
situations in which From the condition of the
Fig. 3.19 Comparison of hollow shaft with solid shaft for
the twisting various ratios of the internal and external radii equilibrium of moments we
moments in shafts get T1 + T2 + 100 kN.m = 0. It
should be clear that either T1 (a) The composite built in shaft
or T2 or both should have a (b) FBD for determining torques in part AB
33 negative sign. This is the only (c) FBD for determining torques in part BC
This, of course, has not taken into account the increased manufacturing equation we get from the (d) Torsion or twisting moment diagram
cost of the hollow shaft. statics for the two unknown
34
This is quite as expected. An element nearer the surface has larger stresses and reactions T1 and T2. Clearly Fig. 3.20
contributes more to the torsional moment. Therefore, the material nearer the this is a statically
surface is used more effectively to bear the torsional load than an equal area near indeterminate problem. We need to consider deformations as well.
the axis. Seen in another way, it is an area nearer the outer surface that makes a
larger contribution to Izz, the value of the polar moment of inertia.
47
We need to determine the twisting moments in the two halves to calculate the θAB = −1.92×10−6T1 =
−6
twists therein. For this purpose, we take a section first in the part AB and draw an (−1.92×10 ) × (−16.6 kN) = 0.032
FBD for the left part as shown in Fig. 3.20b. Here T is the twisting moment in rad
the shaft. The equilibrium of this FBD gives T1 + T = 0, or T = − T1. Fig. 3.20c
shows the FBD for determining the twisting moment in the second part of the Example 3.8 Meshed gears
shaft. From this we get T1 + T + 100 kN.m = 0, or T = − T1 – 100 kN.m. Fig. Fig. 3.21 shows two shafts, one of dia
3.20d shows the complete twisting or torsion moment diagram (TMD) of the 6 cm and the other of 4 cm, carrying
shaft35. If we have chosen the sign of T1 correctly, then both halves of the shaft identical meshed gears in the middle.
have negative twisting moments36. The length of each shaft is 1 m. The
Relating torques and twists two shafts are disengaged, the top
shaft A rotated by 10o in the direction
Now that we know the twisting moments, we can calculate the twist using Eq. shown, and then the two gears are
3.5: ⁄ . meshed again. Shaft A will untwist a
The values of Izz are: bit twisting shaft B till the
equilibrium is reached. Determine the (a) TMD for top shaft
for shaft AB: πD4/32 = π(0.1 m)4/32 = 9.81×10−6 m4 locked in torque at equilibrium. (b) TMD for bottom shaft

for shaft BC: πD4/32 = π(0.15 m)4/32 = 49.7×10−6 m4 Fig. 3.22 TMDs of the two shafts under
Solution: the assumptions that the reactions on the
( )( )
( )( )
, and Equilibrium requires that the contact left wall are T1 and T2, respectively.
( )( ) force between the gears be equal and
( )( ) opposite. Since the two gears are identical, the torques in the two shafts are
equal but opposite. Let this residual torque be To, positive To in (the left part of)
Geometric compatibility shaft A and negative To in (the left part of) shaft B.
Since both ends of the shaft are Let us assume that wall at left applies on shaft A a reaction torque T1, and on
built in, the net twist in the shaft shaft B a reaction torque T2. The TMDs of the two shafts can be calculated and
must be zero: θAB +θBC =0. This are as shown in Fig. 3.22. These have been drawn under the assumption that T1
gives: and T2 are both positive.
−1.92×10−6T1 – Now let us look at the deformation of
−6
0.38×10 T1 – 0.038 = 0, which shaft A. It is clear that the twist in the
gives left half must be the same but opposite
T1 = −16.6 kN of that in the right half, for the total
twist from end to end to be zero. Since
With this value of T1, we can the two halves have identical
calculate the twist at point B as Fig. 3.21 Two meshed shafts
geometries, it stands to reason that the
torque in the two halves must be equal
and opposite. There is, thus, only one
35
The reader can verify that we could have drawn this TMD directly without solution: the twisting moment in the
drawing FBDs if we had followed the procedure outlined in footnote 5. left half is + To/2, and that in the right (a) TMD for top shaft
36
It is clear that the sign of T1 that we have chosen here cannot be correct h lf is − To/2. Similarly, we can argue (b) TMD for bottom shaft
because with this we have negative twisting moments in both halves of the shaft. that the twisting moment in the left
This, of course, is not possible since if one part twists clockwise, the other must half of shaft B be − To/2, and that in the Fig. 3.23 Actual TMDs of the two shafts
twist counter-clockwise. But it does not matter for our analysis here.
48
right half be + To/2. These are shown in Fig. 3.23. strain with the value of G = G1 for the core material, and in the outer sleeve for r
between r1 and r2, we multiply with the value of G = G2 for the sleeve material,
Geometric compatibility
which is higher. Thus, the graph for shear stress is kinky with two different
Refer to Fig. 3.24. Shaft A was rotated by 10o when slopes, one for the core and the other for the sleeve. This is shown in Fig. 3.26.
the two gears were engaged. After engagement and
The next step in Section 3.2 was considering the shear force on a small element,
letting go, the shaft A unwinds by an angle θ2
calculating its contribution to the twisting moment about the axis, and integrating
which is the same as winding up of shaft B (since
over the entire cross-section:
the gear ratio is one). The residual twist in shaft A
is now θ1. It should be clear that the geometric
compatibility condition is: Fig. 3.24 Geometric ∫ ∫
0
θ1 + θ2 = 10 = 0.175 rad compatibility condition
Here, unlike what we did in Section 3.2,
We can calculate the twists θ1 and θ2 using Eq. 3.5: we cannot take G outside the integral
θ = TL/GIzz. Thus, since it is not constant over the section.
( ) ( m) But we can do it part-wise: we divide
, and the area in to two parts, one, with r
( G ) , ( m) -
between 0 and r1, which covers the core Fig. 3.26 Shear strain and shear
( ) ( m) stress distribution across a
. with modulus G1, and the other with r
( G ) , ( m) - composite shaft
between r1 and r2, which covers the
Using these in the geometric compatibility condition, we get sleeve with modulus G2.
2.46×10−6To + 12.45×10−6To = 0.175
∫ ∫ ∫
This gives To, the locked-in twisting moment as 11.7 kN.m.
2∫ ∫ 3
3.6 Composite shaft 2 ∫ ∫ 3
Let us next consider twisting of a composite shaft that is made up of inner core of
one material clad with an outer sleeve of another material as shown in Fig. 3.25. or ( ) (3.8)
To solve this problem, let us look carefully through the derivation of the torsion
formulae in Section 3.2 and figure out the changes we would need to make on where I1 is the polar moment of the core area and I2 is the polar moment of the
account of two different materials in the cross-section. sleeve area. Compare this with Eq. 3.3. The term GIzz for a uniform shaft is
replaced with (G1I1 + G2I2). Similarly,
We had assumed a twist angle θ and showed
using geometry that there is only one shear ( )
(3.9)
strain component γzθ which is given by Eq. 3.1
The torsional rigidity of the composite shaft is, thus,
as ( ) . This does not change with
the material. Thus, here too, stain γzθ varies ( ) ( ).
linearly with the distance r from the longitudinal Here , and ( ) .
axis, independent of material.
We had next converted the shear strain to shear
stress by multiplying it with G, the shear Fig. 3. 25 A composite shaft
modulus. Here the material dependence arises.
In the inner core up to r = r1, we multiply the
49
3.7 Torsion of thin-walled tubes shear force on this element is qδs whose moment about an arbitrary point O is
hqδs, which is 2qδA, where δA is the grey area in Fig. 3.28a. The total twisting
We have so far found exact solution to the problem of torsion in circular (and moment, then, is simply the shear flow q times twice the area A of the cross-
hollow circular) shafts. It is not possible to find such solutions for shafts of
arbitrary cross-sections. However, we can find quite easily the approximate
solution for a thin-walled shaft of arbitrary section. Consider such a shaft as
shown in Fig. 3.27. We use the n, s, z
co-ordinate system as shown, with
the coordinate axis s along the central
line of the tube wall. The dominant
stress component in this case is ηzs
(along with the complementary stress
component ηsz). We shall assume,
without proof, that the other stress (a) (b)
components are either absent or are
Fig. 3.28 Relating shear flow to the twisting moment
negligibly small.
section of the tube: , from which we can extract the value of the shear
We show in Fig. 3.27b a small
(a) (b) flow38 as:
element of length dz of the shaft.
The shear stress ηzs is distributed over Fig. 3.27 A thin-walled torsion tube (3.11)
the top and bottom surface of the
element while the complementary Example 3.9: A hollow circular shaft
shear stress ηsz is distributed over the surfaces marked 1 and 2 in the figure. We
Let us evaluate the stresses in a hollow circular shaft by the approximate method
introduce here the concept of shear flow q defined as the shear force per unit
outlined in Sec.3.7 and compare the results with those obtained by the exact
length of the tubular surface:
method. Consider the hollow circular steel shaft of length 1 m shown in Fig.
∫ (3.10) 3.29. It is subjected to a twisting moment T = 100 N.m. Compare the stresses
obtained by the exact and the approximate methods.
In Fig. 3.27b we have shown the net shear forces on the vertical surfaces as shear
flow q times the length dz of the surface. The first thing to notice is that shear Solution:
force per unit length on the vertical surface 1 is the same as on the horizontal Eq. 3.4 gives the stresses in a circular
surface. The next thing to notice is that the two forces shown are the only
significant vertical forces on the element, and therefore, q1 = q2, that is, the shear shaft: ( ) , where Izz for a hollow
flow is the same at two arbitrary positions, and hence everywhere, along a cross- shaft is given by Eq. 3.7 as (
section of the tubular shaft37. ).
We can now relate the shear flow q to the twisting moment T. Consider an For the given values,
element of length δs in the cross-section of the shaft as shown in Fig. 3.28a. The ,( ) ( ) - Fig. 3.29 Hollow circular shaft
.

37
This of course implies that if the wall thickness is constant, the shear stress is
constant too. And also that as the wall thickness increases the shear stress
38
decreases, and vice-versa. The nomenclature shear stress originates perhaps from One interesting fact about this derivation is that this relation has been obtained
the fact that there is a definite analogy between shear flow in the thin wall of a without any reference to the material properties, and therefore, is applicable both
shaft and the flow of an incompressible fluid in a channel. to elastic and plastic deformations.
50
The shear stress at r = 20 mm, then, is ( ) ( ) deformed up to point A in Fig. 3.32, it follows
13.5 MPa, and at r = 16 mm is ( ) ( ) a straight line AB with slope G, the shear
10.8 MPa. The shear stress varies linearly between these two values across modulus.
the thickness of the shaft.
As a shaft is twisted, and the assumptions
The shear flow q by approximate procedure is given by Eq. 3.11 as , where A about its behaviour that plane sections remain
is the area enclosed by the mean line of the section (here a circle of radius 18 plane and do not distort hold, the shear strain
mm). Thus, A = π(0.018 m)2 = 1.02×10−3 m2, and ( ) ( will vary linearly with the radius ( ( )
) = 49 N/m. The average shear stress can be obtained by dividing the ⁄ ) as shown in Sec. 3.2. The strains
shear flow q by the wall thickness t to get ( ) ( ) are then converted to stresses using the stress- Fig. 3.31 Elastic-plastic behaviour
The maximum stress is underestimated by a mere 9%. strain relation. Fig. 3.32a-d show the shear
stresses for various levels of straining. When the maximum shear strain is less
Example 3.10 A thin rectangular than γY, the shear stress distribution is linear as shown in Fig. 3.32a. The
tubular shaft maximum shear stress is less than ηY. As the twisting increases, the value of the
maximum strain increases, the stresses increase till the stress and strain reach the
Consider a tubular shaft of rectangular
section as shown in Fig. 3.30.It is subjected
to a twisting moment of 100 N.m. What are
the stresses in the various sides of the Fig. 3.30 A rectangular tubular
rectangle? shaft

Solution:
The shear flow q by approximate procedure is given by Eq. 3.11 as , where A
is the area enclosed by the mean line of the section. The value of A for the given
(a) (b) (c) (d)
section is (22 cm – 2 cm)×(12 cm – 2.5 cm) or 0.19 m2. The shear flow is
( ) ( 0.19 m2) = 263.2 N/m. The shear flow is constant Fig. 3.32 Shear stress distribution in an elastic-plastic shaft
throughout the tube. The shear stress in the shorter (thinner) sides is q/t, or
(263.2 N/m)/(0.02 m) = 13.2 kPa, and the shear stress in the longer (thicker) sides yield values as shown in Fig. 3.32b. The slope of the stress-radius line increases
is (263.2 N/m)/(0.025 m) = 10.5 kPa. as shown. Thereafter, the plastic behaviour kicks in. The yield value of shear
stress is obtained at a radius r = ro < R. The shear stress for r between 0 and ro
increases linearly from 0 to ηY, and thereafter remains constant at ηY for r > ro.
3.8 Plastic deformation in torsion This is shown by the stress distribution of Fig. 3.32c.
Let us next look at what happens when a shaft is loaded beyond its plastic limit. For Fig. 3.32c, the shear stress distribution is given as 40:
This discussion will serve as a simple model of how problems in elastic-plastic
deformations can be handled. Consider a shaft made up of an elastic-plastic ( ) ( )
material whose stress-strain behaviour can be modelled as shown in Fig. 3.31. , for (a)
The material is linearly elastic till the yield shear stress level39 ηY is reached, after
which the material deforms perfectly plastically with the stress remaining As the twisting moment increases further, the value of ro, the radius at which the
constant at the yield stress value. If we unload such a material after it has been yield value is reached decreases and more of the shaft is under plastic

39
Fig. 3.30 is similar to Fig. 2.29c, except that linear stress and strain have been
40
replaced with shear stress and strain. It will be shown later in Sec. 6.10 that the Since the value of dθ/dz is not known, γzθ is not determined, and, therefore, ro
yield shear stress ηY is related quite simply to the yield stress ζY: ηY = ½ ζY. is undetermined thus far.
51
deformation. Fig. 3.32d shows the condition when the entire shaft has undergone Using these values and the fact that the shear strain is given by ( )
plastic deformation. ⁄ for all r, we get ⁄ ( ) 1.56×10−3/11.2×10−3 m = 0.14
m−1. Therefore, the twist per meter ⁄ is 0.14 rad/m.
The shear stress distribution is related to twisting moment by using the same
procedure we followed in Sec. 3.2: We take a small area element rdθdr at radius
r and determine the shear force on it as ηrdθdr. The contribution to the twisting 3.9 Limit Torque
moment of this elemental force is obtained by taking its moment about origin (by
multiplying the force with radius r) and integrating over the entire area: In the preceding section we have seen the difficulties that arise when we
undertake the analysis of both elastic and plastic deformations. The analysis was
∫ ∫ ( ) (b) possible at all because we were taking a very simple case of pure torsion on a
uniform circular bar.
If we plug in Eq. (b) the value of η as given by Eq. (a), we can obtain the value of
ro. Once ro is obtained, we can determine the value of dθ/dz and complete the We can however work with a simpler model
solution of the problem. in much of engineering work with
complicated structures. In this model which
Example 3.11 Twist in a shaft with plastic deformation is termed as limit analysis, we take a fully-
plastic stress distribution and omit the earlier
A shaft of 4 cm dia is subjected to a twisting moment of 2 kN.m. Determine the
elastic-plastic considerations. The load for
twist per meter length of the shaft. The yield shear stress for steel is 125 MPa.
which the structure first becomes fully-plastic
Solution: is termed as the limit load.

Let us first assume that only elastic deformation takes place. Eq. 3.4 gives the Limit analysis is based on the idea that any
maximum shear stress as ( ) ( ) structure made up of materials with fairly
Fig. 3.33 Fully-plastic stress
, ( ) - , well above the yield value. Following the well-defined yield point will not undergo very distribution
derivation and notation of the last section, let ro represent the radius at which large deformations till all of it is in the plastic
plastic deformation is reached 41. Then, region. And, therefore, deformations will be small as long as the load is less than
the limit load for that structure. Limits load, in general, are much easier to
∫ ∫ ( ) ∫ ( ) calculate. Let us consider a shaft in the plastic limit when the whole cross-
section is in the plastic state. The stress distribution is then very simple, as
∫ ( ) ∫ shown in Fig. 3.33. The calculation of the limit torque 42 for such a distribution is
quite simple:
0 ∫ ∫ 1
∫ ∫ ( )
0 ( ) ( )1 (a) ∫
Plugging in the values of ηY = 125 MPa, T = 2.0 kN.m, and R = 0.02 m, we get ro As long as the applied torque is below this value, some part of the structure will
= 0.0112 m, or 1.12 cm. The stress at this radius is the yield shear stress 125 be in the elastic range and the twist will not be excessive. Limit analysis is quite
MPa. For the value of shear modulus of 80 GPa, this gives a shear strain of 125 extensively used, at least as a first approximation, in the design of structures.
MPa/ 80 GPa = 1.56×10−3 at r = 0.0112 m.

42
The limit load for this case can be obtained quite easily from Eq. (a) in the
41
We cannot find the value of ro by using Eq. 3.2 because it has been derived solution of Example 3.10. The limit load corresponds to ro vanishing.
under the assumption that the deformation is linear elastic everywhere. Replacing ro by 0, gives the same result as here.
52
3.10 Strain energy in torsion Since Castigliano theorem requires that the total energy be differentiated with
respect to torque applied at the point where the twist angle is to be determined,
We had introduced in Sec. 2.9 the concept of strain energy. The energy method we replace the torque of 100 N.m by T. The TMD of the composite shaft, then,
based on Castigliano theorem was introduced in Sec. 2.10 where a number of is as shown in Fig. 3.35. The total strain energy is given by:
examples involving axially-loaded members were solved. The method was also
applied to a statically indeterminate problem. We introduced in Sec 2.11 the ( ) ( ) ( ) ( ) ( )
6 ( ) ( ) ( )
7,
calculation of strain energy for an arbitrarily loaded structure. We now extend ( )
the energy methods to cases involving torsion.
and
Let us consider a shaft of radius R and of length L subjected to a pure torsional
( ) ( ) ( ) ( ) ( )
moment T. The strain energy is obtained from Eq. 2.16 using the stress 0 1
( )
distribution given by Eq. 3.4.
Using 100 N.m as the value of T, we get θ = 3.9 rad, or 223o, a fairly large twist.
∫ ∫ ∫ . / Let us, as a check, calculate the maximum shear stress in the shaft. The
maximum shear stress in a circular shaft is given by . This has
∫ ∫ ∫ been obtained by substituting the value of Izz in terms of R, and determining the
stress η at r = R. This gives the values 47.7 MPa for the part A of the shaft, 46.6
∫ (3.12) MPa for the part B, and 63.3 MPa for the part C. These appear to be safe even as
the twist appears to be excessive.
where A is the cross-sectional area and the integral of r2 over A has been replaced
by the polar moment of area Izz. Example 3.13 Coil
spring
Castigliano theorem can be quite easily extended for the case of a torque load T
to give the twist θ as: Fig. 3.36a shows a
closely-wound coil spring
(3.13) consisting of n turns of a
We now illustrate the application of Castigliano theorem. wire of radius of r wound
into a coil of radius R.
Example 3.12 Stepped shaft Determine the elongation
of the spring under the
Consider the stepped steel shaft shown in Fig. 3.34. It consists of three parts: action of an axial force P,
part A (dia 20 mm, length and, thereby, obtain the
1 m), part B (dia 16 mm, spring constant. (a) (b)
length 1.5 m), and part C Fig. 3.36 Coiled spring
(dia 10 mm, length 2.0
m). It is subjected to three
twisting torques of 300 Solution:
N.m, 200 N.m and 100
N.m as shown. This example
Determine the total twist provides a
of the free end of the dramatic
Fig. 3.34 Stepped shaft illustration of the
shaft.
power of energy
Solution: method and of
Castigliano
53

(a) (b)

Fig. 3.35 TMD of the stepped shaft Fig. 3.37 Statically indeterminate shaft
theorem. Fig. 3.36b shows the FBD of the spring with the wire cut at an arbitrary The twists of the two shafts are obtained by partially differentiating U with
location. We can, by equilibrium of moments conclude that the twisting moment respect to T1 and T2, respectively. Equating the two twists, we get
T on the wire is clearly equal to PR, the product of the load P and the radius R of
the coil, independent of the location of the cut. Thus, the twisting moment
everywhere in the wire is PR. Therefore, the strain energy due to the twisting
moment in the wire constituting the spring is obtained by writing the strain Here, the value of Izz,1, the polar moment of inertia of the steel rod is πD4/32 =
energy dU of an elemental length and integrating it over the entire length of the π(0.050 m)4/32 = 0.61×10−6 m4, and that of Izz,2, the polar moment of inertia of
wire, i.e., for n coils of radius R. The total energy of the spring is given by the luminium tube is π(Do4 – Di4) /32 = π(0.0804 – 0.604) m4/32 = 2.75×10−6 m4.
The values of G1 and G2 are about 80 GPa and 26 GPa, respectively. Using the
( ) ( ) ( )
∫ ∫ ( ), since the total length of value of L as 0.5 m, the equivalence of twists gives:
the wire is 2πnR. 3.90×10−6T1 = 6.99×10−6T2, or T1 = 1.79T2 (b)
We can evaluate the shear force acting on the wire as V = P. These shear forces Solving Eqs. a and b simultaneously, we obtain the values of T1 and T2 as 3.2
also contribute to the strain energy, but it can be shown that their contribution is kN.m and 1.8 kN.m, respectively. The (equal) twists in the two shafts are then
of order (r/R)2 of the contribution of twisting moments, and hence can be obtained as 0.0125 rad or 7.17o.
neglected.
Use of Castigliano theorem then gives the deflection of the spring in the direction Summary
of application of force P as:
 When a circular shaft is subjected to a pure torsional load, there is no
( ) ( ) warping or distortion of cross-sectional planes, and consequently there
( )
is only one component of strain, namely γzθ (and its complementary
The spring constant strain γθz).
o We first assume a twist angle θ and determine the value of the
strain γzθ as rdθ/dz using purely geometrical considerations.
Example 3.14 Statically-indeterminate torsion problem
o We next convert the shear strains to shear stresses using the shear
Consider a composite shaft consisting of a solid steel shaft and an aluminium modulus G: ηzθ = Gγzθ = Grdθ/dz.
tube as shown in Fig. 3.37. If a torque of 5 kN.m is applied to the rigid disc o We next take a small element within the cross-section of the shaft,
attached to the left end, determine the twist of the composite shaft. write an expression for shear force acting on it in terms of the shear
stresses, take its moment about the axis of the shaft and integrate
Solution: over the entire cross-sectional area to obtain the twisting moment T
as GIzzdθ/dz. Here Izz is the second moment of area (polar) of cross-
Let the torque carried by the steel rod and the aluminium tube be T1 and T2, section, defined as integral of r2dA over the cross-section. The
respectively. Fig. 3.37b shows the FBD of the rigid end disc. From the value of Izz for a solid cylinder was found to be πD4/32, and for a
equilibrium of this disc: hollow shaft as ( ) .
o The twist θ is given by TL/GIzz.
T1 + T2 = 5 kN.m (a)
 The quantity GIzz is seen as the torsional stiffness which is defined as
Since there is no other equilibrium equation, this problem is statically the torque required to produce a unit twist per meter length of a shaft.
indeterminate. All we have besides Eq. a is the geometric compatibility  Since an area nearer the surface has a larger values of stresses that
condition that the twists θ1 and θ2 of the two shafts are equal. Let us determine contributes more to the twisting moment than an equal area nearer the
θ1 and θ2 by energy method. The total strain energy is given by: axis, hollow shafts are much more effective in resisting torsional loads.
 There are many situations in which the twisting moments in shafts
cannot be determined by considerations of equilibrium alone. In such
shafts the geometrical constraints on twists need to be invoked to solve
54
the problems. Such problems are termed as statically indeterminate
ones. We follow the same three-step process that was invoked in the last
chapter:
o Use equilibrium analysis to write equations for the twisting
moments in the various parts of the shaft. There may not be enough
equations to determine the moments explicitly.
o Convert twisting moments to twists in each part.
o Write geometrical compatibility conditions to complete the
analysis.
 For composite shafts made up of a core of one material overlaid with a
sleeve of another material, the torsional stiffness is given by (
).
 The concept of shear flow q defined as the shear force per unit length of
the wall was introduced for the approximate analysis of thin tubular
shafts. It was shown that , where A is the area enclosed by the
mean line of the wall of the shaft.
 Limit load refers to the load under the assumption that the whole of the
structure has yielded and the shear stress everywhere is ηY. If the
applied torque is well below the limit torque so obtained, we can
presume that the twist of the shaft is not excessive.
 The strain energy in torsion of a shaft is given by .

55
For the beam to be in equilibrium, the net force or moment on the beam or on
ny p rt of it must be zero. Let us ‘cut’ the c ntilever be m shown in Fig. 4.1a
near the wall and draw the free-body diagrams (FBDs) of the two parts as shown
in Figs. 4.1b and c. Clearly, we need a force V and a moment M to balance the
applied load P on the part shown in Fig. 4.1c. These are the forces and moments
of internal reaction and these arise because of the deformations that are caused
by the applied load. There are, of course, equal and opposite reactions on the
stump of the beam built in the wall as shown in Fig. 4.1b.
We can, in general, resolve the internal resistance forces and moments in to their
components. Fig. 4.2 shows that resolution. The following are the nomenclature

4 Forces and moments in beams

4.1 Introduction
We have so far considered two types of structural elements: axially loaded truss
elements that resisted axial forces, tension or compression, and shaft-like
elements that resisted twisting moments. Beams that resist transverse loads by Fig.4.2 Forces and moments in a beam
bending are other common structural elements. We define a beam as a slender
element (i.e., an element whose cross-sectional dimensions are much smaller and the actions of the various components:

F x: the axial force that results in elongation (or compression, if


it is negative) of the member.
Fy and Fz: shear forces that result in shearing at the section. The shear
forces are conventionally assigned the symbol V.
Mz: the axial moment that is the torsion moment that causes
twisting of the member. This was the subject matter of the
last chapter.
(a) (b) (c)
Mx and My: transverse moments that are termed as the bending moments
Fig. 4.1 A cantilever beam and cause the member to bend. Moment Mx results in
than its length dimension) that essentially resists loads acting transverse to its bending the beam in the y-z plane while the moment Mz
length. Roof beams that support the roofs of buildings are beams. The horizontal results in bending the beam in the x-y plane.
deck of a road flyover supported on columns is a beam structure. The leaf
We had in Chapter 2 laid out the general strategy for solving the problems of the
springs of an automobile suspension transfer the weight of the car to the axle
mechanics of deformable bodies as consisting of three major steps:
(and further to wheels) through beam action. A wing of an airplane is a beam. So
is the cantilever beam shown in Fig. 4.1.
56
Consideration of static equilibrium and determination of loads in various
members,
Consideration of relations between loads and deformations, (first converting
loads to stresses, then transforming stresses to strain using the properties
of the material, and then converting strains to deformations), and
Considerations of the conditions of geometric compatibility.
In that chapter we had also studied the structures bearing axial loads. In Chapter
3 we considered slender members bearing twisting moments. These were termed
as shafts. We learnt to determine the variations of torsion moments (Mz in the
notation introduced above) as torsion moment diagrams (TMDs), and then to
calculate the resultant shear stresses and twisting angles of the shafts
In view of the importance of the role of beam members in engineering and other Fig. 4.4 Positive shear stress and bending moment
structures, we devote this chapter to beams, and that too, to only the first step of
the 3-step process outlined above. The loads which are of immediate relevance
to a beam are the shear forces Fy and Fz and the bending moments Mx and My Table 4.1 Sign convention43
which are caused by the transverse loads as shown in Fig. 4.3. It can be verified
easily that the shear force component Fz Sign of the Actual direction Assigned sign to the
and the bending moment component My outward normal of the force or shear force or bending
are caused only by the vertical load Pz, to the section moment moment
whereas the shear force component Fy and
the bending moment component Mz are Along the + ve Along the +ve
caused only by the transverse load Py.. coordinate coordinate + ve
This leads us to a very simple stratagem: direction direction
we calculate the bending moments and Along the − ve Along the − ve
shear forces for the vertical and horizontal coordinate coordinate + ve
loads separately (assuming that the other direction direction
loads are absent), and then patch up the
results. We shall, in the discussion that Along the + ve Along the − ve
Fig. 4.3 Loads on a beam − ve
follows, consider only the vertical load coordinate coordinate
system, which can easily be replicated for the horizontal loads. direction direction
Along the − ve Along the +ve
− ve
coordinate coordinate
4.2 Sign convention
direction direction
It is imperative from the point of view of consistency that we define the sign
convention for shear force and bending moment at a section. Fig. 4.4 shows a
beam which has been sectioned and the two resulting parts separated. The shear It can be verified that the shear forces and bending moments shown in Fig. 4.4
forces and the bending moments at the two sections are equal and opposite, as are all positive according to this convention. This convention 44 can be
shown. We adopt the convention given in Table 4.1to ensure that shear force (or
the bending moment) at either of the two sections have the same sign:
43
It is interesting to note that this sign convention results in a beam bending
concave upwards with a positive bending moment and convex upwards for a
negative bending moment.
57
summarized graphically as Fig. 4.5. Either
of these two can be used as an icon for the
sign convention employed.

4.3 Loads and supports


One common classification of beams is Fig. 4.5 Graphically representation
based on the kind of supports on which a of sign convention
beam rest. There are three idealized
support systems for beams as shown in Fig. 4.6. Support A is a termed as a
pinned support. Here the beam is attached to the support in such a manner that it
is not restrained from rotating about this point. Consequently, the reaction from
the support is just a force which can be resolved in to two force components RA,x
and RA,y. The support here cannot apply a moment on the beam. The support
shown at point B is termed as a roller support. Here the reaction from the support Fig. 4.6 Three type of beam supports and their conventional representations
can only be a vertical force RB,y. The roller does not restrain the beam from
moving horizontally, and hence there is no horizontal force applied by the We, in a similar fashion, idealize the loads that the beams carry. The left top
support. As in the case of pinned support, there is no reaction moment at a roller diagram in Fig. 4.7 show a simply-supported beam with a concentrated or a point
support as well. Beams with only pinned or roller supports are termed as simply- load, while the on the right shows one with a distributed load. A point load is
supported. specified as a force P with appropriate force units like N.

The support shown at C is called a built-in support. Here the beam is built into
the wall so that it is restrained from rotating about point C. The beam will have a Table 4.2 Idealized beam supports
zero slope at such a point. This is a consequence of the restraining moment My
that the support applies to the beam at that point. Considerations of equilibrium
Support type Freedom of motion Reactions present
will require that the reaction RC,y be equal to P, and the moment My applied on the
beam by the support will be PL, where L is the length of the beam. A beam can Built-in support No degree of freedom A moment as well as
have a built-in support at either one end or both ends. The beam shown at right horizontal and vertical
in Fig. 4.6 with only one end built-in (and no support at the other end) is termed reaction forces
as a cantilever beam.
The lower two diagrams in Fig. 4.6 show the conventional representations of the Pinned support Single degree of freedom Horizontal and vertical
three types of idealized supports. Please note that these are idealizations. Most - rotation. reaction forces
practical supports are more complicated, but many can be analysed quite
satisfactorily by replacing them with one or the other of these idealizations. The
dashed lines in these represent the (exaggerated) deflected shapes of the beams.
Note that the cantilevered beam has zero slope at the built-in end. Table 4.2 Roller support Two degrees of freedom – Only vertical reaction
summarizes these idealized supports. rotation and horizontal force
movement

44
Please note that many current books use a sign convention entirely different
from this. It is, therefore, advisable that the sign convention used be noted in
each problem that you solve. The sign convention can be denoted easily by a
graphical symbol similar to the ones shown in Fig. 4.5
58
A distributed load, on the other hand, is specified by a load density w as force per A beam could also be subject to an externally-applied bending moment as shown
unit length with appropriate units like N/m. What is shown on the beam on right in Fig. 4.8b. Here too, the dashed line represents the (exaggerated) deflected
shapes of the beam.

4.4 Determining shear forces and bending moments


Determination of shear forces and bending moments at any point in a beam is
rather a straight forward procedure. The procedure consists of taking a section of
the beam at that point to expose the shear force V and the bending moment M at
that point and drawing a free body diagram (FBD) of one portion of the beam.
We will first solve a couple of simple examples to illustrate the procedure and
then draw up a generalized scheme of doing things in more complicated
situations.

Fig. 4.7 Two idealized loadings of beam Example 4.1 A cantilever beam with a load at the free end
is idealized as a uniformly distributed load, wherein the load density w per unit Consider a cantilever beam as shown in Fig. 4.9. It is loaded at the free end.
length does not change along the length of the beam. Draw the shear
force diagram
However, in some situations, the load density could vary, as is shown in Fig. (SFD) and the
4.8a. Here the load is distributed, but the loading density varies along the length bending moment
of the beam. It is modelled as a linearly varying distributed load. If the load diagram (BMD).
density is w N/m at the end of the beam of length L, the load density at any x
from the left end is given simply by wx/L N/m. Solution:
We first calculate
the reactions at the
support. A simple
equilibrium analysis
shows that the end
reaction is a force P
upwards and a
clockwise moment
PL as shown. We
next calculate the
shear force V and
the bending moment
(a) (b) M at an arbitrary
Fig. 4.8 location x from the
support. For this Fig. 4.9 SFD and BMD for a cantilever beam with end load
(a) A simply-supported beam with non-uniformly distributed loading purpose we take a (a) A cantilever beam with support reactions
(b) A simply-supported beam with a concentrated applied bending moment (b) FBD of the section beyond x
section at that
(c) Shear force diagram (SFD)
location and draw (d) Bending moment diagram (BMD)
the FBD of the

59
right-hand portion45 of the beam as Fig. 4.9b. The shear force and the bending beam there and drawing the FBD. It should be obvious that there are two kinds
moment on this section are the external loads acting on this part of the beam and of FBDs depending on whether the value of x is less than L/2 or greater than L/2.
will be shown on the FBD. We do not know their directions. In such a case we Fig. 4.10b shows the FBD for x < L/2. We shall show the SF and BM as external
show them assuming they are positive. Since the exposed face on which they are loads on this FBD. We do not know their directions. As before, we show them
acting has an outward normal in the negative x-direction, the positive shear force assuming they are positive. Since the exposed face on which they are acting has
must be downward and the positive bending moment must be clockwise (as an outward normal in the positive x-direction, the positive shear force must be
shown). upward and the positive bending moment must be counter-clockwise (as shown).
A simple equilibrium analysis46 of this FBD gives: A simple equilibrium analysis of this FBD gives, for x < L/2:
∑ : or (a) ∑ : or (a)
∑ : ( ) , or ( ) (b) ∑ : , or (b)
Since the location of the section x is arbitrary, and changing the value of x does Thus, for x < L/2, SF has a constant value of – P/2 while the value of the BM
not change either the FBD or Eqs. (a) and (b), these equations give the shear increases linearly from 0 at x = 0 to a value of PL/4 at x = L/2.
force and the bending moment at all values of x between 0 and L. The resulting
values have been plotted as shear force and bending moment diagrams in Figs.
4.9c and 4.9d, respectively. These show the variations of the respective
quantities with x.

Example 4.2 A simply-supported beam with a concentrated load in the


middle
Consider a simply-supported beam with a concentrated load at the mid-point as
shown in Fig. 4.10a. Draw SFD and BMD for this beam.

Solution:
The first step in the solution of this problem is determining the reactions at the
support. Drawing an FBD of the entire beam (or using symmetry) we can easily
find the two vertical reactions to be P/2 each. To determine the shear force (SF)
and the bending moment (BM) at any location x (measured from the left support)
of the beam, we need to externalize the SF and BM by taking a section of the

45
We could have chosen either pat of the beam but have chosen to draw the FBD
for the right-hand portion of the beam because this results in an FBD with fewer
unknown forces simplifying the equations.
46
M,x in Eq. (b) denotes that moments have been taken about a point at x. This is
convenient because the moment contribution of the unknown force V about this Fig. 4.10 SFD and BMD for a simply-supported beam with a concentrated load
in the middle
point is zero.
It may further be noted that in the equilibrium equation, the sign to be used with (a) Loading diagram
the bending moment is the standard sign convention for moments: positive for (b) FBD when the section is taken in left half of the beam
clockwise and negative for counter-clockwise. Even though the bending moment (c) FBD when the section is taken in right half of the beam
shown is positive (negative on a face with negative outward normal), it is taken (d) Shear force diagram
as positive in the equilibrium equation. (e) Bending moment diagram
60
This formulation is valid only for x < L/2. For x > L/2, the generic FBD changes  Introduce the unknown shear force V and the bending moment M at the
and is as shown in Fig. 4.10c. Here the applied load P too figures in the FBD. cutting plane. These should be shown assuming they are positive
An equilibrium analysis of this FBD gives, for x > L/2:
∑ : or (c)
∑ : ( ) , or
( ) (d)
Thus, for x > L/2, SF has a constant value of + P/2 while the value of the BM
decreases linearly from PL/4 at x = L/2 to a value of zero at x = 0. The resulting
SFD and BMD47 are plotted in Figs. 4.10d and 4.10e, respectively.

4.5 General procedure for drawing shear force and bending


moment diagrams by method of sections
On the basis of the method adopted in the last two examples, the following
generalized procedure may be formulated:
 Draw an idealized loading diagram of the beam.
 Determine the reactions at all supports. If the reactions cannot be
determined, the beam is statically indeterminate and further progress
cannot be made without considering the deflections of the beam. These
will be introduced in Chapter 7.
 Determine the number of segments with distinct loading pattern to Fig. 4.11 SFD and BMD for a simply-supported beam loaded
cover the entire beam. In Example 4.2, we needed two different types symmetrically with two concentrated loads
of FBD: one without the concentrated load P, and the other with it. In
(a) Loading diagram
practice, this means that we segment the beam such that the end of a
(b) FBD when the section is taken in the left third of the beam
segment is at the location of a discontinuity in loading pattern. These (c) FBD when the section is taken in the middle third of the beam
could be the concentrated loads or moments, or where the type of (d) FBD when the section is taken in the right third of the beam
distributed loads changes. (e) Shear force diagram
(f) Bending moment diagram
 For each of the segments identified above, introduce a cutting plane (at a
location x from the left end) and draw an FBD of either part of the beam according to the sign convention introduced in Table 4.1 above. Thus,
(as convenient). In example 4.1 we had drawn the FBD of the right in Example 4.1 where we had drawn the FBD of the right-hand part of
portion (since it carried fewer loads), while in Example 4.2, we drew the the beam, the cut face has the outward normal in the negative x-direction
FBD of the left portions (since these permitted simpler expressions for and, therefore, the positive SF is downwards and the positive BM is
force moments). clockwise. This is exactly opposite of the case of Example 4.2 where the
outward normal is in the positive x-direction and, therefore, the positive
SF is upwards and the positive BM is counter-clockwise.

47
Since the BM is positive, the beam bends concave upwards, as is physically
apparent from the loading
61
 Determine the expressions for SF and BM by equilibrium or , a constant (d)
considerations, equating the sum of vertical forces and the moment
And from the equilibrium analysis of the FBD of Fig. 4.11c, we get for x > 2L/3:
(about a convenient point) to zero.
∑ : or (e)
 Plot the resulting expression to obtain the shear-force and bending
moment diagrams (SFD and BMD). These diagrams are conventionally ∑ : ( ) ( ) ,
drawn exactly beneath the loading diagram of the beam as in Figs. 4.9
and 4.10.
We give below a few more examples to illustrate the general procedure.

Example 4.3 A simply-supported beam with two concentrated loads


Consider a simply-supported beam of length L with two loads acting at L/3 and
2L/3 from the end as shown in Fig. 4.11. Draw the SFD and BMD of this beam.

Solution:
As usual, we begin with determining the reactions at the supports. Through a
simple equilibrium analysis (or by using the symmetry of the problem) the
reaction at either support is a force P upwards..
It is easy to see that we need to segment the beam in this case in three different
ways. One kind of FBD results when the section is taken for with no
loads acting on it (as in Fig. 4.11b). Another kind results when the section is
taken for a value of x between L/3 and 2L/3. In such an FBD one external load P
acts at x = L/3 (as in Fig. 4.11b). And the third kind results when the section is
taken for a value of x between 2L/3 and L. In such an FBD two external loads
act, one at x = L/3 and the other at x = 2L/3 (as in Fig. 4.11c).
We next show the unknown shear force V and the bending moment M at the
newly exposed sections assuming they are positive and using the sign convention
of Table 4.1. In each of the three FBDs, the positive SF is upwards and the
positive BM is counter-clockwise, since all outward normals are along the
positive x-axis. Fig. 4.12 SFD and BMD for a simply-supported beam with
uniformly distributed load
From the equilibrium analysis of the FBD of Fig. 4.11b, we get for x < L/3:
(a) Loading diagram
∑ : or (a) (b) Typical FBD for a segment of the beam
(c) Distributed load replaced by a statically equivalent load
∑ : , or (b) (d) Shear force diagram
Similarly, from the equilibrium analysis of the FBD of Fig. 4.11b, we get for L/3 (e) Bending moment diagram
>x > 2L/3:
∑ or ( ) (f)
: or (c)
∑ : ( ) ,

62
The resulting SFD and BMD48 have been plotted as Figs 4.11e and f. Solution:

Example 4.4 A simply-supported beam with uniformly distributed The total load on this beam is wL. The symmetry of the beam suggests49 that the
loading reaction at each support is wL/2. Since there is no discontinuity in the loading
with no concentrated load or moment, only one kind of FBD as drawn in Fig.
Consider a simply-supported beam with a uniformly distributed load of w N/m as 4.12b suffices for this problem. The presence of distributed load poses a problem
shown in Fig. 4.12. Draw the SFD and BMD for this beam. in determining the moment contribution of the loading. To this, we replace the
distributed load with its statically equivalent load50 as shown in Fig. 4.12c. It
consists of the total load wx acting at the centroid of the distributed load, which is
at the mid-point of the beam, i.e., at a distance of x/2 from the left end.
From the equilibrium analysis of the FBD of Fig. 4.12c, we get for all values of
x:
∑ :
or (a)
Thus, the shear force varies linearly, with a value of at x = 0, and a value
of at x = L. It is zero at x = L/2. The variations of shear force are
plotted as SFD in Fig. 4.12d.
∑ : ( ) ( )( ) ,
or (b)
This represents a parabola with M = 0 at x = 0 and L, and a maximum value of
at the mid-point x = L/2. These variations of bending moment along the
length of the beam are plotted as BMD in Fig. 4.12e.

Example 4.5 A simply-supported beam loading with a concentrated


moment at the middle
Consider a simply-supported beam of length L loaded in the middle with a
concentrated moment M0 as shown in Fig. 4.13a.Draw the shear force and
Fig. 4.13 SFD and BMD for a simply-supported beam with a concentrated bending moment diagrams for this beam.
moment
(a) Loading diagram
(b) Typical FBD for a section in first half of the beam
(c) Typical FBD for a section in second half of the beam
(d) Shear force diagram
(e) Bending moment diagram 49
This can also be obtained by drawing the FBD of the whole beam and
replacing the distributed load by the statically equivalent load system consisting
of the total load wL acting at the mid-point of the beam. A statically equivalent
load is the alternate load which produces the same net force and moment at any
point.
48 50
Here too, the BM is positive and the beam bends concave upwards, as is It can easily be verified that the loading shown in Figs. 4.12b and 4.12c are
physically apparent from the loading. statically equivalent as defined above.
63
Solution: The deflected (but exaggerated) shape of the beam is shown as the broken line in
Fig. 4.13a. As noted earlier, the beam deflects concave upwards where the BM is
We begin with determining the reactions at the supports. Through a simple
positive and convex upwards where it is negative.
equilibrium analysis the reaction at the left support is determined as a force equal
to M0/L, while t the right support it is − M0/L. The negative sign indicated that
the reaction at the right support is directed downwards, as shown in the figure 51. 4.6 The area method of drawing the SFDs and BMDs
We need to segment the beam in two different ways: one, when the section is The method of sections outlined in the last section is a simple method of
taken at (without including the concentrated moment M0 in it) acting on determining shear forces and bending moments, but it can get quite tedious in all
it (as in Fig. 4.13b), and the other when which included the concentrated but very simple cases. We give here an alternate method known as the area
moment M0 in the resulting FBD shown as Fig. 4.13c. method.
We next show the unknown shear force V and the bending moment M at the
newly exposed sections assuming they are positive and using the sign
convention. In either FBD, the positive SF is upwards and the positive BM is
counter-clockwise since the outward normal in either case is along the positive x-
axis.
From the equilibrium analysis of the FBD of Fig. 4.13c, we get for x < L/2:
∑ : or (a)
∑ : ( ) , or ( ) (b)
Thus, the shear force V is constant in this part, while the bending moment M
increases linearly from 0 to at the mid-point of the beam.
Similarly, from the equilibrium analysis of the FBD of Fig. 4.13c, we get for L/2
>x > L:
∑ : or (c)
∑ : ( ) ,
or ( ) (d)
The shear force V is constant in this part too, with the same value as in the first
part. The bending moment M shows an interesting behaviour: it jumps down
from to at x = L/2 nd then incre ses, g in line rly, from −
at the mid-point to 0 at the end-point of the beam52. The resulting SFD and
BMD are plotted as Figs. 4.13d and e.

51
It has been assumed here, for simplification, that at least one support is not Fig. 4.14 Elemental FBDs for three different kinds of loads
restraining the beam horizontally, so that there is no horizontal reaction force.
Anyway, the presence of a horizontal force will not affect the SFD and BMD.
52
Attention is drawn to and interesting feature of SFDs and BMDs as is apparent
to the five examples given so far: the line depicting shear force has a now seen to be discontinuous at the location of the concentrated moment. We
discontinuity wherever there is a concentrated force as a load, and the BMD is will elaborate on this later.
64
Consider the beam shown in Fig. 4.14, which carries a concentrated load P at We last consider an elemental section at a point within the segment DE which
location B and a concentrated moment M0 at location C as shown. A part of the carries a distributed load. The FBD of such a section is shown as Fig. 4.14e.
beam (segment DE) is subject to a distributed load. The density q(x) of this load
For x within the beam segment DE, the equations of equilibrium53 give:
measured in N/m may vary along the length.
∑ : or , or
Let us determine how the SF and BM change with x along the beam. Let us first
take a small element of length dx at a location x within the segment AB where no (g)
load acts on the beam. We draw an FBD of this element of the beam as shown in
Fig. 4.14b. Here we have shown shear force V and bending moment M at location ∑ : ( ) , or
x, and shear force V + dV and bending moment M + dM at location x + dx. It can
be verified that all directions have been shown such that SF and BM at either (h)
location are positive. The application of equilibrium to this FBD gives:
This suggests that for a distributed load, the rate of change of shear force at a
∑ : , or (a) location is equal to the (downward) load density at that point. And, similarly, the
∑ rate of change of bending moment is equal to the (negative of) shear force at that
: , or (b)
point. We can recast the equations (g) and (h) as:
This implies that the SF as well as BM does not change along a beam segments
which does not carry any load. ∫ (4.1)

We next consider an elemental section of the beam which includes point B, the ∫ (4.2)
location of the concentrated load. Fig. 4.14d shows the FBD of this element.
The application of equilibrium to this FBD gives: The area method gets its nomenclature from these two equations which say state
that the change in SF across a beam segment is equal to the area under the
∑ : , or (c) loading curve of that segment, and the change in BM across a beam segment is
∑ : ( ) , or for equal to the area under the shear force curve of that segment54.
infinitesimal dx, (d) The eight equations (a) – (h) above (along with Eqs. 4.1 and 4.2) provide a
These indicate that at the location of a concentrated load, the shear force jumps framework for an very convenient method for drawing the FBDs and BMDs of
up by a value equal to the load (acting downwards), but there is no change in the the beams directly without drawing FBDs of different segments..
value of the bending moment. It can be deduced quite easily that had the load P Let us first consider Eqs. (a), (c), (e), (g) and 4.1, all pertaining to changes in
been acting in the upwards direction, the SF would have jumped down by the shear force V. Eq. (a) states that V does not change across any element which
same value. does not carry any load. Eq. (c) suggests that at the location of a concentrated
Fig. 4.14d shows the FBD of an element of the beam which includes point C, the load, the shear force jumps up by a value equal to the load (acting downwards).
location of the concentrated moment M0. The application of equilibrium to this Eq. (e) states that V does not change across any element carrying a concentrated
FBD gives:
∑ : , or (e) 53
Two things should be noted here. First, the contribution of the distributed load
∑ : , or for infinitesimal dx, to the moment equation has been obtained by replacing the distributed load by its
(f) statically equivalent concentrated load qdx at the midpoint of the element, i.e., at
location dx/2. Second, this contribution is negligible since this moment is of
Thus, at the location of a concentrated moment, the bending moment jumps down second infinitesimal order, while the other terms are of first order only.
by a value equal to the applied moment, but there is no change in the value of the 54
The difference in signs in Eqs. 4.1 and 4.2 is due to the fact that the load
shear force across such an element. density q has been defined as positive downwards. If we had taken the usual sign
convention that upward forces are positive, we would have got a negative sign
before integral in both the equations.
65
moment, and Eq. (g) says that the rate of change in shear force is equal to the the (positive) applied moment. Eq. (h) states that the rate of change in bending
density of loading. moment is equal to the (negative of) shear force at that point.
In fact, Eq. 4.1 summarizes all this information quite succinctly if we assume a Eq. 4.2 contains all of the above points. The change in bending moment while
concentrated load as the integral of the load density at that point. The change in traversing a segment of the beam is equal to the area under the shear force curve
shear force across a segment of the beam is equal to area under the loading (but with a reversed sign) over that segment.
density curve.
The following procedure may be adopted for drawing the BMDs:
From these we can deduce the following procedure to draw the SFD:
2.1. First complete the shear force diagram of the beam.
1.1. First draw the loading diagram of the beam.
2.2. Start at a point slightly left of the left support where the bending
1.2. Calculate the reaction at the supports. moment is taken as zero.
1.3. Start drawing the SFD at a point slightly left of the left support where 2.3. Travel along the beam to the right, modifying the BM using the rules
the shear force is taken as zero. given below.
1.4. Travel along the beam to the right, modifying the SF using the rules 2.4. The slope of the BMD at any location is equal to (negative of) the SF
given below. value at that location
2.5. Change in BM between any two locations is equal to the (negative of)
1.5. Shear force does not change over the segment of the beam with no
the area under the SF curve.
external load.
2.6. Concentrated moments cause a jump in BMD at the location of the
1.6. The presence of a concentrated moment does not change the value of moment.
shear force. 2.7. Go up for every negative concentrated moment, down for every positive
concentrated moment.
1.7. A concentrated force causes a jump in SFD at the location of the force:
go up for every load downwards, down for every load upwards. 2.8. Proceed till you are slightly right of the right support where the BM
should again be zero again. This last is a check on calculations.
1.8. The slope of the SFD at any location is equal to the distributed load
density (load per unit length). Positive slopes for loads downward, and This is known as the area method of drawing the BMDs. It can be verified quite
negative slopes for loads acting upwards. easily that the BMDs Figs. 4.9d, 4.10e, 4.11f, 4.12e, and 4.13e follow from the
respective shear force diagrams using the above rules.
1.9. Change in SF between any two locations is equal to the area under the
distributed load curve. Positive changes for negative areas and negative The following example illustrates the simplicity of the method of area.
changes for positive areas.
Example 4.6 Beam with an overhang
1.10. Proceed till you are slightly right of the right support where the SF
should be zero again. This last is a check on calculations. Consider a 4 m long simply-supported beam with an overhang as shown in Fig.
4.15. The overhang portion AB carries a distributed load of density 20 kN.m.
This is known as the area method of drawing the shear force diagram. It can be There are two concentrated loads at locations C and E as shown. It also carries a
verified quite easily that the SFDs Figs. 4.9c, 4.10d, 4.11e, 4.12d, and 4.13d concentrated counter-clockwise moment of magnitude 20 kN.m at location D.
follow from the respective loading diagrams using the above rules. Draw the SFD and BMD for this beam.
We, in a similar fashion, can deduce the rules for drawing the BMDs from Eqs.
(b), (d), (f), (h) and 4.2, all pertaining to changes in bending moment M. Eq. (b) Solution:
states that M does not change across any element which does not carry any load We first determine the reactions R1 and R2 at the supports. With reference to the
or moment. Eq. (d) implies that at the location of a concentrated load too, there is loading diagram of Fig. 4.15, we can write the equilibrium equations. Equating
no change in the value of the bending moment. Eq. (f) states that at the location the vertical forces to zero, we get:
of a concentrated moment the bending moment jumps down by a value equal to
66
. / ( ) , We start at zero at the left end. We first encounter the uniformly distributed load.
Using rules 1.8 and 1.9 above, the shear force would increase at a constant the
or (a) rate of 20 kN/m, and after one meter it would have increased from 0 at A to 20
kN at B, as shown. A concentrated load (acting upwards) occurs at the location
We next take the moment of all the forces about the left support and equate it to B. Therefore, following rule 1.7, the value of shear force jumps down by 70 kN
zero to get55: (the value of R1) from 20 kN to − 50 kN. Since there is no lo d on segment BC,
( ) ( ) ( ) ( ) ( ) ( ) the value of SF through this segment remains constant (rule 1.5). At location C,
( ) , (b) the she r force jumps up from − 50 kN to − 10 kN (rule 1.7), and thereafter
remains constant at this value across segment CE, since the this segment does not
From Eq. (b) we get have any load except a concentrated moment at D, and rule 1.6 says that SF does
not change due to the presence of a concentrated moment. The downward load at
E makes the SF jump up through 40 kN from – 10 kN to + 30 kN (rule 1.7
Using this value in Eq. (a) we get again). The shear force remains constant at this value through segment EF, and
then jumps down to 0 at F because of the presence of the concentrated load R2 =
We are now ready to draw the shear force diagram. 30 kN. The final value of 0 completes the check (rule 1.10).
It should be obvious that this method of area is so much simpler than the method
of sections where we would have needed to draw five different kinds of FBDs
and, then, write ten equilibrium conditions for those five FBDs.
After completing the SFD, we can draw the BMD too. We start at zero for a
point slightly left of point A. The SF increases linearly from 0 at A to 20 kN at B.
By rule 2.4, then, the slope of the B D ch nges line rly from zero to − 20
kN·m/m. The linear change in slope implies that the curve is of second order in
x, that is, it is parabolic, as shown. The change in the value of the bending
moment in going from A to B is the area under the SFD in this segment (rule 2.5),
which is ( ) ( ) ( ) . This area is positive and,
therefore, the change in BM is – 10 kN.m, giving a value of BM at point B as –
10 kN.m. The shear force across segment BC is constant at – 50 kN, so the slope
of the BM curve there is constant at + 50 kN.m/m, and the value of the BM
through this section of length 1 m changes (linearly) through 50 kN.m, the area
under the SF curve in this segment (rule 2.5). The value of BM at point C is 40
kN.m.
From point C to D, the SF is – 10 kN, and therefore the BM curve slopes up at
this rate and the value of BM changes from 40 kN.m to 45 kN.m in 0.5 m. The
presence of a positive external concentrated moment of 20 kN.m at point D
makes the BM jump down to 25 kN.m at this point (rule 2.6), after which the
Fig. 4.15 Drawing SFD and BMD using area method curve continues its upward march at a rate of + 10 kN.m/m to acquire a value of
30 kN.m at point E. The SF in the last segment EF is constant at +30 kN.
Therefore, the BM there decreases linearly from a value of 30 kN.m to 0 in a
length of 1 m. The end value of 0 indicates that the calculations are in order.
55
Here again we take the moment of the distributed load by replacing it with the
statically equivalent load, which is the total force (20 kN)·(1 m) acting at the
centroid (of this load) which is located at 0.5 m from the left end of the beam.
67
Example 4.7 A built-in frame
Consider the frame ABCD shown in Fig. 4.16a which is built in at A. Draw the
SFDs and BMDs of the various segments of the frame.

Solution:
To begin the analysis, we need to consider the FBDs of the various sections of
the frame.
Fig. 4.16b shows the upper segment DC of the frame. There will be a force and
moment acting at the point C due to the restraining action of the segment CB.
Using the requirements of equilibrium we evaluate the vertical force as P
upwards and the moment as PL/2 clockwise.
Using the area method outlined above, we can easily obtain the SFD and BMD of
this segment as shown in Fig. 4.16b.
The FBD of the segment CB is shown in Fig. 4.16c. The load at point C of this
segment is equal and opposite to that at point C of the segment DC. Using
equilibrium of this segment we obtain the force at point B as P upwards, and the
moment there to be PL/2 clockwise. The force in this segment is axial and,
therefore, there is no shear force anywhere. The BMD has been obtained using
the rules 2.1, 2.6 and 2.8. The positive sign of the bending moment indicates that
this segment will bend concave upwards (viewing from right), which agrees with
what is expected intuitively.
We then move on to the cantilevered section AB, which carries a force P
downwards and a counter-clockwise moment PL/2 at point B as shown in Fig.
4.16c. We can obtain the reaction force and moment at end A by equilibrium
considerations. Due to the upward force P, the SFD jumps down to –P at the left
end A and remains so up till the other end where it jumps down to 0. The BMD
jumps down to − PL/2 at point A due to the presence of a concentrated moment
PL/2, and thereafter increases at a constant rate of P because of the presence of
the shear force –P throughout this length reaching a value of + PL/2 at the end B.
It jumps back to zero there because of the presence of a counter-clockwise
moment PL/2.

Example 4.8: An optimization problem


Consider a simply-supported beam of length L carrying a load 2P at the middle
as shown in Fig. 4.17a. It also carries a load P at each of the two ends. The two
supports are located a distance a inwards from the ends. Determine the value of Fig. 4.16 Drawing FBDs and BMDs of the various segments of a frame
the distance a required to minimize the maximum bending moment in the beam.

68
value of bending moment at the mid-point is [−Pa + P(L/2 − a)], or +P(L/2 −
2a). We can continue in the same fashion56 and complete the BMD.
The magnitude of the maximum negative bending moment is Pa, and the
magnitude of the maximum positive moment is P(L/2 − 2a). As the value of a
increases, the first value increases while the second decreases. A little reflection
should convince the reader that the minimum of the maximum value will occur
when these two magnitudes are equal:
( ), or , or .
Thus, we minimize the maximum value of the bending moment by locating the
supports at L/6 from either end of the beam.

Summary
Study of the behaviour of beams is very important because the beams are central
to the design of most structures. We, in this chapter have concentrated on
determining the variations in shear forces and bending moments along the length
of the beam.
Fig. 4.17 We first introduced the types of supports and the loads that the beams carry.
Three idealized supports were identified:
Solution:  The built-in support which allows no degree of freedom to the beam and
admits both components of the reaction force as well as a reaction
From the symmetry of the beam (or from the two equilibrium equations for the moment.
whole beam) we determine the reaction force at either support to be 2P.
 The pin support which allows one degree of freedom, the freedom to
We first draw the SFD using the area method. We start the SFD slightly left of rotate about that point. This support admits both components of the
the left end of the beam with a zero value. A downward force P is encountered at reaction force but does not admit a reaction moment
the left end which causes the SFD to jump up to a value of P (Fig. 4.17b). Since  A roller support which restraints the beam only from the vertical motion
there is no other loading till the left support, the SF remains constant at P till and, therefore, admits only the vertical reaction force component.
then. The reaction at the left support is 2P upwards which causes the SFD to
We next introduced the sign convention for the shear force and bending moment.
jump down by 2P to a value of –P at the left support. We continue in a similar
The sign is based on two parameters: the direction of the outward normal at the
manner and complete the SFD as shown.
section, and the direction of the force or the moment itself. If both of these are in
The BMD is also drawn in the same fashion (Fig. 4.17c). We start at zero. In the the positive coordinate directions, or both in the negative coordinate directions,
first segment of length a, the SF is constant at a value of +P, and therefore, the the sign of SF or BM is taken as positive. And if one of them is positive and the
BM in this section decreases at the rate of P per unit length. The change in value other negative, the sign is taken as negative. This is summarized in Table 4.1 and
till the left support is the areas under the SFD of this segment, which is Pa. as icons of Fig. 4.5.
Thus, the v lue of B t the left support is −Pa. In the second segment (of
The method of sections for drawing the SFDs and BMDs uses the following
length L/2 – a) the v lue of SF is −P and, therefore, the slope of bending moment
procedure:
curve here is +P. The change in value of BM is P(L/2 − a) from −Pa. Thus, the

56
Or use the symmetry of the beam.
69
 Draw an idealized loading diagram of the beam. The following process for drawing BMD was concluded:
 Determine the reactions at all supports.  First complete the shear force diagram of the beam.
 Determine the number of segments with distinct loading patterns to  Start at a point slightly left of the left support where the bending
cover the entire beam. moment is taken as zero.
 For each of the segments identified above, introduce a cutting plane (at a  The slope of the BMD at any location is equal to (negative of) the SF
location x from the left end) and draw an FBD of either part of the beam value at that location
(as convenient).  Change in BM between any two locations is equal to the (negative of)
the area under the SF curve.
 Introduce the unknown shear force V and the bending moment M at the
 Concentrated moments cause a jump in BMD at the location of the
cutting plane.
moment. Go up for every negative concentrated moment, down for
 Determine the expressions for SF and BM by equilibrium every positive concentrated moment.
considerations, equating the sum of vertical forces and the moment
(about a convenient point) to zero.
The other method of determining SFDs and BMDs is the method of areas is
based on the two equations:

, and .

From the integration of these we concluded that the change in SF across a beam
segment is equal to the area under the loading curve of that segment, and the
change in BM across a beam segment is equal to the area under the shear force
curve of that segment.
The following process for drawing SFD was concluded:
 First draw the loading diagram of the beam.
 Calculate the reaction at the supports.
 Start drawing the SFD at a point slightly left of the left support where
the shear force is taken as zero.
 Shear force does not change over the segment of the beam with no
external load.
 A concentrated force causes a jump in SFD at the location of the force:
go up for every load downwards, down for every load upwards.
 The slope of the SFD at any location is equal to the distributed load
density (per unit length). Positive slope for loads downwards and
negative slope for loads acting upwards.
 Change in SF between any two locations is equal to the area under the
distributed load curve. Positive changes for negative areas and negative
changes for positive areas.
70
consider a very simple case of a beam subject to pure bending57 as shown
in Fig. 5.1.
To obtain the relations between the bending moment Mb and the stresses
and strains we shall follow the strategy that we outlined in Chapter 3
while dealing with the case of torsion of a shaft. This was the reverse of
the generalized strategy outlined in Chapter 2. The reason for this again
is the same as in the case of torsion: unlike the situations involving axial
loading treated in Chapter 2, it is not possible to make the assumption of
uniform stresses in the case of bending of beams.
Before going
further, let us
determine the
nature of
deformations in
beams under
5 Stresses in beams pure bending and
the nature of
stresses that
result from such
deformations.
5.1 Introduction Fig. 5.2 shows a
beam with a grid
In problems involving bending of beams we may be interested in drawn on it. This
determining: beam is
 The stresses that result when a beam carries the specified loads. subjected to a
We may consequently determine the maximum load that a shaft positive bending
can carry without failure. moment, and
 The deflection of a beam when carrying the specified loads. bends as shown.
It is clear from Fig. 5.2 Bending stresses in beams
We have seen in the last chapter that almost all loads in beam result in the enlarged (a) Undeformed beam
variation of the resulting bending portion shown in (b) Beam after bending
moment and shear stress along its Fig. 5.2c that the (c) Enlarged portion showing that there is contraction
length. This inevitably introduces near the top and extension near the bottom
beam elements
complications in the determination of
the behaviour of the beam. To obtain a
Fig. 5.1 Beam in pure bending
foothold on the problem we first 57
The bending moment can be constant along the length of a beam only if the
shear force V is zero everywhere. This is possible only if there is no other load on
the beam except concentrated moments at the two ends.

71
near the top surface of the beam are under compression, while those near assumptions are valid in this case of pure bending of a beam of uniform
the bottom are in elongation. As a consequence, there are compressive and symmetrical (about a vertical axis) section:
stresses near the top and tensile stresses near the bottom. It should be
 The beam bends in the shape of a circular arc. This follows from
clear that since there in no net tension (or compression) in the beam, the
the argument that a uniform bending moment acting along the
forces due these stresses must total out to zero. The bending moment at
length of the beam must result in a uniform curvature.
the section is the integrated moment of the elemental forces due to these
stresses.  Plane cross-sections of the beam remain plane after the bending.
Thus, plane sections 1-2 and 3-4 of the undeformed beam of Fig.
The strategy for this problem then consists of: 5.4 remain plane in the deformed shape as well.
 First using symmetry considerations to establish the nature of  Initially parallel sections (sections 1-2 and 3-4 of the undeformed
deformation in the beam under pure bending. beam of Fig. 5.4) must deform so that they have a common point
 Using the geometric considerations to determining the tensile of intersection as shown. This common point of intersection is
strain across the beam section as a function of the vertical the centre of curvature of the deformed beam.
coordinate. This is the macro to micro conversion discussed in Calculation of strain from the geometry of deformation
Section 2.1.
We now consider the geometry of deformation further to obtain an
 We next use the material properties to convert these strains to
expression for strain.
stresses. This is a micro to micro transformation.
 In the last stage we convert the stresses into bending moment, a It was argued above that the beam elements near the top are under
micro to macro conversion. We use the fact that the total compression while those near the bottom are under tension. It stands to
bending moment is the given Mb, and the total tensile force at the logic, then, that there must be a plane in the beam (near the middle) which
section is zero. This completes the solution to the problem. is neither under tension nor
compression. Let AB
represent such a plane for
5.2 Relating curvature of the beam to the bending the beam segment 1-2-3-4 of
moment Fig. 5.4. This plane with its
extension to the rest of the
Consider a beam with a cross- beam is termed as the
section that is laterally symmetric neutral plane of the beam.
(i.e., it has symmetry about a It should be understood that
vertical axis) subjected to a pure we do not, as yet, know
bending moment Mb as shown in where this is located, except
Fig. 5.3. Under the action of this for the fact it must be
positive bending moment, the somewhere between the top
beam bends and acquires a and the bottom surfaces of
curvature. The coordinate axes the beam.
used here are shown in the figure.
Fig. 5.3 A symmetrical beam Let us consider a plane CD a
By using symmetry arguments we distance y up from the
can show that the following neutral plane AB. The
undeformed lengths of Fig. 5.4 Geometry of deformation of a beam
72
planes (along the x-axis) AB and CD are equal to begin with. Since the like that shown in Fig. 5.5. The trace zz of the neutral plane is termed as
length of the plane AB is unchanged (this being the neutral plane), the the neutral axis. Note here we have shown the neutral axis to be nearer
undeformed length of the plane AB or CD is seen as from Fig. 5.4b, the bottom plane of the beam. This results in the top plane of the beam to
where ρ is the radius of curvature58 of the neutral plane of the beam. The be farther away from the neutral axis (NA) than the bottom plane is.
deformed length of the plane CD is ( ) . Thus, the contraction in Consequently, the maximum compressive stress is larger than the
the length of this plane located a distance y up from the neutral plane is maximum tensile stress in the beam.
ydθ. The strain at this location is, thus, given by Since there are no shear strain components, there will be no shear
stresses. Further, there are no other longitudinal stresses, ζyy or ζzz, in this
case of pure bending60.
(5.1)
Converting stress into
It can be seen that in this simple case of pure bending (i.e., with no loading
loading other than the bending moment which is constant along the length
of the beam) there are no shear strain components. However, there will Consider an elemental area
be longitudinal strains εyy and εzz due to the presence of ζxx through dA in the right face of the
Poisson ratio. beam as shown in Fig. 5.6.
The force acting on this
Converting strain to stress elemental area due to the
Conversion of strain to stress is simple. The tensile stress component can bending stress is ζdA acting
be obtained by multiplying the strain with the elastic modulus E: along the normal to the
Fig. 5.6 Calculating loading at a section
section as shown. Note that
( ) ( ) (5.2) we have shown this force outwards from the area, assuming ζ to be
tensile (i.e., positive). The algebra below will automatically take care of
Note that we know neither the sign if the stress is compressive. The neutral plane of the beam is
the value of y (because the shown as grey61. The z-axis is the neutral axis of the section under
location of the neutral plane consideration.
has not yet been established)
nor the value of ρ, the radius The total axial force on the beam can be found by integration as
of curvature59. But we have ∫ . Substituting the value of ζ using Eq. 5.2, we get
two conditions that we shall
shortly use.
The distribution of the ∫ ∫ ∫
stresses across the section is Fig. 5.5 Stress distribution on a section of the
beam
where y is the vertical distance measured from the neutral axis.
58
The radius of curvature ρ is ∞ for a flat beam. As the beam bends, the value of But there is no axial force Fx in this case of pure bending. Therefore,
ρ decreases. We introduce the term curvature for 1/ρ. Curvature is usually
denoted by κ (read kappa). The value of κ increases as the beam bends. It is 0 for
a flat beam (corresponding to ρ = ∞). 60
59
But we have two conditions that we have not used so far. The total axial force We can now determine the lateral shear strains εyy and εzz as .
61
on the section is zero, and the total moment acting on the section is Mb. Note again that we do not as yet know the location of the neutral plane.
73
∫ (5.3) The bending formulae (Eqs. 5.4 and 5.5) are sometimes summarized63 as:

What does this imply? It simply defines the location of the neutral axis (5.6)
from which the distance y is measured. Thus, the neutral axis of a beam is It may be noted that the equation above have been obtained for the case of
located such that the first moment of area about that axis is zero. pure bending, i.e., for the case of constant bending moment in the absence
Recall that the centroid of an area is defined as the point about which the of any other load. The results for the more general case are very difficult
first moment of area is zero. Therefore, the neutral axis of a beam passes to obtain. But wherever these have been obtained, it is seen that for
through the centroid of the cross-section62. slender members, the resulting stress distribution is quite close to those
obtained above. It is for this reason that we use the results obtained here
We next calculate the moment (about the z-axis) of these elastic forces
in the general case as well.
and equate it to Mb, the bending moment on the beam:
Since the values of and are zero, the lateral strains and
are given by
∫ ∫ ( ) ∫
(5.7)

The integral ∫ is recognized as the second moment of area Izz Thus, the normal strains in the plane of the cross-section are proportional
about the z-axis, which, in this case, is the centroidal axis. This is a to the axial strains, but of opposite sign. Further, since εxx varies with y,
geometric quantity. Once the shape of a cross-section is known, the value the vertical distance measured from the neutral axis, the strains εyy and εzz
of Izz can be calculated. See Appendix A to learn some important change across the cross-section. This leads to the deformation64 of the
properties of the second moment of area and the procedure for its section as shown in Fig. 5.7.
calculation. Notice from Eq. 5.6 that the curvature of the beam 1/ρ is given by
We can, therefore, write an expression for 1/ρ, the reciprocal of the radius
of curvature (which is termed as the curvature κ = 1/ρ) of the beam as
(5.4)

The product is termed as the bending rigidity of the beam. The more
the value of the bending rigidity, the less is the curvature κ of the beam 63
(for a given value of the bending moment), and more is the radius of Notice the similarity of this with a similar equation that we can obtain for the
curvature ρ. case of torsion of a shaft from Eqs. 3.2 and 3.3:

We can combine Eqs. 5.2 and 5.4 to obtain .

(5.5) Note that in this equation for shafts, the z-axis is the longitudinal axis, unlike in
the present case where the z-axis is a transverse axis while the longitudinal axis is
These stresses are termed as the bending stresses. represented as the x-axis.
64
Note, in particular, that the neutral axis (which is the trace of the neutral
surface in the section) is now curved. This transverse curvature of the cross-
62
We had chosen the location of the neutral plane in Fig. 5.6 closer to the bottom section of the beam is termed as anticlastic curvature. The neutral plane now has
of the beam anticipating this result. double curvature, one in the x-y plane, and the other in the y-z plane.
74
see straight away that there will be an upward reaction of 200 kN at the
support. From the moment balance we obtain the moment at support to
be 300 N.m, counter-clockwise as shown.
We draw the SFD by the method of areas. We start at 0, jump down 200
N at the left support (load being upwards), continue at ‒200 N till the first
load is encountered. The downwards load makes the SF jump up to ‒100
Fig. 5.7 Deformation of the shape of a rectangular beam N at 1 m. The SF jumps up again at the second load.
(a) The undeformed rectangular section (b) variation of longitudinal stress
ζxx with distance y from the neutral axis (c) variation of longitudinal strain
The BMD is drawn in the same fashion. We start at 0, jump down to ‒300
εxx across y (d) variation of lateral strains εyy and εzz with y (e) the deformed N.m at the support because of the presence of the positive reaction
shape of the cross-section (deformation exaggerated). Note that the neutral moment. Through the first one meter of the beam, the BM line has a
axis which is the trace of the neutral surface in the section) is curved. positive slope of 200 N.m/m (equal to the negative of the constant shear
force). The value of BM at 1 m, the midpoint of the beam, is ‒100 N.m.
The change in value of BM between x = 0 m to x = 1 m is equal to the
area under the SFD over this length. From x = 1 m to x = 2 m, the slope
The quantity EIzz can be interpreted as the stiffness of the beam. The of the B D is +100 N.m/m, the SF being const nt t ‒100 N over this
more its value, the less is the curvature of the beam, i.e., less is the length.
bending.
We, thus, see that the maximum bending moment occurs at the root of the
We give below a few examples of calculation of bending stresses. c ntilever where its v lue is ‒300 N.m. This, then, would be the loc tion
of the maximum stresses. Since the BM is negative, the beam would
Example 5.1 Maximum stresses in a cantilever beam bend convex up. The curvature 1/ρ given by Eq. 5.4 will be negative.
Consider a 2 m long cantilever MS beam of section 10 mm × 20 mm The value of Iyy needed in this equation has been obtained in Appendix B
loaded as shown in Fig. 5.8. Determine the location and magnitude of the (Eq. B.6) as bh3/12, where b is the width of the section (10 mm in this
maximum bending case), and h is the height of the section (30 mm). Thus, the value of Iyy is
stress. (10×10‒3 m) × (30×10‒3 m)3/12, which is 2.25×10‒8 m4.
Solution: The maximum tensile stress occurs at the top of the beam (y = 0.010 m)
We need to obtain the at the root (x = 0), and is given by Eq. 5.5 as
bending moment ( ) ( )
= 200 MPa
distribution first. For
this, we follow the The value of the maximum compressive stress will be the same, except
procedure introduced in that it will be at the bottom of the root of the beam. The beam is loaded
the last chapter, and almost to its ultimate strength.
first calculate the
reactions at support. Note that given a bending moment, the stresses are independent of the
From the balance of material of the beam. A beam with lesser elastic modulus will bend more
Fig. 5.8 A cantilever beam (giving larger values of curvature 1/ρ and strains) but will produce same
vertical forces we can
stresses.

75
It is interesting to compare these results with the case when the beam Example 5.2 Bending stresses in an angle-section beam
section has the longer side horizontal. Notice that the length of the
Consider a simply supported angle-section steel beam loaded as shown in
horizontal side now is a factor of 3 larger, the vertical side a factor of 3
Fig. 5.9. Its section is the same as was used in Example B.5. Determine
smaller, the value on Izz as now two factors in 3 smaller. The maximum
the maximum tensile and compressive stresses in the beam.
bending stress that varies like y/Izz is 3 times larger, and the radius of
curvature that varies like Izz is two factors in 3 smaller, i.e., the curvature Solution:
(1/ρ) is 1/9th of when the beam section is vertical. These results are
summarized in Table 5.1. To draw the SFD and BMD, we calculate the reactions at the supports.
We determine the reaction at the right support by writing the balance of
Table 5.1 Comparison of two different orientations of beam sections moments about the left support. This reaction is found to be 50 N. Then,
by vertical force balance, the reaction at left support is found to be 250 N
Section orientation Factor as shown.

b 10 mm 30 mm 3
The SFD of the beam can be drawn easily by the method of areas. The
SFD jumps up through 100 N at the left-end because of the presence of a
h 30 mm 10 mm 1/3 downward force, then jumps down through 250 N to ‒150 N due to the
‒8 4 ‒8 4
Izz 2.25×10 m 0.25×10 m 1/32 concentrated reaction at 1 m, jumps up through 200 N to +50 N because
ζxx 200 MPa 600 MPa 3 of the downward load at 2 m length, and then jumps down to zero because
of the 50 N upward reaction at the right support.
Radius of curvature ρ (at the 15 m 1.67 m 1/32
point of maximum bending The BMD of the beam starts at zero and has a slope of 100 N.m/m
moment) through the first 1 m length. The value at 1 m is equal to the (negative of)
area under the SFD over this length, which is 100 N.m. We complete the
It is clear that when more material of the beam is away from the BMD in the same fashion following the rules given in Sec. 4.6.
centroidal axes (on either side of it), more is the value of the second
moment of area, and the stiffer is the beam. There are two extrema in the BMD: a negative 100 N.m value at x = 1 m,
and a positive 50 N.m value at x = 1 m.
The centroid of the cross-section of the beam can be determined by taking
the first moment of the area about an axis passing through the bottom of
the section as was done in Example A.5 in Appendix A where the vertical
value of it was found to be 16.1 mm.
The second moment of the area about the centroidal axis was also
determined there. The strategy was to divide the area into two rectangles,
and then for each rectangle first determining the value of I about its own
centroidal axis, and then using the parallel-axes theorem to transfer it to
the neutral axis, i.e., the centroidal axis of the full section. The value was
determined as Izz = 2.99×10‒7 m4.
We can next determine the stresses. At location x = 1 m where the
Fig. 5.9 A simply supported angle-section beam
m ximum neg tive bending moment of ‒100 N.m acts, the beam bends
convex up so that the maximum tensile stress acts at the top plane which
76
has a value of y = +33.9 mm measured from the neutral axis. Similarly, Example 5.3 Comparison of rectangular, T-section and I-section
the maximum compressive stress act at the bottom plane which has a beams
value of y = +16.1 mm measured from the neutral axis.
Consider three cross-
We use Eq. 5.5 to determine the stresses. These are arranged in the Table sections of beams shown
5.2 given below. in Fig. 5.10. The three
At location x = 1 m where the bending moment is ‒100 N.m, the sections have equal
maximum tensile stress (at y =0.0339 m) is given by: areas and, hence equal
weights per unit length
( )( ) of the beams. Compare
the maximum tensile
The maximum compressive stress occurs at the bottom plane at y = and compressive (a) (b) (c)
‒0.0161 m: stresses in the three Fig. 5.10 Three beam sections of equal areas
( )( ) beams for a bending
moment Mb.
We can similarly determine the stresses at the location x = 2 m where the Solution:
bending moment is +50 N.m. These are shown in the table above.
The calculations have been organized in Table 5.3. We first calculate the
location of the neutral axes by calculating the y-coordinates of the
Table 5.2 Stresses at the locations of maximum positive and negative bending centroids of the three sections. This is determined by dividing the areas
moments
into the requisite number of rectangles and then using the formula:
x-location 1m 2m
̅ ∑ ̅ ∑ .
Bending moment, Mb ‒100 N.m +50 N.m Next, we calculate the second moment of inertia. For this, again, we
divide the cross-section in a number of rectangles, calculating the value of
Stress distribution
I for each rectangle about its own centroidal axis, and the transferring it to
the neutral axis using the parallel-axes theorem. Thus,
∑. (̅ ) /.

Maximum tensile stress At top plane (y = 0.0339 At bottom plane (y =


m) ‒0.0161 m)
11.3 MPa 2.7 MPa
Maximum compressive At bottom plane (y = At top plane (y = 0.0339
stress ‒0.0161 m) m)
‒5.4 MPa ‒5.6 MPa

77
The stresses are then evaluated from Eq. 5.5. These stresses vary linearly 5.3 Composite beams
from zero at the neutral axis. The stresses are compressive above the
neutral axis and tensile below it. In each case, the maximum compressive Consider the case of pure bending of a composite beam made up by
stress occurs in the top plane, while the maximum tensile stress occurs at bonding two materials of different elastic properties, in particular, of
different elastic moduli. Let us consider a section with symmetry about y-
Table 5.3 Comparative stresses in beams of equal weight axis as shown in Fig. 5.11. Let this beam be subjected to pure bending
with a positive bending moment Mb. Let us determine the moment
curvature relations for this beam following the procedure used in Sec. 5.2.
Section
Using the symmetry of the situation, we can again argue that plain
sections will remain plain and that the beam will bend in a circular arc.
Location of NA from the 30 mm 38 mm 30 mm
base
The geometry of deformation will be the same as depicted in Fig. 5.4, and
we would get an expression for the linear strain similar to Eq. 5.1
Izz 5.40×10‒7 m4 5.21×10‒7 m4 6.77×10‒7 m4
Stress distribution (5.8)

where ρ is the curvature of the neutral plane, i.e., a plane that does not
have any strain, and y is the distance measured from the neutral plane.

y for max tensile stress ‒30 mm ‒38 mm ‒30 mm Note that, as before, for a given curvature of the beam, the strain varies
linearly the distance from the neutral plane and does not depend on the
y for max comp. stress +30 mm +22 mm +30 mm
material of the beam.
4 4
max tensile stress 5.56×10 Mb Pa 7.29×10 Mb Pa 4.43×104Mb Pa
However, when we continue to the next step of converting to stress, the
Max comp. stress 5.56×104Mb Pa 4.22×104Mb Pa 4.43×104Mb Pa material properties come in and the stress is given by
the bottom most plane. ( ) ( ) (5.9)
It should be noted that in the case of a T-beam, the maximum
compressive stress is much smaller than the maximum tensile stress since where Ei stands for the elastic modulus of the specific material at that
the neutral axis is closer to the compressive side65. location. Note, in particular, that while the strain at the junction of two
materials is continuous, the stress shows a discontinuity (see Fig. 5.12).
Another thing to note is the reduction in the maximum stress in the I- Note that the strain, as well as the stress, is zero at the neutral plane. The
section. This is a result of a larger value of Izz, which in turn results from strain increases linearly on either side of the neutral axis, compressive
more of the section area (compared to the rectangular section) being away above it and tensile below it66. The stress distribution is discontinuous.
from the central axis. Since an are ’s contribution in Izz is weighted by y2, The stress jumps at the location where the material changes,. The strain,
the areas far away from the centroidal axis contribute more, making the being a purely geometrical quantity in a beam, shows no such jump.
beam stiffer. Consider two points close together on either side of the interface of the

65
But the total compressive force is exactly equal to the total tensile force on the
section, there being no net axial force. There is more area of the section on the
66
compressive side than on the tensile side. It is for this reason that the neutral axis The bending moment, being positive, introduces positive curvature (concave
has shifted upwards causing a reduction of tensile stresses. upwards) resulting in the strains as shown.
78

Fig. 5.11 Beam with composite section


two materials. Since the strains are the same, the changed value of E will or, ∫ ∫ ̅ ,
result in different values of
stress on the two sides. or, ̅∫ ∫ .
Hence the jump67.
We now consider the integral over the whole area as a sum of a number of
The location of neutral
integrals, each over an area of uniform material.
plane and the value of ρ are
as yet undetermined. To Thus, ∫ ∑ ∫ ∑ .
evaluate these we, as
(a) (b) (c)
before, use the ∑ ∑ ̅ , where ̅
Fig. 5.12 (a) beam section (b) strain distribution Similarly, ∫ ∫
considerations of (c) stress distribution
equilibrium. The total represents the location of the centroid of the ith area in coordinate
system. From these we get
force on the section is evaluated as by integration as ∫ . ∑ ̅̅̅
Substituting the value of ζ using Eq. 5.9, we get ̅ (5.10)

∫ ∫ , This gives the location of the neutral axis within the section68.
To obtain the moment curvature relation, we write the equilibrium
where Ei is the elastic modulus of the
condition for the moments:
material specific to the area element dA
(and y is the distance of the area element
measured from the neutral plane). ∫ ∫ ( ) ∫
This axial force should vanish, and
therefore, the neutral plane should be The last integral is again interpreted as the sum of integrals over areas of
located such that ∫ . different (homogeneous) materials to obtain ∑ . Then,
the curvature κ = 1/ρ can be seen as
To interpret this equation, consider Fig. Fig. 5.13
5.13. The distance y is measured from the ∑
(5.11)
neutral axis. Let distance y’ be measured from an arbitrary axis from
which the neutral axis is located at a distance of ̅. Then ̅, and where Izz,i is the second moment of the area Ai about the neutral axis of
the equation of equilibrium becomes the section.
( ̅) Once the curvature is known, we can calculate the strain as
∫ ,

68
It may be noted that in Eq. 5.10 the areas of different materials are weighted by
the elastic moduli of the materials. These happens because this relation is
67
Two things may be noted in Fig. 5.12. First, near the interface the value of obtained from the balance of force equation where forces are determined from
stress in the upper material is less than that in the lower material. This indicates the stresses which are E times the strain. The strains, by themselves, are
that the value of E for the upper material is less than that for the lower material. geometric quantities.
Second, note that all the straight lines depicting the stresses in the various A consequence of this is that the neutral plane shifts towards the area with the
materials originate from zero value at the neutral axis. Why? larger modulus of elasticity.
79
(5.12) moment of the lower area about its own axis + area×

(distance between the neutral axis and the centroid of
and (5.13) area 1)2

= (0.08 m)×(0.02 m)3/12 + (0.08 m)×(0.02 m)×(0.04 m –
where E is the elastic modulus of the 0.01 m) 2

relevant material.
= 1.49×10‒6 m4.
Example 5.4 Stresses in a sandwich Similarly, moment of the upper area about its own axis + area×
beam
(distance between neutral axis and the centroid of area 3)2
Consider a beam of sandwich construction
= (0.08 m)×(0.02 m)3/12 + (0.08 m)×(0.02 m)×(0.04 m –
in which two steel plates are separated by 2
0.07 m)
a block of lightweight rigid foam material
with a negligibly small elastic modulus Fig. 5.14 A sandwich beam = 1.49×10‒6 m4.
compared to that of steel (Fig. 5.14). Find
the stress distribution for a bending moment of 100 N.m. Then, ( ) ( ) ( ) ( )
‒4 ‒1
Solution: = 1.68×10 m
It is easy to see from the symmetry of the section that the neutral plane is The value of the maximum compressive stress is then determined from
situated at the middle. We can also use Eq. 5.10 to obtain the location of Eq. 5.9 with E = 200 GPa, and y = 0.04 m,
the centroid. ( ) ( ) ( )
∑ ̅̅̅ ̅̅̅ ̅̅̅ ̅̅̅ ‒4 ‒1
̅ , where A1 refers to the (1.68×10 m )

area of the lower steel plate with E1 = 200 GPa and ̅ = 10 mm, A2 = ‒1.34 ,
refers to the area of foamy material with a negligibly small vale of E, and The minus sign indicating that it is a compressive stress. We can
A3 to the area of the upper plate with E3 = 200 GPa and ̅ . similarly determine the maximum
Using these values, we get tensile stress (at y = ‒ 0.04 m) s
+1.34 MPa.
̅
( ) ( ) ( ) ( ) ( ) ( ) The rigid foam serves the purpose of
( ) ( ) ( ) ( ) shifting the two steel areas away from
0.40 m the centroid increasing the Fig. 5.15 The stress distribution in
contribution of each area to Izz, sandwich beam
The curvature is given by Eq. 5.11 thereby increasing the stiffness of the
beam. If the foam was not separating the two steel plates, the total height
, E2 being negligibly small.
∑ of the section would have been 40 mm and the value of Izz would have
Here refer to the second moment of the three areas about the neutral been (0.04 m)×(0.04 m)3/12 = 0.213×10‒6 m4, much less than that of the
composite beam with the resulting decrease in beam stiffness, increase in
axis (passing through the middle of the section as shown above).
curvature and increase in the maximum value of stresses. It is always

80
beneficial to keep the section areas far 5×π×(0.020 m)2/4, and for concrete: Ec = 20 GPa, ̅ m, and Ac
away from the axis. It was for the same = (0.450 m)×(0.200 m). Using these values in Eq. 5.10, we get
reason that I-beam of Example 5.3 was
∑ ̅̅̅
better than the rectangular section. ̅ m.

Example 5.5 Reinforced concrete Note that as expected, the stiffer material in the lower half of the section
beam brings the neutral axis down (from 225 mm for the concrete alone).
Fig. 5.16 Reinforced
Reinforced concrete construction is a very concrete beam section
ingenious use of two materials with The stiffness of the composite beam is obtained from ∑ (refer Eq.
complementary properties. Common concrete is a brittle material which 5.11). The values of Izz’s re obt ined by first ev lu ting the I’s bout the
has good strength in compression (about centroidal axis of the concerned area and, then, transferring it to the
20 MPa). Structural steel on the other hand neutral axis using the parallel-axes theorem.
has good strength a tension (about 200
( )( )
MPa). All beams have compression on ( )( )(
one side and tension on the other. This
)
suggests that we combine the two
materials to produce beams in which steel m4
is loaded in tension while concrete is
loaded in compression. Fig. 5.17 Stress distribution ( ) 64 5 7
in a reinforced concrete beam
Fig. 5.16 shows the section of such a
reinforced concrete beam. What is the maximum bending moment this ( ) ( )
beam can transmit? 4 ( ) 5
Solution:
Let us take the value of Ec, the elastic modulus of concrete as 20 GPa, m4
while that for steel, Es as 200 GPa.
Therefore, the stiffness of the beam is ∑ =
We will use the following strategy to solve this problem:
( )( ) ( )( ) =
1. Locate the neutral axis using Eq. 5.10. 3.86×107 N.m2,
2. Determine the stiffness of the beam using Eq. 5.11.
3. Assuming a bending moment Mb, calculate using Eq. 5.13 the And the curvature is
maximum compressive stress in concrete and maximum tensile
stress in steel. The maximum compressive load will occur in the top plane of the beam
4. Equate these stresses to the specified limiting stresses. The which is (450 ‒ 205) mm w y from the neutr l pl ne. From Eq. 5.13,
minimum of the two maximum moments will be the answer to the ( )( )
problem.
We first determine the location of the neutral axis for the section using Pa, and
Eq. 5.10. Here, for steel: Es = 200 GPa, ̅ m, and As =
81
( )( ) and therefore, it must be ‒Pd
at point B. The BM over
portion AB has, thus, a
Pa constant value of ‒Pd. This
would require a clamping
For a maximum permissible compressive stress of 20 MPa in concrete,
moment of +Pd. The resultant
the maximum permissible moment is given by , or
BMD must be as shown in
Mb = 157.6 kN.m. For a maximum tensile stress of 200 MPa in steel, the
Fig. 5.19. It can be verified
maximum permissible moment is given by , or Mb
from the loading diagram that
= 249 kN.m. The minimum of these two, then, is the permissible
this indeed is in agreement
maximum moment that this reinforced concrete beam can sustain. Fig. 5.19
with the other loads on the
Example 5.6 Force required to bend a strip strip.

Consider a strip of length l, width Now we are ready to evaluate the load P in terms of the given parameters.
b and height h clamped to a block We know the radius of curvature of the strip over the length AB s ‒R.
with a circular arc surface of The negative sign arises from the fact that the beam is bending convex
radius R as shown in Fig. 5.18. upwards. Using Eq. 5.4, we get , or .
( )
On application of a force P to the
free end, a length c of the strip
(AB) comes in contact with the 5.4 Stresses in beams carrying shear forces
block. Find the force P.
We have so far considered the
Solution: case of a beam in pure bending
Fig. 5.18
only. The only stresses that
We are given that the length AB of exist in such a beam are tensile
the strip has a constant radius of curvature. This implies that the bending stresses, termed as the bending
moment over this length is constant, which, in turn, implies that the shear stresses because they arise from
stress on this part of the strip vanishes and that there is no other loading bending of the beam. In a more Fig. 5.20 Relative sliding in a stack of
on this part. Let us consider the shape of the SFD of this strip. The SF planks acting as a beam
general case, there are other
over the length AB is constant at zero. There is no load over the portion stresses as well, notably shear stresses. To demonstrate the plausibility of
BC. This implies that SF over this segment too is constant. This constant the presence of shear stresses, consider the simply-supported stack of
SF will jump up through P at the end C because of the presence of a planks69 shown in Fig. 5.20. The planks will exhibit relative sliding as
concentrated load P (downwards) at that point.. But the shear force must shown. If these planks were glued together, they would tend to slide and
be zero after end C. This is possible only if the SF is constant at +P over the glue will be in shear trying to hold the planks back from sliding. If it
the segment BC. Since there no SF over AB, and the SF over BC must be were a solid beam, this would cause shear stresses to be present which
+P, there must be a concentrated reaction at point B where the strip loses tend to hold the different layers together.
contact with the radius block. The resultant SFD must be as shown in
Fig. 5.19.
69
This stack of five planks will have five times the moment carrying capacity of
The BM at the free end C must be zero. The presence of a negative SF
a single plank. But if we had used a beam of the combined thickness, the
over the length BC implies that the BM is increasing over this segment, moment carrying capacity would have been 25 times! Can you show this?
82
Consider next the simply- development of shear stresses on the bottom
supported beam loaded in surface of the slice acting towards right as
the middle as shown in shown.
Fig. 5.21. The SFD and
BMD of the beam have This shear is ηyx with a negative sign (using
also been shown in the the sign convention of stresses introduced in
figure. It is clear that the Chapter 2) . We had seen earlier (Sec. 1.7) Fig. 5.23 Complementary
that shear stresses occur in complementary shear stress on the left face
beam will have shear
forces acting over its pairs: . Therefore, a stress component acts on the exposed
entire length. The x-face of the element (Fig. 5.23). The forces due to these shear stresses
presence of the shear acting on this face give the resultant shear force V at the section.
forces will cause the
bending moment to vary Fig. 5.21
along the length as 5.5 Relating shear stresses to the shear force in a
shown. In fact, this follows from the fact that the slope of the bending beam
moment line is equal to (negative of) the shear force value.
The discussion above leads us to the strategy that we adopt to relate shear
The presence of a positive bending moment will cause the beam to bend stresses to the shear force in a beam. We take a slice of the beam of
with a positive curvature resulting in compressive stresses in the upper length dx extending from a distance y (from the neutral axis) to the top
part and tensile stresses in the lower part of the beam. But since the surface of the beam (see Fig. 5.24a). Let the bending moment on the left
moment varies along the length of the beam, the bending stresses (at the face of this element be M, while that on the right face be M + dM (Fig.
same value of y) also varies with x. Thus, if 5.24c).
we take a slice of the beam (like the shaded
portion in Fig. 5.21), the bending stress The bending stresses due to these bending moments at the two locations, x
distributions on its two ends are entirely and x +dx (and vertical location y’ from the neutral axis) are then given
different resulting in a mismatch of by:
horizontal forces. ( )
( ) , and ( )
This fact is illustrated by drawing a free Fig. 5.22 FBD of a slice of
the beam of Fig. 5.21 The net axial
body diagram of this shaded portion as
shown in Fig. 5.22. The (compressive) bending stresses on the right end forces on the two
vary linearly in the vertical direction. These would be given by equation sections are:

,
∫ , and
where y is measured from the neutral axis of the section. There are no
stresses on the left end since the bending moment there is zero. The ( )

bending stresses alone then give rise to a net horizontal force to the left.
This force tends to slide this slice to the left which results in the where the
integration is to

83

Fig. 5.24 Bending stresses on an elemental slice of a beam


be carried out on area A’, which is the area of the elemental section from Here V is the shear stress, Q is the first moment (about the neutral axis) of
y upwards. Note that at x, the positive ζ is directed in the negative x- the section area above y, b is the width of the section at location y, and Izz
direction (the outward normal there being in the negative x-direction) and, is the second moment of the whole area. It can
therefore, F,x (the force, by convention, being positive in positive be shown easily that we can write the following
coordinate direction) is obtained as the negative of the integral of stresses. expression for Q in terms of the distance ̅ of the
However at x + dx, the positive stresses are in the positive coordinate centroid of area A’ from the neutral axis (Fig.
direction and, hence, F,x+dx is the integral of the stresses on that face. 5.25):
The presence of a shear force on this element causes the bending ( ) ∫ ̅
moments on the two sides of the element to be different, resulting in
(5.16) Fig. 5.25
different values of the axial forces on either side. The net axial force is
given by The following examples illustrate the procedure
for calculating the shear stress distribution across beam sections.
.∫ /
Example 5.7 Shear stress distribution across a uniformly loaded
where y’ is the distance measured from the neutral axis and, thus, can be cantilever beam
replaced with y. The integral above is the first moment of area A’ about
the neutral axis, where the area A’ is the area of the section above the Consider a cantilever beam loaded uniformly with a load intensity of 100
location where we need to determine the shear stress (see Fig. 5.23). This N/m as shown in Fig. 5.26. The beam has a rectangular section of width
is given the symbol Q. 10 mm and depth 30 mm. Find the distribution of shear stresses at the root
section.
(5.14)
Solution:
This difference in axial forces is balanced by the shear stresses acting on We first determine the reaction at the built in support. By vertical force
the bottom surface of the element (Fig. 5.24d). If we assume the average balance the vertical reaction is determined to be 200 N upwards. To
stress on this surface to be ηyx (= ηxy), the net shear force on this face is determine the reaction moment at root, we replace the distributed load by
ηyx×(b×dx), where b is the width of the section at y. Since ηyx is positive in its statically-equivalent
the negative x-direction, this force is in the negative direction. Writing the concentrated load of 200 N
axial force equilibrium of the FBD of Fig. 5.24d, we get at the centroid of the
( ) , distributed load, which is
at 1 m point. The moment
balance equation then
or ( )
gives the reaction moment
as 200 N.m counter-
We replace dM by ‒Vdx (realizing from Eq. 4.2 that ) to obtain clockwise.
( ) To draw the FBD, we start
Fig. 5.26 A uniformly loaded cantilever beam
at zero. The shear force and the corresponding SFD and BMD
Thus, the average shear stress is given by jumps down 200 N due to
( ) ( ) (5.15)

84
the concentrated force at x = 0. Thereafter, the SFD increases at a Fig. 5.27 shows graphically the shear stress distribution70 across the beam
constant rate of 100 N/m due to the uniformly distributed load of the same section.
density, till it reaches the value of 0 at the free end of the beam.
This shear distribution results in a similar shear strain distribution across
To calculate the shear stress, we do not need to calculate the bending height of the beam. Fig. 5.28 shows an exaggerated view of the
moments. However, we do so here just to illustrate again the area method deformation of the beam under the action of shear forces.
of drawing the BMDs. After starting at 0, the BMD jumps down to 200
N.m because of the positive reaction moment of the same magnitude. Example 5.8 Shear stress distribution on wide-flanged I-beam
Then, due to a negative shear force which is constantly decreasing in Consider a simply-supported overhang beam with uniformly distributed
magnitude, the bending moment load as shown in Fig. 5.29. The beam section is a wide-flanged I-
increases with a linearly decreasing section71. Obtain the variation of shear stresses in the beam section.
rate, i.e., the BMD has a parabolic
shape as shown in Fig. 5.26. Total Solution:
change in BM (as per Eq. 4.2) over
the length of the beam is equal to the As with all statically-determinate beams, we start with the loading
area under the SF curve over the diagram of the
same length. This is 200 N.m, and beam, calculate
therefore, the BM at the free end the reactions at
turns out to be 0, as it should be. Fig. 5.27 Shear stress distribution on the support and
a rectangular section draw the SFD of
The maximum shear force is V = the beam using
‒200 N nd occurs t the root of the be m. The she r stress t dist nce the method of
y from the NA is given by Eq. 5.16 as ( ) . Because the section of areas. Because
of a uniformly
the beam is rectangular (b =10 mm, h = 30 mm), its Izz is given by bh3/12
distributed load
or 2.25×10‒8 m4, and the NA is located at 0.015 m from the top. The
of 500 N/m, the
value of b, the width of the section at a distance y from NA is constant at
SFD has a line
̅ . Fig. 5.29 A simply-supported overhang beam with uniformly
0.010 m. The value of Q is obtained from Eq. 5.16: ∫ sloping up at the distributed load
Here A’ represents the area above the location y (shaded area in Fig. same rate. But
5.26). Here ̅ ( ) , and therefore,
( )( ). / (
70
) . The shear stress, then, is Let us compare the magnitudes of the shear stresses with those of the bending
( ) ( ) stresses in this simple case. The maximum bending stresses are of order (
( ) (
( ) ( ) ) , while the maximum shear stresses are of order . Therefore,
) . . The maximum value of Q in this case is (b.h/2).h/4, and so
. It is reasonable to suppose that M is of order V.L, where L is
It is a parabolic distribution with the length of the beam, and therefore, . This ratio is small for
the maximum at y = 0, i.e., at the slender beams.
neutral axis, and going to zero at 71
The horizontal portions away from the central axis are termed as flanges while
the top and the bottom surfaces. the central vertical portion of a beam is termed as web. A wide-flanged I-beam is
also termed as a W-beam
85
Fig. 5.28 Distortion of a rectangular
beam due to a shear force
the presence of the two reactions (750 N each) makes the SF line jump at y = 0, and is given by ( )
down by 750 N at the two supports. The resulting SFD is as shown. ( )( )
The BMD has also been drawn in Fig. 5.29 even though we do not require )( ) .
it for the purposes of this example. The value at y =0.01 m is obtained
by using 6×10‒6 m3 for the value of (a) (b)
The maximum positive shear force of 375 N magnitude occurs just before Fig. 5.31
Q, and is obtained as 0.77 MPa.
the two supports, while the maximum negative magnitude occurs just
after the supports. The maximum shear stresses will also occur at these In calculating this last value, we have used
same locations. We calculate the shear stresses using Eq. 5.15. We 0.01 as the value of b, the width of the section.
require for this purpose the value of Izz. This is obtained by dividing the A value of 0.77 MPa is actually the value of
section area into three rectangles A, B, and C, as shown in Fig. 5.30. The shear stress at a point slightly below y = 0.01
symmetry of the section ensures that the centroid of the section is in the m where we can take the value of b to be 10
middle. mm. At a point slightly above this point, the
width of the section suddenly increases by a
We use the parallel axes theorem to calculate Fig. 5.33 Shear stress ηxy
factor of 4 to 0.040 m and, therefore, there is a distribution in an I-beam
Izz. discontinuity in the value of shear stress which
, ( ) - decreases by a factor of 4 to 0.19 MPa. This has been shown in Fig.
( ) , 5.32c.

or Izz =2.93×10‒7 m4. Fig. 5.33 shows the distribution of shear stress ηxy across the section. It
may be noted that the web of the section is the primary shear-bearing
To calculate the value of Q across the section, element in a wide- flanged beam. The value of the stress ηxy in the flanges
Fig. 5.30 I-section divided up
first consider the shaded area at a distance y in three rectangles is quite low.
from the NA as shown in Fig. 5.31a. The value
of Q for this is given by ( ) ( )( ),
( ) - or ( ) valid for y between 5.6 Shear flow in beams
0.010 m to 0.020 m. Thus, the value of Q across the flange varies It was shown above that the shear stress ηxy in the flanges of the wide-flanged I-
parabolically from 6×10‒6 m3 at y = 0.01 m to 0 at y = 0.2 m. beam of the previous
example is quite low. But
For y between 0 and 0.010 m, the shaded area (Fig. 5.31b) is divided in it can be shown that the
two parts. The contribution of the top part is clearly the value of Q for y = flanges carry ηxz as well.
0.01 m, and the contribution for the second part is (0.01 m –y)(0.010 To evaluate ηxz, consider
m)×[y + (0.01 m –y)/2], or (0.5×10‒6 ‒ 0.005y2) m3. Thus, the value of Q an area at one corner (at a
in the web is ( ) , which is again parabolic, with distance z from the
a value of 8.5×10-6 m3 at y = 0 and 6×10‒6 m3 at y = 0.01 m. Fig. 5.32b vertical axis of
shows the variation of Q. symmetry) as shown in
Fig. 5.34. The FBD of Fig. 5.34 An element and its FBD to determine the
The shear stress ηxy can now be this area is also shown. shear stress ηxz
determined from Eq. 5.15. At the Take a small area element
location of the maximum shear force dA in the back face of the section. Let this area be
(= 375 N), the maximum shear occurs located at a distance y from the NA. The tensile
(a) (b) (c) stress acting on this area where the bending moment
86
Fig. 5.32 (a) The I-section, (b) the
variation of Q, and (c) the
variation of the shear stresses η
across the section. Fig. 5.35 Shear flow in
an I-beam
is M c n be t ken to be ‒My/Izz, where Izz is the second moment of the section tips and is maximum at the junctions with the web72.
re . The tot l force on this element is ‒(My/Izz).dA. The total force on the back
face is then the integral of this over the area of the face. We can , in a similar It is pertinent to point out here that the shear flow in bending has been
fashion, calculate the force acting on the front face of the area. It will be exactly obtained from purely equilibrium considerations. We have not used any
the same, except that M will be replaced by M + dM, and that this force is other tool for these derivations.
backwards. The net force due to bending stresses on the two faces is the
difference of the two, i.e., the integral over the area of (dM/Izz)ydA. The integral Example 5.9 A built-up beam
of ydA is the first moment of area which has been given the symbol Q. Thus, the
Consider the built up beam shown in Fig. 5.36. It is held together with 10
net force due to bending stresses is QdM/Izz. The change in moment, as before,
can be replaced by ‒Vdx, where V is the shear force at the section. mm bolts as shown. If the maximum shear force acting on the beam is 50
kN and the maximum shear stress a bolt can resist is 80 MPa, determine
This backward force is balanced by the shear force acting on the front the maximum spacing s between the bolts.
face (of area t.dx) of this element. If ηxz is the average shear stress on this
area Solution:
. To externalize the shear forces in the bolts, we consider the FBD of an
element of length dx as shown in Fig. 5.37. Because of the presence of
From this we obtain
the shear force, the bending moments, and therefore, the bending stresses
(5.16) on the two opposite faces of this
element are different. The
This is same as Eq. 5.15 with section width b replaced with t, the
difference in the axial forces
thickness of the flange.
because of this difference results
It is convenient to use the concept of shear flow q first introduced in Sec. in a net force on the element due
3.7 as ∫ . Thus, both for flange as well as web, we can to the bending stresses. This
force is resisted by the shear
write
action Fs,b in the bolts. But there Fig. 5.37
(5.17) are two sides to this element.
The unbalanced force is, therefore, resisted by 2Fs,b.
Fig. 5.35 shows the shear
The unbalanced force is determined by Eq. 5.14 as
flow distribution for an I-
beam subject to a negative
shear force V. If the shear
force was positive, all the Each bolt resists this unbalanced load for a length s of the beam, where s
arrows would have been is the distance between the bolts (i.e., the pitch).
reversed. The shear flow in
the flanges starts at zero at the .

Fig. 5.36 A built-up beam

72
The stress distributions at the junctions are quite complicated. It is for this
reason that the commercially available rolled-sections are provided with
generous fillets to avoid stress concentration.
87
The value of Izz is evaluated by breaking up the beam section into a The maximum value of the shear flow can be determined from Eq. 5.17 as
difference of two areas: a rectangle 140 mm wide, 160 mm deep, from VQ/Izz. The Q here is the first moment about neutral axis of the area of
which is subtracted a rectangle 120 mm wide, 140 mm deep. The value the upper flange, which is (bt)h/2. The value of Izz is found by breaking
of Izz is up the area into three parts, determining the I of each about its own
centroidal axis, and then transferring it to the neutral axis using the
( )( ) ( )( )
parallel-axes theorem:
, ( ) - ( ) .
The first moment of the area of the face of the FBD about the NA is
The maximum value of q is then Vbht/2Izz. The total horizontal shear
obtained by multiplying the area (10 mm × 50 mm) by the distance of its
force F1 in the flange is the area under the shear flow triangle, which is
centroid from the neutr l xis (80 mm ‒ 5 mm). Thus, Q = 3.75×10‒5 m3.
½qmax.b, or F1 = Vhb2t/4Izz, pointing to left. The lower flange has the
The maximum shear stress in a bolt is prescribed as 80 MPa. Therefore, same value of shear force, except that it is pointed to the right. The shear
( ) force in the web F2 is equal the sectional shear force V.
maximum value of Fs,b is . /( ) . Using this
value in the equation above along with V = 50 kN, we get The resultant force on the section is now V, but the pair of forces F1
produces a couple. Fig. 5.38(c) shows a statically equivalent force system
( )( )
in which the force V is displaced to a point O, a distance e to the left, with
, or . The point O is termed as the shear
This gives a value of s as 0.136 m. Thus, the spacing between the bolts centre of the section. As is shown in Fig. 5.39 the channel bends without
should be less than 13.6 cm. twisting only when the applied load passes through the shear centre.

5.8 Plastic deformations in beams


Consider a beam in pure bending
5.7 Shear centre which is loaded beyond the onset of
yielding. Let the beam be made of a
Let us next consider
perfectly elastic-perfectly plastic
beams with sections that
material whose behaviour is as
have symmetry only
depicted in Fig. 5.40. The yield stress
about one axis, like a
of the material is ζY, and the
channel section shown in
(a) (b) (c) corresponding strain is εY. As the
Fig. 5.38. Let the shear
bending moment applied to the beam Fig. 5.40
force on the section be V. Fig. 5.38 (a) shear flow distribution; (b) resultant
increases the magnitude of the strain
We can easily find the forces in the three legs of the section; (c) point of
at any point increases linearly with the bending moment. The strain for
shear flow distribution action of the statically-equivalent shear force
any applied moment varies linearly with y according to Eq. 5.1:
along the three legs of the section.
These are shown in Fig. 5.38a. The (5.1)
shear flow in the flanges increases
from 0 at the tips to the maximum
at the junction with the web.
(a) (b)
Fig. 5.39 (a) The channel twists while 88
bending when the load does not pass
through the shear centre b) The
channel bends without twisting when
the load acts through the shear centre.
And as long as the strain The value of yY is 0 when the plastic
is less than the yield region spans the whole depth of the beam.
strength value of εY, the The beam in this condition is resisting the
bending stress variation is maximum possible moment it can resist.
given by Eεxx, where E is Thus, the maximum bending moment
the elastic modulus of the Mmax that the beam can support is obtained
material. Under these (a) (b) (c) by setting yY = 0 in Eq.5.19:
conditions, the stress Fig. 5.41
distribution is as shown (5.20)
in Fig. 5.41a. The value of ρ is found by considerations of equilibrium, so The value of ρ is easily determined in
that the stresses are given by Eq. 5.5 as . Fig. 5.42 Variation of bending
terms of yY by noting that the strain at yY
moment with curvature in elastic-
This is valid as long as εmax < εY, and consequently, ζmax < ζY. The Value is ‒ Y, so that , or . plastic deformation of a
rectangular-section beam
of bending moment MY for which the maximum stress reaches the yield Combining this with the fact that the
strength value is obtained from Eq. 5.6 as
( )
, where h is the radius of curvature ρY at the point where the plastic deformation just starts
is given by , we get . Using this in Eq. 5.19, we get
height of the section.
For a rectangular b×h beam, this gives [ . / ], for ρ < ρY. (5.21)
(5.18)
Fig. 5.41 shows the variation of bending moment with the radius of
For values of bending moment larger than MY, the material starts curvature of the beam for a rectangular-section beam. The maximum
deforming plastically at the either end. Though the strain distribution is bending moment approaches a value 1.5 times MY asymptotically. The
still linear, the bending stress in no longer linear with y. The value of the ratio of the maximum bending moment to the yield bending moment
stress acquires the value ζY at a value of y less than h/2 as shown in Fig. changes with the section shape. For circular sections, this ratio is 1.7, and
5.41b and remains constant thereafter. If the radius of curvature is ρ for for thin-walled circular tubes it is about 1.3. For typical commercially
this case, then ( ) , and ( ) till its value is ζY, available I-beams, this ratio varies between 1.1 and 1.2.
after which it remains constant at ζY. The value of y at which ζ first
becomes ζY is . We denote by yY. Thus, the beam is elastic 5.9 Strain energy in bending
within the r nge ‒yY to +yY, and plastic outside this range. The value of ρ
is as yet undetermined. We evaluate the bending moment by integrating The expression for strain energy in a structure subject to arbitrary stresses
ζdA over the whole area. is given by Eq. 2.17 as

∫ ( )
∫ ∫ ∫ . / ∫ ( ) (2.17)
where the integration is carried over the entire volume of the beam. For
or, [ . / ] [ . / ] the case of a beam in pure bending (no shear force and bending moment
constant over the entire length), there is only one stress, ζxx given by Eq.
(5.19)
5.5 as . The strain energy for such a beam is, therefore
89
∫ ∫ . / .∬ / ∫ . / ( ) for 0 < x < L/2,
(5.22) for L/2 < x < L (a)
For a uniform beam subject to a constant bending moment, this equation And ( ) for 0 < x < L/2,
reduces to . ( ) for L/2 < x < L
In a more general case when a beam is also subjected to transverse shear, (b)
Eq. 2.17 gives the value of strain energy as: The contribution of bending stresses to the total energy is given by
∫ ( ) , or ∫ , where A is the cross-sectional area of the beam.

∫ . / Expressing ζxx in terms of M(x), and replacing the value of M(x) using Eq.
(5.23) b above, we get
( ) ( )
It was shown in Example 5.7 that the ratio of the maximum shear stress to ∫ ∫ ∫ ,
the maximum tensile stress in the case of a uniformly loaded cantilever
beam was . It can be shown that the same holds true to And expressing Izz in terms of b and h, we get
almost all beams and loadings. This ratio is small for typical slender
beams. Since the stresses are squared in Eq. 5.23, the contribution of
shear stresses to the strain energy is quite small compared to that of the
It can be verified that this has the dimensions of energy. Please note that
bending (tensile) stresses and can, therefore, be safely neglected.
we have not included the energy due to shear which is an order of
Example 5.10 Strain energy of a simply-supported beam magnitude smaller for slender beams.

Consider the simply-supported


beam of length L loaded at the mid- Summary
point as shown in Fig. 5.43. Find
 A beam is a slender member which carries transverse loads or
the strain energy of the beam if the
moments which tend to bend it.
section of the beam is rectangular
o A beam resists bending moments by developing axial
with depth h and width b.
stresses. These stresses arise because curving of a beam
Solution: necessarily implies contraction of material on one side
and elongation on the other side.
The reader can verify that the SFD o There exists a plane within the beam which undergoes
and the BMD of the beam are as neither contraction nor extension. This is termed as the
Fig. 5.43
drawn in the figure. Since we need neutral plane. The trace of the neutral plane within a
to integrate the squares of shear stresses and bending stresses over the section of the beam is termed as the neutral axis.
length of the beam, let us express SF and BM as functions of the  To obtain a relation between the curvature of the beam, the
longitudinal variable x. bending stresses and the moment causing the bending we adopt
It can be seen that we can write: the following strategy:

90
o We first use the geometry of bending in the case of pure  The presence of shear forces causes the variation of bending
bending to relate strains to the distance from the neutral moments along the length of a beam resulting in an unbalance of
plane. axial forces. This gives rise to shear stresses ηzx or ηxz. The shear
o We convert the strains to stresses using the elastic stresses are given by Eq. 5.15 as ( ) ( ) ,
modulus. where Q is the first moment of the area of the section above the
o We next use the equilibrium conditions. The balance of location y, and b is the width of the section at y. The shear stress
axial forces gives us the location of the neutral axis as is the maximum at the neutral axis and decreases towards either
passing through the centroid of the section area. The end of the section.
balance of moments then relates the curvature in the  The shear stresses in slender beams are an order of magnitude
beam to the bending moment acting on it. Eq. 5.6 gives smaller than the bending stresses.
the relation between the moment Mb, the radius of  It is convenient to use the concept of shear flow q as
curvature ρ, the strain εxx, and the bending stress ζxx.
 The bending stresses vary linearly with distance from the neutral ∫ . Thus, both for flange as well as web, we can write
axis, zero on the axis and increasing as we move away. There is .
compression on one side and tension on the other side of the  The concept of shear centre was introduced as a point within the
neutral axis. beam section through which the external load must pass for the
 We introduce the concept of bending rigidity as the curvature beam to bend without twisting. It is the point where the single
produced per unit bending moment. The bending rigidity for a shear force which is statically equivalent to the sectional shear
uniform beam in pure bending is seen to be the product of the distribution must be applied.
elastic modulus E and the second moment Izz of the cross-  The strain energy in slender beams is dominated by the energy
sectional area about a transverse neutral (centroidal) axis. The due to bending stresses and is given by ∫ . / .
rigidity of a beam increases as the third power of the depth of a
beam.
 It was shown through some solved examples that for the beam
sections of the same areas the beam with more material away
from the centroid is more rigid.
 The method for handling of composite beams was developed
following the same process as for simple beams. It was shown
that strains in composite beams are determined by geometric
considerations alone as in the case of simple beams, and are
continuous across different materials. The stresses, however, are
not continuous. It was shown that the location of the neutral axis
is obtained by a relation that weights the areas of different
materials with their elastic moduli (Eq. 5.10). The flexural
relation is the same as that for a simple beam except that the
bending rigidity is now replaced by the sum of the bending
rigidity of the constituent areas. The process was explained using
the example of a steel-reinforced concrete beam which exploits
beautifully the differing capabilities of the two materials.
91
92
Example B.1 Centroid of a T-section
Consider the T-section shown in Fig. B.2.
Find the location of the centroid.

Solution:
The x location of the centroid is
immediately obtained by using symmetry.
Appendix B Properties of Areas Thus, xc = 2.5 units. The y-coordinate of
the centroid can be obtained by using Eq.
B.3.
We introduce here a very convenient Fig. A.2 T-section
B.1 First moments of area and centroid method for determining the centroid of areas that are made up of simple
areas. Thus, we divide the T-section in to two rectangular areas, a 1×4
Consider an area as shown in Fig. B.1. Two first moments of areas are
unit area labelled A, and a 4×1area labelled B. We know that centroid of
defined: one about the x- axis, and the other about the y- axis.
area A is located at its mid-point, i.e., at y = 4.5 units. The centroid of
∫ , and ∫ (B.1) area B is located at its mid-point, i.e., at y = 2.0 units. The location of the
centroid of the composite area is given by
The values of Qx and Qy depend on the location of the origin O.
(∑ ) ∑ (B.4)
The centroid of the area is defined as the
location of the origin such that Qx and Qy Using this formula, we get
vanish. We can use this property to determine ( )
units
the location of the centroid.
Let the coordinate of the origin in the given The location of the centroid is shown in the figure. The centroid is
coordinate system be xc and yc. We calculate the located nearer the top since the area is top heavy.
first moment of area about this point. The
distances y and x in Eq. B.1 are the replaced by Fig. A.1 B.2 Second moments of area
(y – yc) and (x – xc), respectively:
A second moment of area is a moment in which two lengths are used. We
∫ ( ) , or define three second moments73:

∫ ∫ , or 73
The nomenclature of the second moments has two indices, each indicating the
∫ axis from which the distance is taken. Thus, Izz uses the distance y from the z
(B.2) axis twice, Iyy uses the distance z from the y axis twice, and Iyz uses the distance y
from the z axis as well as the distance z from the y axis. This last is also called a
∫ cross moment of area.
And similarly, (B.3) The polar moment of area used in Chapter 3 is also a second moment where the
distance of the area is measured from the polar axis, i.e., the axis normal to the
area. It will be labeled Ixx in the convention used here:
93
∫ ; ∫ , and ∫ (B.5) Solution:
We can, using symmetry, see that the centroid will be at the centre of the
The dimensions of these second moments are L4, and its SI units are m4.
section.
Example B.2 Second moment of a rectangular section
∫ ∫ ∫
Determine the second moment of area of the rectangular section shown in
Fig. B.3 about its centroidal axis. Recognizing that the first integral is the Ixx of the outer square (=
a.a3/12), and the second integral is the Ixx of the inner square (= b.b3/12),
Solution: we get the Ixx of the section as ( ) .
The centroid of this symmetric section obviously This is a general method of determining the moments of composite
lies at the middle of the area. We take the sections.
coordinate axes with the centroid as the origin.
Let us take a rectangular elemental area of
dimensions δy and δz. Then, by Eq. B.5: B.3 Parallel axes theorem
Fig. A.3 A
rectangular section We now introduce a very important property of the second moments of
∫ ∫ ∫ area which is very handy in calculations of
the moments. Refer to Fig. B.6. Consider
Carrying out the indicated integrations, we get:
First a coordinate system x-y with the
(B.6) origin O and x-axis passing through the
centroid. The moment in this coordinate
Similarly, .
system is given by
We next calculate these moments about the y-axis
that coincides with the bottom plane of the ∫
section. Fig. A.4 Fig. A.6
Next consider an arbitrary coordinate
∫ ∫ ∫ system x’-y’ with the x’-axis located a distance d away from the
centroidal axis. The second moment in this new system is

four times the value when the x-axis passes through the centroid. Clearly, ∫ ∫ ( ) ∫ ( )
the value depends on the location of the x-axis. We shall show below that
that the value with the x-axis passing or, ∫ ∫ ∫
through the centroid is the minimum of all
possible values. Since y is measured from the centroidal axes, ∫ is immediately
Example B.3 Second moment of a seen as Ixx, ∫ is the first moment of area about a centroidal axis and
hollow square section
hence zero, and ∫ is nothing but the total area A. This gives:
Determine the second moment of area of
the hollow square section shown in Fig. B.5 (B.7)
Fig. A.5 Hollow square section
about its centroidal axis.
94
This is known as the parallel axes theorem. The second moment about an Example B.5 Second moment of an angle section about the
axis a distance d away from the centroid is equal to the moment about the centroidal axis
centroidal axes plus area times the square of d.
Consider the angle section shown in Fig.
One can immediately see from this theorem that the second moment about B.8. Determine its second moment about
the centroidal axis is the minimum. This theorem provides a very the centroidal axis.
convenient tool to obtain moment of inertia of a composite area as is
illustrated below. Solution:
We first determine the location of the
Example B.4 Second moment of a T-section about the centroidal
centroid of the section. We work with the
axis
division of the section in two rectangles A
Consider the T-section of Example B.1. Determine its second moment and B as shown. The area of A is 400 mm2
about the centroidal axis. and its centroid is at (5 mm, 30 mm) and
the area of B is 500 mm2 and its centroid is Fig. A.8 An angle section
Solution: at (25 mm, 5 mm). The coordinates of the
The centroid of the section was determined in Example B.1 to be located centroid are:
on the centre line a distance 3.25 units up
from the bottom of the T. The origin of

the axes is now taken at this location as ∑
mm,
shown.

We split up the area of the section into two and ∑
mm.
rectangles A and B as before (Fig. B.7).
The location of the centroid has been marked on the figure.
Here, too, we calculate the second moment using the parallel axis
We can calculate the second moment of Fig. A.7 theorem. We first calculate the moments of the two areas about their own
area of A about its own centroidal axes (at centroidal axis, and the transfer them to the centroidal axis of the
a distance of 4.5 units from the bottom) using Eq. B.6 as (5×13)/12 = composite section:
0.417. We then use the parallel axis theorem (Eq. B.7) to shift the axis to
,( ) ( ) -
3.25 units above the bottom (from 4.5 units above the bottom. Thus, the
shift distance d is 4.5 – 3.25 = 1.25, and the value of becomes 0 ( ) 1
0.417 + (Area = 5×1)×(1.25)2 = 8.23 units. Similarly, the second
moment of area B about its own centroid (which is 2 units from the 0 ( ) 1 mm4.
bottom) is 1×43/12 = 5.33 units. When we transfer it to the centroid of the
whole section which is a distance d (= 3.25-2.0 = 1.25 units) away, it
becomes (Area = 5×1)×(1.25)2 = 13.14 units. Thus, the
net moment is units.

95
Thus, the second moment of this angle section about the centroidal axis the polar moment of area ∫ . But . Therefore,
has been determined as 2.99×10÷7 m4.
∫ ∫ ∫ . Thus,
Table A.1 Properties of some areas (B.8)
This is the perpendicular axes theorem which states that the polar moment
Geometry Area Vertical Second moment of
of inertia of a section is the sum of the second moment of the area about
location area about
two perpendicular axes within the plane of the area.
of centroidal axis
centroid Example B.6 Second moment of a circular section
Fig.
3 A.8
Rectangle bh h/2 bh /12 Determine the second moment of a circular
section.

Solution:
Isosceles triangle bh/2 h/3 bh3/36 Consider the circular section of Fig. B.9. We
need to determine ∫ . Direct
determination of this integral is not easy.
Therefore, we use the perpendicular axes Fig. A.9 A circular section
theorem (Eq. B.8):
Circle πD2/4 D/2 πD4/64

The symmetry of the circular section gives us ensures that the moments
and are equal. Thus, . The polar moment had been
evaluated in Sec. 3.3 as (Eq. 3.6). Therefore,
I-beam [2k1+(1‒2k1)] bh h/2
,(
) (

)-

B.4 Perpendicular axes theorem


Consider an area as shown in Fig. B.8. Let us consider a right-handed
triad of coordinate axes with x- and y- axes within the plane of the section
and the z-axis pointing outwards from the paper. Using the definitions of

96
Index
area method, 64 distributed load, 59 plastic deformation in torsion, simply-supported beam, 58
area, cross moment of, 93 elastic limit, 29 51 spring, kinky, 22
area, first moment, 84, 93 elastic modulus, 5, 17 plastic deformations in beams, St. Venant principle, 8
area, polar moment, 43 electroplating, residual stresses 89 statically equivalent loads, 8
area, second moment, 43, 93 in, 27 point load, 58 statically indeterminate
axis, neutral, 73, 91 energy methods, 33 Poisson ratio, 23 problems, 20, 37
beam, 56 energy, strain, 33 polar moment of area, 43 statically indeterminate shaft,
beam supports, idealized, 58 first moment of area, 84, 93 pre-stressing, 32 47, 54
beam, built-up, 87 gauge length, 29 proportional limit, 29 strain, 5
beam, composite, 78 generalized Hooke law, 24 radius of curvature, 73 strain energy, 33
beam, plastic deformations in, Hooke law, generalized, 24 rectangular section, second strain energy, bending, 90
89 Hooke, Robert, 5 moment of, 94 strain energy, elastic body, 38
beam, sandwich, 80 idealized beam supports, 58 reinforced concrete beam, 81 strain energy, torsion, 53
beam, shear stresses, 82 idealized stress-strain curves, 30 right-hand thumb rule, 44 strain hardening, 30
beam, simply-supported, 58 key, 10 rigidity, bending, 74, 91 strain, bending, 73
beams, shear flow, 87 keyway, 10 rigidity, torsional, 43 strain, shear, 25
bearing stress, 5 limit analysis, 52 roller support, 58 strength, yield, 29
bending moment, 7, 56 limit load, 52 sandwich beam, 80 stress, 5
bending rigidity, 74, 91 linear elastic range, 29 second moment of area, 43, 93 stress at a point, 7
bending strain, 73 load, distributed, 59 second moment of rectangular stress on oblique planes, 11
bending stress, 73 load, point, 58 section, 94 stress, bearing, 5
bending, pure, 71 loading density, 59 shaft, composite, 49 stress, bending, 73
bending, strain energy, 90 method of sections, 61 shaft, statically indeterminate, stress, double-index notation, 12
built-in support, 58 moment, bending, 7, 56 47, 54 stress, shear, 6
built-up beam, 87 moment, twisting, 6 shaft, tapered, 45 stress, sign convention, 13
cable in a sleeve, 45 necking, 30 shear centre, 88 stresses, pressure vessel, 14
Castigliano theorem, 33, 40 neutral axis, 73, 91 shear flow, 50 support, built-in, 58
centroid, 74, 93 neutral plane, 72, 91 shear flow, beams, 87 support, hinged, 9
coefficient of thermal expansion, normal stress, 11 shear force, 56 support, pinned, 58
27 parallel axes theorem, 95 shear modulus, 26 support, roller, 9
coil spring, 53 Pascal, 7 shear strain, 25 symmetry arguments, 15
complementary planes, 12 permanent set, 29 shear strain in torsion, 42 tensile test, 28
composite beam, 78 perpendicular axes theorem, 96 shear stress, 6 thermal expansion, coefficient
composite shaft, 49 pinned support, 58 shear stress in torsion, 42 of, 27
concrete, pre-stressed, 32 plane, neutral, 72, 91 shear stresses in beams, 82 thermal strains, 27
concrete, reinforced, 81 planes, complementary, 12 shrink fit, 28 thermal stresses, 27
cross moment of area, 93 plastic deformation, 29 sign convention for shear force torsion, 41
deformation, plastic, 29 and bending moment, 57 torsion bar, 41
97
torsion moment diagram, 44 torsional strain, 42 two-force member, 9, 10 yield point, 29
torsion, plastic deformation, 51 torsional stress, 42 ultimate stress, 30 yield strength, 29
torsion, strain energy, 53 twisting moment, 6 unit force method, 35
torsional rigidity, 43 twisting moment diagram, 44 vibration isolator, 26

98

S-ar putea să vă placă și